Вы находитесь на странице: 1из 65

Grand Test - 36

Exam Duration - 31/2 hours Method of Marking in Pink OMR Sheet


1. The replication fork consists of four 9. Most specific and sensitive for identification is:
components. Which of the following components is a. Anthropometry b. Dactylography
responsible for synthesis of the short RNA c. Skull d. Pelvis
molecules essential for DNA replication?
a. Helicase b. Primase 10. Inhibition of the synthesis of ATP during
c. Polymerase d. SSB oxidative phosphorylation by oligomycin is
thought to be due to which of the following events?
2. Which of the following types of mutations a. Blocking of the proton gradient between NADH-Q
generally leads to a truncated protein? reductase and QH2
a. Deletion b. Frameshift mutation b. Blocking of the proton gradient between
c. Insertion d. Missense mutation cytochrome c1 and cytochrome c
c. Dissociating of cytochrome c from mitochondrial
3. First internal sign of putrefaction is founds: membranes
a. Below the liver b. In mediastinum d. Inhibiting of mitochondrial ATPase
c. Below the spleen d. In kidney
11. Lipogenesis occurs in which cellular
4. True statement about ethmoidal polypi is: compartment
a. Usually multiple a. Cytosol b. Endoplasmic reticulum
b. Usually arise from ethmoid cells in middle c. Golgi body d. Mitochondria
turbinate
c. Polypectomy is treatment of choice 12. Rigor mortis starts in:
d. All of the above a. Eyelids b. Heart
c. Voluntary muscles d. Limbs
5. Which two amino acids are most commonly
found in  turns in proteins? 13. Which is the rate-controlling step of pyrimidine
a. Arginine and lysine synthesis that exhibits allosteric inhibition by
b. Aspartic acid and glutamic acid cytidine triphosphate (CTP)
c. Leucine and valine a. Aspartate transcarbamoylase
d. Glycine and proline b. Hypoxanthine-guanine phosphoribosyl transferase
(HGPRT)
6. Hanging causes large amount of injury to: c. Thymidylate synthase
a. Vertebral artery b. Carotid A d. Ribose-phosphate pyrophosphokinase
c. Trachea d. Esophagus
14. During starvation, which of the following
7. In COPD there is chronic hypoxia that is shows the most marked increase in plasma
particularly obvious in vascular tissues such as the concentration?
lips or nail beds (cyanosis). Poorly perfused areas a. Free fatty acids b. Glucose
exposed to chronic hypoxia have decreased c. Glycogen d. Ketone bodies
metabolic energy for tissue maintenance and
repair. Which of the following is an important 15. Haemorrhagica external otitis media is caused
reason for this? by:
a. Increased hexokinase activity owing to increased a. Influenza b. Proteus
oxidative phosphorylation c. Staph d. Streptococcus
b. Increased ethanol formation form pyruvate on
changing from anaerobic to aerobic metabolism 16. A 58-yrs-old man has a brain lesion that causes
c. Increased glucose utilization via the pentose him to feel euphoric, laugh uncontrollably, and
phosphate pathway on changing from anaerobic to joke and make puns. Where is this brain lesion
aerobic metabolism most likely located?
d. Decreased ATP generation and increased glucose a. Fornix
utilization on changing from aerobic to anaerobic b. Right prefrontal cortex
metabolism. c. Hippocampus
d. Left orbitofrontal cortex
8. Which of the following reactions in glycolysis
consumes ATP? 17. A 55-yrs-old man comes to the physician with
a. Glucokinase b. Phosphofructokinase the chief complaint of daytime drowsiness. He
c. Phosphoglycerate kinase d. Pyruvate kinase states that although he goes to bed at 10 P.M. and
doesn’t get up until 6 A.M., he is chronically tired

DBMCI, Ph: 9810150067, 9868387223, 27568832, E-mail: drbhatiainstitute@rediffmail.com, Visit us at: www.dbmci.com 1
Grand Test - 36

and must take naps during the day. He wakes up are a sign of lymphoma. She also worries about
in the morning with a headache and a dry mouth. developing laryngeal cancer due to the second-
His wife states he snores loudly. Which of the hand smoke she is exposed to at home. For a
following is the most appropriate diagnosis? diagnosis of hypochondriasis, her symptoms
a. Obstructive sleep apnea should have been present for at least what period
b. Narcolepsy of time?
c. Central apnea a. 1 month b. 3 month
d. Recurrent hypersomnia c. 6 months d. 1 year

18. Myringotomy is indicated in: 25. In infant most sensitive audiometric screening
a. Coalescent mastoiditis b. Choleastetoma is:
c. ASOM d. External otitis media a. Electrocochleography b. BERA
c. Cortical evoked response d. Tympanometry
19. A 40-yrs-old woman is arrested by the police
after she is found crawling through the window of 26. Every 4 or 5 weeks, a usually well-functioning
a movies star’s home. She states that the movie and mild-mannered 35-yrs-old woman experiences
star invited her into his home because the two are a few days of irritability, tearfulness, and
secretly married and “It just wouldn’t be good for unexplained sadness. During these days, she also
his career if everyone knew”. The movie star feels fatigued and bloated and eats large quantities
denies the two have ever met, but notes that the of sweets. What is the most appropriate diagnosis?
woman has sent him hundreds of letters over the a. Cyclothymia
past 2 years. The woman has never been in trouble b. Borderline personality disorder
before, and lives and otherwise isolated and c. Dissociative identity disorder
unremarkable life. Which of the following d. Premenstrual dysphoric disorder
diagnoses is this patient most likely to have?
a. Delusional disorder b. Schizoaffective disorder 27. Which nonopioid medication can be used to
c. Bipolar I disorder d. Cyclothymia treat some of the symptoms of opioid withdrawal
syndrome?
20. A young man in psychoanalysis has recurrent a. Chlordiazepoxide b. Haloperidol
dreams of snakes shedding their skins. What are c. Methadone d. Clonidine
these images called by jungian psychotherapists?
a. Phallic representations b. Archetypes 28. Best view for evaluating sphenoid sinus is:
c. Illusions d. Primary process a. Water’s with open mouth. b. Schuller’s view
c. Towne’s view d. Lateral view
21. Trismus in parapharyngeal abscess is due to
spasm to: 29. Which of the following antipsychotic
a. Masseter muscle b. Medial pterygoid medications is the most potent?
c. Lateral pterygoid d. Temporalis a. Chlorpromazine b. Thiothixene
c. Trifluoperazine d. Haloperidol
22. A 27-yrs-old woman comes to a psychiatrist
with the chief complaint of feeling depressed her 30. A 25-yrs-old woman with bipolar disorder
entire life. While she states she has never been so develops a high fever with chills, bleeding gums,
depressed that she has been unable to function, she extreme fatigue, and pallor 3 weeks after starting
never feels really good for more than a week or on carbamazepine. Which of the following is she
two at a time. She has never been suicidal or experiencing?
psychotic, though her self-esteem is chronically a. Stevens-Johnson syndrome
low. Which of the following diagnoses is most b. Acute aplastic anemia
likely? c. Serotonin syndrome
a. Major depression b. Adjustment disorder d. Neuroleptic malignant syndrome
c. Cyclothymia d. Dysthymia
31. Pain along hip, back of thigh and loss of
23. All cause haemorrhagic conjunctivitis except: sensation along the lateral border of foot and
a. Ebola b. Entero – 70 decreased ankle jerk, site of lesion is:
c. Cox sackie virus – 24 d. Echo virus - 18 a. L4 – 5 b. L5 – S1
c. L3 d. L12 – T1
24. A 27-yrs-old librarian has been worried that
the small lymph nodes she can palpate in her groin

DBMCI, Ph: 9810150067, 9868387223, 27568832, E-mail: drbhatiainstitute@rediffmail.com, Visit us at: www.dbmci.com 2
Grand Test - 36

32. “First-generation” histamine H1 blockers such c. Radius, lower end d. Ulna, upper end
as diphenhydramine, phenothiazine antipsychotic 39. Von-Rosen’s splint is used in:
drugs (e.g. chlorpromazine), and tricyclic a. CDH b. CTEV
antidepressants (e.g., imipramine) have c. # shaft of femur d. Scoliosis
pharmacologic actions, side effects, toxicities, and
contraindications that are very similar to those of 40. A patient with edema fails to respond
which of the following? adequately to maximum recommended dosages of
a. Atropine b. Bethanechol chlorthalidone. Which of the following is the most
c. Isoproterenol d. Neostigmine appropriate and most fruitful next step?
a. Add hydrochlorothiazide
33. A patient with undiagnosed coronary artery b. Add metolazone
disease is given a medication. Shortly thereafter c. Replace chlorthalidone with furosemide
she develops intense tightness and “crushing d. Replace chlorthalidone with hydrochlorothiazide
discomfort” of her chest. An EKG reveals ST-
segment changes indicative of acute myocardial 41. A young boy is diagnosed with asthma. His
ischemia. Which of the following drugs is most primary symptom is frequent cough, not
likely to have caused this reaction? bronchospasm or wheezing. Other asthma
a. Clozapine b. Pentazocine medications are started, but until their effects
c. Phenytoin d. Sumatriptan develop fully we wish to suppress the cough
without running a risk of suppressing ventilatory
34. Trigger finger is caused by: drive or causing sedation or other unwanted
a. OA b. Rheumatoid arthritis effects. Which of the following would best meet
c. Tenosynovitis d. Injury to tendons these needs?
a. Codeine b. Dextromethorphan
35. A 64-yrs-old man with arteriosclerotic heart c. Diphenhydramine d. Hydrocodone
disease and CHF who has been treated with
digoxin complains of nausea, vomiting, and 42. About TMJ dislocation false is:
diarrhea. His EKG reveals a bigeminal rhythm. a. Forward displacement of condyle
The symptoms and EKG findings occurred shortly b. Rx is downward and backward push
after another therapeutic agent was added to his c. Causes pain
regimen. A drug interaction is suspected. Which of d. None of the above
the following add-on agents most likely provoked
the problem? 43. A patient develops acute gout. Which of the
a. Lovastatin b. Hydrochlorothiazide following is an accurate description of how uric
c. Phenobarbital d. Nitroglycerin acid causes the arthritic response?
a. Activated microtubular formation in leukocytes
36. Frequency (In MHZ) of USG used in b. Directly activated leukotriene B4 receptors
ophthalmics is: c. Has intrinsic tumor necrosis factor (TNF) activity
a. 8 b. 18 c. 15 d. 12 d. Mechanically damages articulating surfaces of the
joints
37. A 60-yrs-old woman with deep-vein thrombosis
(DVT) is given a bolus of heparin, and a heparin 44. Which would we administer for adjunctive
drip is also started. Thirty minutes later, she is management of a patient with hepatic portal-
bleeding profusely from the intravenous site. The systemic encephalopathy?
heparin is stopped, but the bleeding continues. You a. Diphenoxylate b. Lactulose
decide to give protamine to reverse the adverse c. Loperamide d. Omeprazole
effect of heparin. How does protamine act?
a. It causes hydrolysis of heparin 45. Mc Murrays test is for:
b. It changes the conformation of antithrombin III to a. Medial meniscus b. Lat meniscus
prevent binding to heparin c. Both A and B d. Post cruciate ligament
c. It activates the coagulation cascade, overriding the
action of heparin 46. A 60-yrs-old man on long-term therapy with a
d. It combines with heparin as an ion pair, thus drug develops hypertension, hyperglycemia, and
inactivating it. decreased bone density. Blood tests indicate
anemia. Stool samples initially were positive for
38. Gun stock deformity is seen in fracture of: occult blood and then developed a “coffee-
a. Lateral condyle b. Humerus, lower end grounds” appearance. Which of the following

DBMCI, Ph: 9810150067, 9868387223, 27568832, E-mail: drbhatiainstitute@rediffmail.com, Visit us at: www.dbmci.com 3
Grand Test - 36

drugs is most likely responsible for the patient’s 54. Invertogram to be done in a new born:
symptoms? a. Immediately b. After 2 hours
a. Beclomethasone b. Hydrochlorothiazide c. After 4 hours d. After 6 hours
c. Metformin d. Prednisone
55. Which of the following statements about small
47. An 82-yrs-old woman is brought to the intestinal motility is correct?
emergency room complaining of extreme thirst a. Contractile frequency is constant from duodenum to
and generalized weakness. She has consumed a terminal ileum
large amount of orange juice to quench her thirst. b. Peristalsis is the major contractile pattern during
Laboratory analysis reveals a significant feeding
hyperkalemia. Which one of the following changes c. Migrating motor complexes occur during the
in nerve membranes will most likely be observed? digestive period
a. The membrane potential will become more negative d. Contractile activity is initiated in response to bowel
b. The sodium conductance will increase wall distention
c. The potassium conductance will increase
d. The membrane will become more excitable 56. Contraction of the gallbladder is correctly
described by which of the following statements?
48. Trident hand is seen in: a. It is inhibited by a fat-rich meal
a. Mucopolysaccharidosis b. Achondroplasia b. It is inhibited by the presence of amino acids in the
c. Diaphyseal achlasia d. Chondrodysplasia duodenum
c. It is stimulated by atropine
49. The rate of diffusion of a particle across a d. It occurs in response to cholecystokinin
membrane will increase if:
a. The area of the membrane decreases 57. In X-ray, loops of bowel on left side of
b. The thickness of the membrane increase hemithorax and shift of heart shadow is seen in:
c. The size of the particle increase a. Eventration of diaphragm
d. The lipid solubility of the particle increase b. Foraman of Bochdalek hernia
c. Morganian hernia
50. Tubercular otitis media is characterized in: d. Any of the above
a. Multiple perforations of tympanic membrane
b. Pain in ear 58. Which one of the following is higher at total
c. Low grade fever lung capacity than it is at residual volume?
d. Haemorrhagic discharge a. Anatomical dead space
b. Maximum static inspiratory pressure
51. The intensity of colour in Doppler is c. Lung compliance
determined by: d. Airway resistance
a. Direction flow b. Velocity of flow
c. Strength of returning echo d. None of the above 59. Which of the following values will decrease in a
patient with ventilation-perfusion (V/Q)
52. The electrocardiogram is most effective in abnormalities?
detecting a decrease in which of the following? a. Anion gap b. Arterial pH
a. Ventricular contractility c. Arterial carbon dioxide tension
b. Mean blood pressure d. A – a gradient for oxygen
c. Total peripheral resistance
d. Coronary blood flow 60. Phase II blocker is:
a. D – TC b. Cocaine
53. The percentage of the total cardiac output c. Scoline d. Vencuronium
distributed to any single organ is most dependent
on: 61. Na+ is reabsorbed from the basolateral surface
a. The contractile state of the heart of the renal epithelial cells by which of the
b. The magnitude of mean blood pressure following?
c. The magnitude of diastolic pressure a. Na/H exchange b. Na-glucose cotransport
d. The ratio of an organ’s vascular resistance to total c. Na-K pump d. Facilitated diffusion
peripheral resistance (TPR)

You can see the Answers key of AIIMS, UPSC & AIPG
Weekly subject wise test & Monthly Grand Test are now
available Online also. For more details contact Mr. Amit: exam next day (after 8 PM) at our website:
09314183700 www.dbmci.com

DBMCI, Ph: 9810150067, 9868387223, 27568832, E-mail: drbhatiainstitute@rediffmail.com, Visit us at: www.dbmci.com 4
Grand Test - 36

62. Glomerular filtration rate would be decreased 71. Which of the following receptors is responsible
by which of the following? for measuring the intensity of a steady pressure on
a. Constriction of the efferent arteriole the skin surface?
b. An increase in afferent arteriolar pressure a. Pacinian corpuscle b. Ruffini ending
c. Compression of the renal capsule c. Merkel’s disk d. Meissner’s corpuscle
d. A decrease in the concentration of plasma protein
72. Inert particles are sensitized with either
63. Mendelson syndrome is due to: antigen or antibody. Which of the following tests is
a. Aspiration pneumonitis used extensively to detect microbial antigens
b. Chemical pneumonitis rapidly (five minute or less)?
c. Oesophagitis a. Coagglutination (COA)
d. Oesophageal spasm b. Counterimmunoelectrophoresis (CIE)
c. Enzyme-linked immunosorbent assay (ELISA)
64. The supraoptic nucleus of the hypothalamus is d. Latex agglutination
believed to control secretion of which of the
following hormones? 73. The obstetric conjugate diameter is the
a. Antidiuretic hormone distance between the sacral promontory and the:
b. Oxytocin a. Nearest point on the posterior surface of the
c. Growth hormone symphysis pubis
d. Adrenocorticotropic hormone b. Upper part of the inner surface of the symphysis
pubis
65. Which of the following is the principal steroid c. Apex of the pubic arch
secreted by the fetal adrenal cortex? d. Ilio-pectineal eminences
a. Cortisol b. Corticosterone
c. Dehydroepiandrosterone d. Progesterone 74. Elevated IgG and IgM antibody titers to
parvovirus suggest a diagnosis of which one of the
66. Drug causing anaphylactoid reaction: following?
a. Propofol b. Alcuronium a. Acute Lyme disease
c. Thiopentone d. Glycopyrrolate b. Fifth disease
c. Possible hepatitis B infection
67. Which of the following is a function of sertoli d. Possible subacute sclerosing panencephalitis
cells in the seminiferous tubules?
a. Secretion of FSH into the tubular lumen 75. A patient has developed an acute congestive
b. Secretion of testosterone into the tubular lumen glaucoma in his right eye. The prophylactic
c. Maintenance of the blood-testis barrier measure to be taken in the fellow eye is:
d. Synthesis of estrogen after puberty a. Topical steroids b. Peripheral iridectomy
c. Antibiotics d. Laser iridotomy
68. Astigmatism is:
a. Defect in curvature 76. Amebas that are parasitic in humans are found
b. Defect in thickness of cornea in the oral cavity and the intestinal tract. Which
c. Defect in refractive index one of the following statements best describes these
d. Defect in opacity of lens intestinal amebas?
a. Infection with E. histolytica is limited to the
69. A 10-yrs-old girl with type I diabetes develops a intestinal tract
neuropathy limited to sensory neurons with free b. They are usually nonpathogenic
nerve endings. Quantitative sensory testing will c. They are usually transmitted as trophozoites
reveal higher-than-normal thresholds for the d. They can cause peritonitis and liver abscesses
detection of which of the following?
a. Fine touch b. Vibration 77. Water content of liquor amnii is said to be
c. Pressure d. Temperature replaced every:
a. Hour b. Three hours
70. Hypothermia is used in all except: c. Ten hours d. 24 hours
a. Neonatal asphyxia b. Cardiac surgery
c. Hyperthermia d. Arrythmia 78. Tinea versicolor is caused by which of the
following?
a. E. floccosum b. E. werneckii
c. M. furfur d. M. canis

DBMCI, Ph: 9810150067, 9868387223, 27568832, E-mail: drbhatiainstitute@rediffmail.com, Visit us at: www.dbmci.com 5
Grand Test - 36

79. Which one of the following is transmitted by 85. Malaise and fatigue with increased “atypical”
the bite of a hard Ixodes tick? lymphocytes and a reactive heterophil antibody
a. B. (Rochalimaea) henselae b. C. trachomatis test are most commonly caused by which of the
c. C. burnetii d. E. chaffeensis following?
a. Borrelia burgdorferi b. Epstein-Barr virus
80. The distance between foetal vessels decreases c. Parvovirus d. Rubella virus
and syncytium cover of the villi thins out at the
ageing of placenta. This will enhance: 86. The treatment of choice in respect of secondary
a. Transplacental haemorrhage abdominal pregnancy when diagnosed at 28 weeks
b. Exchange of nutrients would be:
c. Deposition of calcium a. Immediate operation (laparotomy)
d. Deposition of fibrin b. To continue pregnancy to term and then operate
c. Induction by oxytocin drip
81. Whooping cough, can best be detected by d. To allow vaginal delivery at term
which of the following procedures?
a. Cold agglutinin test 87. Reverse transcriptase is an enzyme found in
b. Culture of respiratory secretions in HeLa cells after retroviruses and hepadnaviruses. Which one of the
centrifugation of the inoculated tubes. following is a function of the enzyme reverse
c. Culture of respiratory secretion on Regan-Lowe transcriptase?
agar a. DNAse activity
d. Direct microscopy of sputum by Gram stain b. DNA-dependent RNA polymerase activity
c. RNA-dependent DNA polymerase activity
82. Brucella is pathogenic for humans and animals, d. RNA-dependent RNA polymerase activity
which one of the following statements best
characterizes this organism? 88. Neisseria meningitidis, group B, is
a. It cause spontaneous abortion and has tropism for characterized by which one of the following?
placental tissue due to the presence of erythritol in a. -glutamyl polypeptide
allantoic and amniotic fluid b. Hyaluronic acid
b. It has a capsule of polyglutamic acid, which is toxic c. Outer-membrane proteins
when injected into rabbits d. Sialic acid polymers
c. It has 82 polysaccharide capsular types; capsule is
antiphagocytic; type 3 capsule (b-d-glucuronic acid 89. For a 30-yrs-old primigravida at 35 week’s
polymer) most commonly seen in injected adults gestation, head presenting, FHS 140, estimated
d. It secretes two toxins, A and B, in the large bowel weight 2.25 kg, liquor adequate, having BP 160/100
during antibiotic therapy mmHg, pedal oedema and no albuminuria, the
best treatment now would be:
83. If a pregnant woman develops jaundice and if a. Induction of labour
she is HbsAg positive, then the most appropriate b. Hospitalisation and close monitoring of BP and
management for the newborn at the time of FHS, and antihypertensive drugs
delivery would be: c. Elective caesarean section
a. No treatment d. Antihypertensive drugs and diuretic only
b. Administration of hepatitis B immunoglobulin
c. Administration of hepatitis B vaccine 90. A patient with a non obstructing carcinoma of
d. Administration of both hepatitis B immunoglobulin the sigmoid colon is being prepared for elective
and hepatitis B vaccine resection. To minimize the risk of postoperative
infectious complications, your planning should
84. A 70-yrs-old female patient was readmitted to a include:
local hospital with fever and chills following a. A single preoperative parenteral dose of antibiotic
cardiac surgery at a major teaching institution. effective against aerobes and anaerobes.
Blood cultures were taken and a gram-positive b. Avoidance of oral antibiotics to prevent
coccus grew-positive coccus grew from the blood emergence of clostridium difficile
cultures within 24-hours. Initial tests indicated c. Postoperative administration for 2 to 4 days of
that this isolate was resistant to penicillin. parenteral antibiotics effective against aerobes
Which of the following is the most likely and anaerobes
identification? d. Postoperative administration for 5 to 7 days of
a. Enterococcus b. Group A streptococcus parenteral antibiotics effective against aerobes
c. Group B streptococcus d. Neisseria species and anaerobes

DBMCI, Ph: 9810150067, 9868387223, 27568832, E-mail: drbhatiainstitute@rediffmail.com, Visit us at: www.dbmci.com 6
Grand Test - 36

91. Dopamine is a frequently used drug in is no evidence of intraabdominal sepsis. Correct


critically ill patients because: treatment includes.
a. At high doses it increases splanchnic flow a. Early reoperation to close the fistula tract
b. At high doses it increases coronary flow b. Broad-spectrum antibiotics
c. At low doses it decreases heart rate c. Total parenteral nutrition
d. In inhibits catecholamine release d. Somatostatin to lower fistula output.

92. Consider the following foetal heart rate 97. A 49-yrs-old woman undergoes surgical
patterns: resection of a malignancy. The family asks about
1. Late deceleration the prognosis. The histopathology is available for
2. Loss of beat to beat rhythm review. For which of the following malignancies
3. Variable deceleration does histologic grade best correlate with
4. Persistent early deceleration prognosis?
Ominous foetal heart rate patterns in a. Lung cancer b. Melanoma
cardiotocography would include: c. Colonic adenocarcinoma
a. 1, 2 and 3 b. 1 and 3 d. Soft tissue sarcoma
c. 2, 3 and 4 d. 4 alone
98. Friedman’s cervical dilatation curve:
93. While you are on duty in the emergency room, a. Has the shape of hyperbolic curve
a 12-yrs-old boy arrives with pain and b. Has maximum slope in the active phase
inflammation over the heel of his left foot and red c. Has a steep slope of descent of head in the latent
streaks extending up the inner aspect of his leg. He phase
remembers removing a wood splinter from the sole d. Shows none of the above features
of his foot on the previous day. The most likely
infecting organism is 99. A 57-yrs-old woman discovers a breast mass
a. Clostridium perfringens b. Clostridium tetani which is biopsied and found to be carcinoma. After
c. Staphylococcus d. Streptococcus her treatment, she assembles her family and
attempts to encourage them to obtain regular
94. A 25- year old woman arrives in the emergency examination and mammograms. Regarding the
room following an automobile accident. She is risk of breast cancer, which of the following
actually dyspneic with a respiratory rate of 60 / statements is true?
min. Breath sounds are markedly diminished on a. Breast cancer occurs more commonly among
the right side. The first step in managing the women of the lower social classes
patient should be to? b. A history of breast cancer in a first-degree family
a. Do bronchoscopy relative is associated with a fourfold increase in risk.
b. Draw arterial blood for gas determination c. Women with a first birth after age 30 years have
c. Decompress the right pleural space approximately twice the risk of those with a first birth
d. Administer intravenous fluids before age 18.
d. Cigarette smoking increases the risk of breast
95. Consider the following features: cancer.
1. Triple descending gradient of uterine activity
2. Resting intra-amniotic pressure of 15 to 20 mmHg 100. Glaucoma causes:
3. Start of relaxation simultaneously in all parts of the a. Secondary optic atrophy b. Cavernous optic atrophy
uterus c. Pressure optic atrophy d. No optic atrophy
The normal uterine contractions in labour are
characterized by: 101. A 52-yrs-old woman sees her physician with
a. 1 and 2 b. 1 and 3 complaints of fatigue, headache, flank pain,
c. 2 and 3 d. 1, 2 and 3 hematuria, and abdominal pain. Which is most
suggestive of her diagnosis?
96. A 22-yrs-old man sustains a gunshot wound to a. Serum acid phosphatase above 120 IU/L
the abdomen. At exploration, an apparently b. Serum alkaline phosphatase above 120 IU/L
solitary distal small-bowel injury is treated with c. Serum calcium above 11 mg/dL
resection and primary anastomosis. On d. Urinary calcium below 100 mg/day
postoperative day 7, small-bowel fluid drains
through the operative incision. The fascia remains 102. Which of the following statements regarding
intact. The fistula output is 300 mL/day and there cervical effacement and dilatation during normal
labour are correct?

DBMCI, Ph: 9810150067, 9868387223, 27568832, E-mail: drbhatiainstitute@rediffmail.com, Visit us at: www.dbmci.com 7
Grand Test - 36

1. Cervical effacement and dilatation occur from 107. A newborn has a midline defect in the
below upwards anterior abdominal wall. The parents ask what, if
2. In primigravida cervical effacement is followed by anything, should be done, Spontaneous closure of
cervical dilatation which of the following congenital abnormalities of
3. In multigravida cervical effacement and dilatation the abdominal wall generally occurs by the age of
occur simultaneously 4?
Select the correct answer using the codes given a. Umbilical hernia b. Patent urachus
below: c. Patent omphalomesenteric duct d. Gastroschisis
a. 1 and 2 b. 1 and 3
c. 2 and 3 d. 1, 2 and 3 108. Campimetry is:
a. Field of vision b. Acuity of vision
103. Sabitri, a 62-yrs-old woman has a lump in her c. Colour vision d. Includes all
neck. Which statement concerning radiation
induced tyroid cancer is true? 109. A 55-yrs-old woman with cancer of the cervix
a. It usually follows high-dose radiation to the head undergoes hysterectomy and is found to have
and neck. pelvic lymph nodes involved with cancer. She
b. A patient with a history of radiation is safe if no receives a course of external beam radiation (4500
cancer has been found 20 years after exposure. rads). When the physician counsels her prior to
c. Approximately 25% of patients with a history of her radiation treatment, she should be told of all
head and neck irradiation develop thyroid cancer the possible complications of radiation enteritis.
d. The treatment of choice is a near-total (or total) Which of the following is generally not associated
thyroidectomy. with radiation injury?
a. Malabsorption b. Intussusception
104. A 41- year old man complains of regurgitation c. Ulceration d. Fistulization
of salvia and undigested food. An barium study
reveals a bird’s- beak deformity. Which of the 110. A 56-yrs-old woman has been treated for 3
following statements is true about this condition? years for wheezing on exertion, which was
a. Chest pain is common in the advanced stages of diagnosed as asthma. Chest radiograph, reveals a
this disease midline mass compressing the trachea. The most
b. More patients are improved by forceful dilation likely diagnosis is:
than by surgical intervention a. Lymphoma b. Neurogenic tumor
c. Manometry can be expected to show high resting c. Lung carcinoma d. Goiter
pressures of the lower esophageal sphincter
d. Patients with this disease are at no increased risk 111. Which of the following is not seen in
for the development of carcinoma otosclerosis:
a. Tinnitus b. Vertigo
105. A primigravida is referred for non-progress of c. Conductive deafness d. Sensorineural deafness
labour. She had been given inj. Pethidine 75 mg
two hours earlier. On examination it is found that 112. The most common cause of dilated
the patient’s height is 142 cm, she is dehydrated, pulmonary artery in children is:
her BP is 100/70 mmHg, uterus is term, the lower a. Stenosis of pulmonary valve
uterine segment is stretched, foetal heart sound is b. Patent ductus arteriosus
absent, the cervix is fully dilated, the vertex is 2 cm c. Ventricular septal defect
above the ischial spines and the sagittals suture is d. Atrial septal defect
in the transverse diameter of the pelvis.
The intrapartum death of the foetus can be 113. An 18-yrs-old male is rendered paraplegic
explained as due to: after blunt vehicular trauma. After his immediate
a. Placental separation injuries are resolved, he is admitted to a
b. The rupture of membranes rehabilitation unit. Inability to void is addressed.
c. The pethidine administered Initial management of a patient who has a flaccid
d. Asphyxia arising out of prolonged labour neurogenic bladder may include which of the
following measures?
106. The most appropriate mode of intervention in a. Surgical bladder augmentation
this case would be: (In patient of above question) b. Self-catheterization
a. Forceps application b. Craniotomy c. Supravesical urinary diversion
c. Vacuum extraction d. Caesarean section d. Limiting fluid intake to less than 300 mL/day

DBMCI, Ph: 9810150067, 9868387223, 27568832, E-mail: drbhatiainstitute@rediffmail.com, Visit us at: www.dbmci.com 8
Grand Test - 36

114. A 60-yrs-old woman presents to her physician following biopsies is most characteristic of this
with a 3-week history of severe headaches. A disorders?
contrast CT scan reveals a small, circular, a. A conjunctival biopsy that reveals noncaseating
hypodense lesion with ringlike contract granulomas
enhancement. The most likely diagnosis is: b. A peripheral nerve biopsy that reveals rare acid-
a. Brain abscess fast bacteria
b. High-grade astrocytoma c. A skin biopsy that reveals dermal fibrosis with an
c. Parenchymal hemorrhage absence of adnexal structures
d. Metastatic lesion d. A subcutaneous fat biopsy that reveals an
infiltrate of plasma cells and eosinophils
115. Persistent diarrhoea in children is defined as:
a. The passage of frequent loose stool 120. In pathological myopia, power is:
b. Watery stools passed for more than three days a. 1-5D b. 5-10D
c. Episodes of loose stools lasting 14 days or more c. 10 – 15D d. 15-25D
d. Large bulky stools daily for one week
121. A 33- year old man presents with a markedly
116. A 9-yrs-old boy is being evaluated for edematous right foot that has multiple draining
deafness. Physical examination reveals a child with sinuses. A Gram stain from one of these draining
short stature, coarse facial features (low, flat nose; sinuses reveals gram-positive filamentous bacteria
thick lips; widely spaced teeth; facial fullness), a that are partially acid-fast. What is this organism?
large tongue, and clear corneas. Laboratory a. Actinomyces israelii
examination reveals increased urinary levels of b. Corynebacterium diphtheriae
heparan sulfate and dermatan sulfate. c. Listeria monocytogenes
Metachromatic granules (Reilly bodies) are found d. Nocardia asteroids
in leukocytes from a bone marrow biopsy. These
leukocytes are also found to be deficient in 122. Urinary incontinence in children is a feature
iduronosulfate sulfatase. Which of the following is of all of the following conditions except:
the most likely diagnose? a. Ectopic ureters
a. Hunter’s disease b. Hurler’s disease b. Chronic urinary tract infection
c. I cell disease c. Myelodysplasia
d. Metachromatic leukodystrophy d. Meningomyelocele

117. A 52-yrs-old man presents with symptoms of 123. A premature male infant born at 29 weeks
gastric pain after eating. During work-up, a 3-cm gestation to a 22-yrs-old woman with gestational
mass is found in the wall of the stomach. This cells diabetes develops problems breathing about 14 h
having elongated, spindle-shaped nuclei. The after delivery. Physical examination finds cyanosis,
tumor does not connect to the overlying epithelium tachypnea, and nasal flaring. A chest x-ray reveals
and is found only in the wall of the stomach. a bilateral diffuse reticular (“ground glass”)
Which of the following is the cell of origin of this appearance. His respiratory problems worsen and
tumor? he is put on a ventilator. Which one of the
a. Adipocyte b. Endothelial cell following therapies should also be given at this
c. Glandular epithelial cell d. Smooth-muscle cell time?
a. Dietary vitamin C b. Intramuscular insulin
118. Which one of the following is the most c. Intranasal vasopressin d. Intratracheal surfactant
common cause of lower urinary obstruction in a
male infant? 124. Which of the following is the cause of the neck
a. Bladder neck obstruction swelling of a patient’s who is monosomic for X
b. Acute haemorrhagic cystitis chromosome?
c. Urethral stricture a. Bacillary angiomatosis b. Cystic hygroma
d. Posterior urethral valves c. Glomus Tumor d. Nevus flammeus

119. A 36-yrs-old woman presents with increased


trouble swallowing. Physical examination finds
We are also starting Grand Test Series on every last Sunday of
hypertension and sclerodactyly, while laboratory
the month for Dental students, those who are preparing for
examination finds an autoantibody against DNA MDS Entrance Exam. We shall be starting from last Sunday of
topoisomerase (Anti-Scl- 70). Which of the June 2006. All India Ranking will be done.

DBMCI, Ph: 9810150067, 9868387223, 27568832, E-mail: drbhatiainstitute@rediffmail.com, Visit us at: www.dbmci.com 9
Grand Test - 36

125. A five years old child is brought with fever hematocrit, increased RBC count, increased total
and pallor. On examination he is found to have red cell mass, normal plasma volume, normal
anaemia, petechial haemorrhages, oxygen saturation of hemoglobin, and increased
lymphadenopathy, tenderness of the bones and serum erythropoietin. Which of the following
hepatosplenomegaly. The most likely diagnosis is: abnormalities is most likely to be present in this
a. Acute rheumatic fever individual ?
b. Infectious mononucleosis a. Acute gastroenteritis
c. Idiopathic thrombocytopenic purpura b. Chronic ulcerative colitis
d. Acute leukaemia c. Cyanotic heart disease
d. Renal cell carcinoma
126. Prior to surgery, which of the following is the
best medical therapy for a new born infant with 131. If a child develops cataract in infancy, then
transposition of the pulmonary artery and aorta? which one of the following possibilities should be
a. Give prostaglandin E2 to keep the ductus arteriosus ruled out?
open a. Wilson’s disease
b. Give prostaglandin F2 to close the ductus arteriosus b. Maple syrup urine disease
c. Give oxygen to keep the ductus arteriosus open. c. Galactosemia
d. Give indomethacin to keep the ductus arteriosus d. Juvenile diabetes
open.
132. A 5-yrs-old girl presents with a several day
127. A 49 – year old woman presents with signs of history of localized swelling in the right side of her
anemia and states that every morning her urine is neck. There is no recent history of sore throat.
dark. Workup reveals that her RBC lyse in vitro Physical examination finds a low-grade fever, and
with acid. Which of the following is the most likely one very tender, firm, slightly enlarged lymph
diagnosis ? node is palpated in the right cervical region. A
a. Warm autoimmune hemolytic anemia CBC reveals a mild leukocytosis. A fine needle
b. Paroxysmal nocturnal haemoglobinuria aspiration of the lymph node reveals scattered
c. Paroxysmal cold hemaglobinuria neutrophils. Which of the following is the most
d. Isoimmune hemolytic anemia likely diagnosis?
a. Bacterial lymphadenitis
128. In Indian childhood cirrhosis, the most b. Granulomatous lymphadenitis
common biochemical change seen is that: c. Necrotizing lymphadenitis
a. Serum copper level is increased while serum zinc d. Toxoplasmic lymphadenitis
level is decreased.
b. Both serum copper level and serum zinc level are 133. Histologic sections of lung reveal the alveoli to be
decreased filled with pale, nongranular pink fluid. Neither
c. Serum copper level is decreased while serum zinc leukocytes nor erythrocytes are present within this
level is increased fluid. Which of the following is the most common
d. Both serum copper level and serum zinc level are cause of this abnormality?
increased. a. Bacterial pneumonia
b. Congestive heart failure
129. A 5-years-old child develops the sudden onset c. Lymphatic obstruction by tumor
of bloody diarrhea, vomiting of blood, hematuria, d. Pulmonary embolus
and renal failure following a flulike
gastrointestinal illness. The BUN level is markedly 134. Which of the following statements are true of
increased, but FDP and clotting time are within toxic epidermolysis bullosa?
normal limits. A P/S reveals poikilocytes, 1. May be caused by bacterial or viral infection
schistocytes, and a decrease platelets count. No 2. Skin lesions are present with systemic reaction
fever or neurologic symptoms are present. Most 3. Nikolsky’s sign positive
likely diagnosis? 4. Transmitted as autosomal dominant
a. Autoimmune thrombocytopenic purpura Select the correct answer using the codes given
b. Disseminated intravascular coagulopathy below:
c. Hemolytic-uremic syndrome a. 1, 2 and 3 b. 1 and 3
d. Isoimmune thrombocytopenic purpura c. 2 and 4 d. 1, 2, 3 and 4

130. Medical evaluation of a 55- year old man finds 135. A 24- year old woman presents with
the following laboratory data: increased nonspecific symptoms including fever and

DBMCI, Ph: 9810150067, 9868387223, 27568832, E-mail: drbhatiainstitute@rediffmail.com, Visit us at: www.dbmci.com 10
Grand Test - 36

malaise. A chest X-ray reveals enlarged hilar a. Centrilobular necrosis


lymph nodes, while her serum calcium level is b. Focal scattered necrosis
found to be elevated. Which of the following c. Midzonal necrosis
histologic abnormalities is most likely to be seen in d. Periportal necrosis
biopsy specimens from these enlarged hilar lymph
nodes? 141. Vitamin D refractory rickets is transmitted
a. Caseating granulomas by:
b. Dense, granular , PAS- positive , eosinophilic a. X-linked recessive inheritance
material b. X-linked dominant inheritance
c. Markedly enlarged epithelial cells with c. Autosomal recessive inheritance
intracellular inclusions d. Polygenic inheritance
d. Noncaseating granulomas
142. A 53- year old man presents with severe
136. An 18-yrs-old woman presents with a small, headaches, nausea , and vomiting. He also relates
painless, swelling in the lateral left side of her seeing spots before his eyes and is found to have a
neck. Physical examination finds a 7- mm, smooth, diastolic blood pressure of 160 mmHg. Gross
nontender, fluctuant mass located along the lower examination of his kidney is most likely to reveal
one-third of the sternocleidomastoid muscle. The which one of the following changes?
mass does not move when she swallows. The mass a. A finely granular appearance to the surface
is excised and histologic sections reveal a cystic b. Multiple small petechial hemorrhages on the
lesion lined by stratified squamous epithelium. surface
Lymphoid tissue is present in the wall of the cyst. c. Diffuse, irregular cortical scars overlying dilated
Which of the following is the most likely diagnosis? calyces
a. Branchial cleft cyst d. Cortical scars overlying dilated calyces in renal
b. Dentigerous cyst poles
c. Odontogenic keratocyst
d. Radicular cyst 143. A 69-yrs-old man presents with urinary
frequency, nocturia, dribbling, and difficulty in
137. All of the following are characteristics of starting and stopping urination. Rectal
cephalohematoma except: examination reveals the prostate to be enlarged,
a. Does not increase in size while crying firm, and rubbery. A needle biopsy reveals
b. The swelling is pulsatile increased numbers of glandular elements and
c. It is benign, self-limiting and resolves in a few stromal tissue. The glands are found to have a
week’s time double layer of epithelial cells. Prominent nuclei or
d. Does not cross suture line. back-to –back glands are not seen. Which of the
following is the most likely diagnosis?
138. A 45- years old male alcoholic presents with a. Atrophic prostatitis
vomiting of blood and hypotension. He denies any b. Atypical small acinar proliferation
history of vomiting nonblood material or retching c. High-grade prostatic intraepithelial neoplasia
prior to vomiting blood. During work-up he dies d. Benign prostatic hyperplasia
suddenly. Which of following histologic changes is
most likely to be seen in a biopsy specimen taken 144. A 3-yrs-old male child is admitted in the
from his esophagus? casualty ward with dyspnoea, brassy type of cough
a. Metaplastic columnar epithelium and swelling in the neck which is tender on
b. Decreased ganglion cells in the myenteric plexus palpation. The child also complains of dysphagia.
c. Dilated blood vessels in the submucosa The most likely diagnosis is:
d. Mucosal outpouchings a. Oesophagitis b. Achalasia
c. Hiatus hernia d. Retropharyngeal abscess
139. Diplopia produced in lateral rectus palsy is:
a. Crossed b. Uncrossed 145. A 25-yrs-old woman in her fifteenth week of
c. Both d. None pregnancy presents with uterine bleeding and
passage of a small amount of watery fluid and
140. An autopsy is performed on a 19-year old tissue. She is found to have a uterus that is much
woman who died from an overdose of larger than estimated by her gestational dates. Her
acetaminophen. Which of the following histologic uterus is found to be filled with cystic, avascular,
changes is most likely to be seen in a biopsy grapelike structures that do not penetrate that
specimen taken from her liver?

DBMCI, Ph: 9810150067, 9868387223, 27568832, E-mail: drbhatiainstitute@rediffmail.com, Visit us at: www.dbmci.com 11
Grand Test - 36

uterine wall. No fetal parts are found. Which of the a. Actinic keratosis b. Bowen’s disease
following is the most likely diagnosis? c. Keratoacanthoma d. Seborrheic keratosis
a. Partial hydatidiform mole
b. Complete hydatidiform mole 150. Which of the following gases cause green-
c. Invasive mole house effect and depletion of earth’s ozone layer?
d. Placental site trophoblastic tumor 1. Carbon dioxide
2. Sulphur dioxide
146. During a routine breast self-examination, a 3. Chlorofluorocarbons
35-yrs-old woman is concerned because her breast Select the correct answer using the following:
feel “lumpy”. She consults you as her primary care a. 1, 2 and 3 b. 1 and 2
physician. After performing an examination, you c. 2 and 3 d. 1 and 3
reassure her that no masses are present and that
the “ lumpiness” is due to fibrocystic changes. 151. A 36-yrs-old woman presents because of
Which of the following pathologic findings is a type increasing pain in her hands and knees, which, she
of nonproliferative fibrocystic change? says, is worse in the morning. Physical
a. A blue-domed cyst examination finds her fingers to be swollen and
b. A radial scar stiff, and there is ulnar deviation or her
c. Atypical ductal hyperplasia metacarpophalangeal joints. A biopsy from her
d. Papillomatosis knee would likely show areas where histiocytes
were palisading around irregular areas of necrosis.
147. Only one of the following reports of The biopsy would also likely show proliferation
examination of samples of water from different and hyperplasia of the synovium with destruction
treatment plants meets the WHO standards for of the articular cartilage. Which one of the
drinking water. Which one is it? following terms best describes these pathologic
a. Only 5% of samples in the year showed coliforms, changes?
but the number was less than 3/100 ml. two of these a. Eburnation b. Gumma
positive samples were consecutive ones. None of the c. Pannus d. Spondylosis
sample showed E.coli
b. Only 10% of samples showed coliforms of less than 152. Twilight vision is due to:
3/100 ml. none of the positive were consecutive ones. a. Rods b. Cones
E. coli were absent from all the samples c. Both d. None
c. None of the samples showed coliforms E. coli were
present in 2% of the samples 153. A 34 – yrs-old woman runner presents with
d. Only 5% of samples showed coliforms of less than pain in the plantar portion of her foot between the
3/100 ml. None of the positive were consecutive ones. third and fourth metatarsal bones. Which of the
E. coli were absent from all the sample. following is the most likely cause of her foot pain?
a. Ganglion b. Ganglioneuroma
148. An XX infant is found to have external male c. Traumatic neuroma d. Morton’s neuroma
genitalia and internal female genitalia. Physical
examination reveals decreased BP, while 154. Which one of the following, if presents in the
laboratory examination reveals a serum sodium atmosphere, is not an ‘indicator’ of air pollution?
level of 132 meq/L. additionally, bilateral adrenal a. Carbon monoxide b. Smoke
cortical hyperplasia is present. A deficiency of c. Sulphur dioxide d. Suspended particles
which of the following enzymes is most likely to
produce the clinical findings in this infants? 155. A 42-yrs-old immunosuppressed man presents
a. 3--dehydrogenase b. 11-hydroxylase with rapidly progressive neurologic symptoms
c. 17-hydroxylase d. 21-hydroxylase including mental deterioration, visual loss,
abnormal speech, and ataxia. Radiographic studies
149. A 65-yrs-old man who is a long-time farmer demonstrate multifocal lesions in the white matter
presents with a small, scaly erythematous lesion on without mass effect (no shift in the cerebral
the helix of his left ear. A biopsy from this lesion hemispheres is seen). A stereotactic brain biopsy
reveals marked degeneration of the dermal reveals areas of demyelination. Which of the
collagen along with atypia of the squamous following histologic features is most likely to be
epidermal cells. The atypia, however, does not present in this biopsy specimen?
involve the full thickness of the epidermis, and no a. Beta – A4 amyloid within blood vessels
invasion into the underlying tissue is seen. Which b. Prion protein within extracellular plaques
of the following is the most likely diagnosis? c. Rosenthal fibers within astrocytes

DBMCI, Ph: 9810150067, 9868387223, 27568832, E-mail: drbhatiainstitute@rediffmail.com, Visit us at: www.dbmci.com 12
Grand Test - 36

d. Viral particles within oligodendrocytes are positive. Without treatment, the next stage of
this disease is clinically characterized by:
156. After an initial pregnancy resulting in a a. Optic nerve atrophy and generalized paresis
spontaneous loss in the first trimester, your patient b. Tabes dorsalis
is concerned about the possibility of this recurring. c. Gummas
An appropriate answer would be that the chance d. Macular rash over the hands and feet
of recurrence
a. Depends on the genetic makeup of the prior 162. Which of the following is a true statement
abortus regarding the psychological symptoms of the
b. Is no different than it was prior to miscarriage climacteric?
c. Is increased to approximately 20% a. They are considerably less important than hormone
d. Is increased most likely to greater than 50% levels
b. They commonly include insomnia, irritability,
157. Incidence of malaria fell in India till 1966 and frustration, and malaise
thereafter it began to show an increase. All of the c. They are related to a drop in gonadotropin levels
following are the correct reasons for this except: d. They are not affected by environmental factors
a. Inadequate supply of insecticides
b. Migration of labour from one part of the country to 163. The provision that “an insured woman in
another pregnancy is entitled to full wages for a period of
c. Anopheline species of mosquito developed 12 weeks of which six weeks shall precede the date
resistance to insecticides of confinement” is stipulated under the:
d. Non-anopheline species of mosquitoes began to a. Indian factories Act
transmit malaria b. ESI Act
c. Minimum Wages Act
158. Velamentous insertion of the cord is d. Model Public Health Act
associated with an increased risk for:
a. Premature rupture of the membranes 164. A 65- year old woman complains of leakage of
b. Fetal exsanguinations before labor urine. The most common cause of this condition in
c. Torsion of the umbilical cord such patient is
d. Fetal malformations a. Anatomic stress urinary incontinence
b. Urethral diverticula
159. A 70-yrs-old woman presents for evaluation of c. Overflow incontinence
a pruritic lesion on the vulva. Examination shows a d. Unstable bladder
white, friable lesion on the right labia majora that
is 3 cm in diameter. No other suspicious areas are 165. A 50-yrs-old develops sudden onset of bizarre
noted. Biopsy of the lesion confirms squamous cell behavior. CSF shows 80 lymphocytes; magnetic
carcinoma. In this patient, lymphatic drainage resonance imaging shows temporal lobe
characteristically would be first to the: abnormalities. Most likely organism is:
a. External iliac lymph nodes a. Herpes simplex type 1
b. Superficial inguinal lymph nodes b. Herpes simplex type 2
c. Deep femoral lymph nodes c. Coxsackievirus B
d. Periaortic nodes d. Cryptococcus neoformans

160. Progressive massive fibrosis (PMF) is a 166. Prolonged exposure to inorganic lead causes
condition of: all of the following symptoms except:
a. Lungs affected by occupational exposure to silica a. Sever diarrhoea
dust b. Stippling of red cells
b. Brain in Alzhemer’s disease c. Blue lining on the gums
c. Spinal cord in multiple sclerosis d. Foot drop
d. Liver as a sequel to hepatitis due to non-A, non-B
hepatitis virus 167. In which of the following Involvement of the
upper and lower respiratory tracts;
161. At the time of annual examination, a patient glomerulonephritis occurs most likely?
expresses concern over exposure to sexually a. Wegener’s granulomatosis
transmitted diseases. During your pelvic b. Multiple cholesterol embolization syndrome
examination, a singular, indurated, nontender c. Polyarteritis nodosa
ulcer is noted on the vulva. VDRL and FTA tests d. Giant cell arteritis

DBMCI, Ph: 9810150067, 9868387223, 27568832, E-mail: drbhatiainstitute@rediffmail.com, Visit us at: www.dbmci.com 13
Grand Test - 36

168. A 60-yrs-old female complains of dry mouth 175. A 90-yrs-old male complains of hip and back
and a gritty sensation in her eyes. She states it is pain. He has also developed headaches, hearing
sometimes difficult to speak for more than a few loss, and tinnitus. On physical exam the skull
minutes. There is no history of diabetes mellitus or appears enlarged, with prominent superficial
neurologic disease. The patient is on no veins. There is marked kyphosis, and the bones of
medications. On exam, the buccal mucosa appears the leg appear deformed. Plasma alkaline
dry and the salivary glands are enlarged phosphatase is elevated. A skull x-ray shows
bilaterally. The next step in evaluation is: sharply demarcated lucencies in the frontal,
a. Lip biopsy parietal, and occipital bones. X-rays of the hip
b. Schirmer test and measurement of autoantibodies show thickening of the pelvic brim. The most likely
c. IgG antibody to mumps virus diagnosis is?
d. Use of corticosteroids a. Multiple myeloma
b. Paget’s disease
169. The commonly used socio-economic status c. Hypercalcemia
scale does not take into consideration: d. Metastatic bone disease
a. Education b. Occupation
c. Income d. Housing 176. Which one of the following has been shown to
have direct association with social disintegration
170. A patient with low-grade fever and weight loss and thus a greater propensity to produce mental
has poor expansion on the right side of the chest illness?
with decreased fremitus, flatness to percussion, a. Low self-esteem
and decreased breath sounds all on right. The b. Low socio-economic status
trachea is deviated to the left. The most likely c. Low education status
diagnosis is d. Low morale
a. Pneumothorax b. Pleural effusion
c. Pneumonia d. Atelectasis 177. A 55-yrs-old type 2 diabetic patient has lost
weight and has had good control of his blood sugar
171. A 35-yrs-old male complains of substernal on oral agents. He has a history of mild
chest pain aggravated by inspiration and relieved hypertension and hyperlipidemia. He asks for
by sitting up. He has a history of tuberculosis. advice about an exercise program. Which one of
Lung fields are clear to auscultation, and heart the following statements is correct?
sounds are somewhat distant. Chest x-ray shows a. Exercise should be avoided because it may cause
an enlarged cardiac silhouette. The next step in foot trauma
evaluation is: b. An active lifestyle cannot slow the complications of
a. Right lateral decubitus film diabetes
b. Cardiac catheterization c. Vigorous exercise cannot precipitate hypoglycemia
c. Echocardiogram d. A stress test should be recommended prior to
d. Serial ECGs beginning and exercise program

172. Health education: 178. A 40-yrs-old male with long – standing alcohol
a. Develops reflexive behavior abuse complains of abdominal swelling, which has
b. Trains people to use judgement for health action been progressive over several months. He has a
c. Appeals to emotions to adopt health history of gastrointestinal bleeding. On physical
d. Does not change attitude exam, there are spider angiomas and palmar
erythema. Abdominal collateral vessels are seen
173. For hyponatremia select the around the umbilicus. There is shifting dullness,
electrocardiographic finding with which it is most and bulging flanks are noted. An important first
commonly associated. step in the patient’s evaluation is
a. No known electrocardiographic abnormalities a. Diagnostic paracentesis
b. Prolonged QT interval b. Upper GI Endoscopy
c. Short QT interval c. Blood Ethanol level
d. Widened QRS complex d. CT scan of abdomen

174. Increased lactic acid in aqueous humour is 179. Which one of the following drug habits causes
found in: in a person both ‘psychic’ and ‘physical’
a. Aphakia b. Ectopia lentis dependence.
c. Traumatic dislocation d. Senile cataract a. Amphetamine b. Barbiturates

DBMCI, Ph: 9810150067, 9868387223, 27568832, E-mail: drbhatiainstitute@rediffmail.com, Visit us at: www.dbmci.com 14
Grand Test - 36

c. Cannabis d. Heroin d. Planning, programming and budgeting system

180. How would you test for the for the destruction 186. Normally, the left adrenal venous drainage is
of anterior cruciate ligament: into which of the following?
a. Excess ability to displace a flexed leg posterior a. Inferior vena cava b. Left azygos vein
b. Excess ability to displace a flexed leg anteriorly c. Left inferior phrenic vein d. Left renal vein
c. Excess ability to displace the ankle medially
d. Excess ability to displace the ankle laterally 187. In a surgical procedure, at groin area it is
noticed that a nerve has been inadvertently
181.A physician examines a patient who complains sectioned. This nerve exited through the superficial
of pain and paresthesia in the left leg. The inguinal ring and was applied to the anterior
distribution of the pain – running down the lateral aspect of the spermatic cord. As a result of this
aspect of the leg and the dorsal surface of the foot operative error, it is probable that which of the
– is suggestive of a herniated intervertebral disk. following will apply to this patient?
The physician links the distribution of symptoms a. He will be unable to produce spermatozoa in the
with nerve L5 and concludes that herniation has right testis
occurred at which location? b. He will become impotent
a. L3 – L4 intervertebral disk c. He will lose the cremasteric reflex on the right side
b. L4 – L5 intervertebral disk d. He will lose sensation over portions of the base of
c. L5 – S1 intervertebral disk the penis and anterior scrotum
d. S1 – S2 intervertebral disk
188. Multiple, painful vesicles on the glans penis of
182. Which of the following can be described as a sexually active young patient would most likely
kanor drawbacks in India’s Primary Health Care indicate.
System? a. Epispadias b. Syphilis
1. Lack of adequate technical manpower c. Herpes genitalis d. condylomata acuminata
2. Lack of commitment and discipline among health
workers 189. Which bypassing occluded coronary arteries,
3. Lack of adequate managerial skills on the part of sections, of internal thoracic artery have been
the health care specialist preferred in recent years over lower leg veins,
Select the correct answer using the codes given since they seem to last longer. In coronary bypass
below: surgery, the transplanted blood vessel is placed in
a. 2 and 3 b. 1 and 2 which of the following positions?
c. 1 and 3 d. 1, 2 and 3 a. The proximal end of the artery is anastomosed to
the pulmonary trunk
183. Which part of the vertebra would connect to b. The distal end of the artery is anastomosed to the
the three other parts: the body, the lamina and the great cardiac vein
transverse process? c. The proximal end is attached to the ascending aorta
a. Inferior articular facet and the distal end is attached distal to the occluded
b. Superior articular facet coronary artery
c. Both A + B d. The distal end of the artery is anastomosed to the
d. Pedicle great cardiac vein and the proximal end is attached
distal to the occluded coronary artery.
184. Principal supply to the body and tail of the
pancreas. 190. A 40-yrs-old male patient comes to you with
a. Common hepatic artery angina and tachycardia. This patient’s tachycardia
b. Inferior phrenic artery probably is mediated by reflex arcs associated with
c. Splenic artery decreased cardiac output and possibly reduced
d. Superior mesenteric artery blood pressure. The visceral efferent (motor)
pathway of this cardiac response is mediated by
185. A network is a graphic plan of all events and which of the following?
activites to be completed in order to reach an a. Carotid branches of the glossopharyngeal nerves
objective. Which one of the following is a common b. Greater splanchnic nerves
type of network analysis? c. Phrenic nerves
a. Input-output analysis d. Sympathetic cervical and thoracici cardiac fibers
b. Systems analysis
c. Critical path method

DBMCI, Ph: 9810150067, 9868387223, 27568832, E-mail: drbhatiainstitute@rediffmail.com, Visit us at: www.dbmci.com 15
Grand Test - 36

191. The action taken when a person comes to 196. The retinal pigment epithelium (RPE) is
India without a valid vaccination certificate for characterized by which of the following?
yellow fever, is: a. The presence of the photoreceptor (rod and cone)
a. Isolation b. Quarantine perikarya
c. Immunoprophylaxis d. Chemoprophylaxis b. Phagocytosis of worn-out components of
photoreceptor cells
192. Muscle relaxants are used routinely during c. Origin from the inner layer of the optic cup during
anesthesia with resultant closure of the vocal folds. embryonic development
Laryngeal intubation by the anesthesiologist is d. Presence of amacrine cells
necessary because which of the following muscles
is unable to maintain the glottis open? 197. When the disease agent multiplies and
a. Cricothyroid muscle undergoes a cycle of development in the insect
b. Lateral cricoarytenoid muscles vector, the type of biological transmission is
c. Posterior cricoarytenoid muscles referred to as:
d. Thyroarytenoid muscle a. Propagative b. Cyclo-developmental
c. Cyclo-propagative d. Transgenic cycle
193. A 63-yrs-old woman was brought into the
emergency room by her son, who suspected she 198. A correlation coefficient of  = 0 indicated
had suffered a stroke the previous night. that:
Subsequent examination revealed spastic a. There is no linear relationship between the two
hemiplegia on the left side with hyperreflexia and a variables of the sample studied
positive Babinskis sign. The left side of the patients b. The sample was too small to arrive at any tangible
face was paralyzed below the eye, and the right eye conclusion
was turned out and down. The right pupil made c. The relationship is highly significant
direct and consensual responses to light, but the d. There is absolutely no relationship between the two
left pupil was fixed and unresponsive. There were variables
no apparent sensory deficits. Which is the most
likely location of the lesions? 199. Synthesis of milk by the mammary gland
a. Left motor cortex b. Right sensory cortex specifically requires which of the following?
c. Right midbrain d. Left thalamus a. Oxytocin
b. Production of prolactin by the corpus luteum
194. Consider the following data: c. The influence of vasopressin
The mid-year population of a town was : 10,000 d. Placental lactogen
The number of deaths in the town during a
year : 150 200. Subsequent to a fall, a young man is unable to
The number of live births during the same lift injured upper limb. On inspection, the normal
year : 100 rounded contour of the shoulder is found to have
The number of death under one year of age : 10 been lost; the upper arm appears longer on the
The number of death of infants of 28 days to injured side and the acromion is prominent. The
one year of age :8 most likely diagnosis is:
The number of maternal deaths during the a. Fracture neck of the humerus
year :1 b. Anterior dislocation of the shoulder
Based on the above data, the calculated post neonatal c. Posterior dislocation of the shoulder
mortality rate will be: d. Acromio-clavicular dislocation
a. 100 b. 20
c. 50 d. 80 201. A pheochromocytoma is a common tumor of
the adrenal medulla. In the presence of this tumor,
195. The direction in which vestibular hair cell which of the following symptoms would most likely
stereocilia are deflected is important for which of be observed?
the following reasons? a. Hypotension b. Hypoglycemia
a. Differentiates between 1 and type 2 hair cells c. Hirsutism d. Paroxysms
b. Determines whether cells are depolarized or
hyperpolarized 202. Hirschsprung’s disease and Chagas disease
c. Determines whether linear or angular acceleration result in disturbance of intestinal motility. The site
is detected of this disruption is most likely which of the layers
d. Determines the direction of blood flow in the on the accompanying micrograph?
stria vascularis a. Lamina Propria b. Smooth muscle cell

DBMCI, Ph: 9810150067, 9868387223, 27568832, E-mail: drbhatiainstitute@rediffmail.com, Visit us at: www.dbmci.com 16
Grand Test - 36

c. Meissneur plexus d. Muscularis externa c. Pseudopolyposis coli


d. Gardner’s syndrome
203. Presence of calcification in an intracranial
lesion is best made out by: 211. Once the osteosarcoma cells each the lungs,
a. CT b. MRI enter the lung parenchyma, and conally expand,
c. Ultrasound d. Contrast study they produce bone. In that process, the tumor cells
synthesize which of the following?
204. A first –year female medical student presents a. Type III collagen b. Type II collagen
with patches of raised red skin covered by a flaky c. Acid phosphatase d. Alkaline phosphatase
white buildup on her knees and elbows. The
patches enlarge and become itchy and burning 212. After birth, growth in the length of long bones
immediately before and during major exams occurs primarily through which of the following?
during the firs year of medical school. A biopsy a. Increased bone deposition under the periosteum
from her skin is shown to the right. Which of the b. Proliferation of osteoblasts in the primary
following is the underlying cause of this disorder? ossification center
a. Hyperplasia of dermal cells c. Proliferation of osteoblasts in the secondary
b. A longer keratinocyte cell cycle ossification center
c. Production of cytokines by infiltrating d. Interstitial growth of cartilage cells in the epiphyses
inflammatory cells
d. Microabscesses of the dermis 213. Bullous lesions occur in:
a. Diabetes mellitus b. Porphyria
205. Immunoglobulin switching from IgM to IgG c. Haemachromatosis d. Uraemia
occurs primarily in the:
a. Bone marrow b. Peripheral blood 214. Which of the following is the function of
c. Germinal centers d. Thymus fibronectin in the extracellular matrix?
a. Structural support
206. Resistant kala-azar is defined by the b. Binding of signaling molecules
persistence of all of the following features despite c. Selectivity for passage of molecules
therapy, except: d. Adhesion and cell attachment
a. Fever
b. Non-regression of splenomegaly 215. The mechanism for tube formation as occurs
c. LD bodies in more than 5% cells in bone marrow during development of the neural tube could best
d. Hyperglobulinemia be explained by which of the following?
a. Contraction of microfilament bundles associated
207. In the human fetus, the predominant site of with the zonula adherens
Hemopoiesis in gestational months 5 to 9 occurs in b. Increased condensation of the transmembrane
which of the following? linkers of the desmosomes
a. Liver b. Yolk sac c. Expansion of the sealing strands in the zonulae
c. Spleen d. Bone marrow occludentes
d. Condensation of the gap junctions
208. Christmas tree appearance is seen in which of
the following: 216. A 30-year old male presents with non-itchy
a. Downs syndrome b. Myotonic dystrophy generalised papulo-nodular lesion of three months
c. Rubella d. Toxoplasmosis duration. Physical examination does not show any
other abnormality. Slit smear from a nodule does
209. The sarcoplasmic reticulum of skeletal muscle not show AFB. Blood VDRL is reactive in 1:2
functions in which of the following? dilution. The most likely diagnosis is:
a. Cellular calcium storage A. Drug eruption
b. Cellular glycogen storage B. Lepromatous leprosy
c. Glycogen degradation C. Post-kala-azar dermal leishmaniasis
d. Transport of Ca2+ into the terminal cisternae during D. Secondary syphilis
muscle contraction
217. Chaperonins function in which of the
210. All of the following are believed to be pre- following ways?
cancerous lesions of the colon except: a. Ensure correct folding of the signal peptide with its
a. Familial polyposis coli receptor on the rough endoplasmic reticulum
b. Peutz – Jeghers syndrome

DBMCI, Ph: 9810150067, 9868387223, 27568832, E-mail: drbhatiainstitute@rediffmail.com, Visit us at: www.dbmci.com 17
Grand Test - 36

b. Control the docking of the signal peptide with its a. Prepubertal female
receptor on the rough endoplasmic reticulum b. Postpubertal, premenopausal, nulliparous female
c. Serve as a start-transfer signal in the membrane of c. Premenopausal, multiparous female
the endoplasmic reticulum d. Menopausal, nulliparous female
d. Stabilize microtubules
225. Serosanguinous discharge from a laparotomy
218. The formation of the acrosome involves which wound on the ninth post-operative day is
of the following? suggestive of:
a. Loss of decapacitation factors A. Impending burst abdomen
b. Release of the developing spermatids from Sertoli B. Wound infection
cells C. Synergistic gangrene of the abdominal wall
c. Maturation of lytic enzymes D. Intestinal gangrene
d. Mitotic activity
226. The prader-willi syndrome (PWS) is a rare
219. A 30-year old male presents with lower limb syndrome of progressive obesity, hypotonia, and
polyneuropathy. His lateral popliteal nerve is mental retardation. On physical examination,
thickened. Which one of the following diagnosis is children with PWS also have small hands and feet
unlikely? and male children also have cryptorchidism.
A. Primary amyloidosis Which of the following statements concerning the
B. Familial hypertrophic neuropathy mode of inheritance of PWS is true?
C. Tuberculoid leprosy a. PWS is inherited as an X-linked recessive disorder
D. Diabetes mellitus b. PWS is inherited as an X-linked dominant disorder
c. PWS is a result of a gene duplication
220. Which of the following processes is involved in d. PWS is caused by maternal uniparental disomy
lipid absorption and processing by the enterocyte?
a. Active transport of glycerol across the apical 227. A 35-yrs-old previously healthy woman was
membranes given a 10-day course of erythromycin for a
b. Exocytosis of chylomicra at the basolateral nonproductive cough, diffuse interstitial infiltrates,
membranes and a presumptive diagnosis of mycoplasma
c. Endocytosis of triglycerides at the apical membrane pneumonia. She now has a well demarcated rash
d. Uptake of chylomicra by phagocytosis across the on her fingers. She also has fever and severe
apical membrane erosions on the buccal mucous membranes. The
most likely diagnosis is:
221. In saliva, IgA is complexed with: a. Hypersensitivity vasculitis
a. Amylase b. Mucins b. Polyarteritis nodosum
c. Lactoferrin d. Secretory component c. Toxic epidermal necrolysis
d. Erythema multiforme major
222. A 65-year old farmer has chronic itchy
dermatitis with erythematous indurated plaques 228. Duke’s stage C refers to carcinoma:
on the face, upper trunk, extensor aspects of the A. Bladder penetrating the extravesical fat
arms, hands and exposed parts of the body. The B. Bladder with metastasis to internal iliac lymph
condition has progressively worsened over the past nodes
five years. The most likely diagnosis is:, C. With histological feature of 75% anaplastic cells
A. Scabies B. Psoriasis D. Rectum with metastasis to inferior mesenteric
C. Lichen planus D. Actinic reticuloid lymph nodes

223. Which of the following cells or parts of the 229. A 30-yrs-old woman with long-standing sickle
pituitary are derived embryologically from cell anemia presents with severe pain in the chest
neuroectoderm? and abdomen ~ 1 week after having an upper
a. Gonadotrophs b. Pars intermedia respiratory infection. No intrathoracic or
c. Pars tuberalis d. Herring bodies intraabdominal pathology was immediately
obvious on routine physical examination and
224. Naturally occurring, nonpathologic cervical laboratory evaluation. The most appropriate
eversions (“erosions”) are usually naturally therapeutic intervention at this point is:
corrected by re-epithelialization. These eversions a. Hypertransfusion
are most prevalent in which one of the following b. Hydration and narcotic analgesia
reproductive classifications of women? c. Hydroxyurea

DBMCI, Ph: 9810150067, 9868387223, 27568832, E-mail: drbhatiainstitute@rediffmail.com, Visit us at: www.dbmci.com 18
Grand Test - 36

d. Broad-spectrum antibiotics b. Their effect can be blocked by high doses of beta2


selective inhibitor
230. A 73-yrs-old woman with known c. Transdermal patches are more likely to be
myelodysplastic syndrome and chronic anemia has associated with headaches than are sublingual
required multiple transfusion over the past several nitrates
months. She now presents with profound fatigue d. Oral preparations are more effective than
and a hematocrit of 20%. Two units of blood are sublingual ones
ordered, but the blood component lab informs you
that they expect at least a day before a product will 237. Absolute contraindications to the use of
be ready for the patient. The most likely thrombolytic agents in the setting of an acute
explanation for this problem in findings anterior myocardial infarction include which of
appropriate blood for transfusional therapy in this the following?
patient is: a. Left carotid artery occlusion with hemiparesis 1
a. Presence of allo-antibodies in the patient’s serum month ago
b. Presence of autoantibodies in the patient’s serum b. Diabetic retinopathy
c. Rare blood group c. Cardiopulmonary resuscitation (CPR) for ~1 to 2
d. Careful screening necessary in myelodysplasia min
patients to prevent blood borne infection d. Patient age > 70

231. Acute mechanical large bowel obstruction 238. About six to eight hours after peptic
should be operated early because of the risk of: perforation, the disappearance of abdominal wall
A. Electrolyte imbalance due to third space loss rigidity is due to:
B. Septicaemia from absorption of bowel contents A. Cessation of acid secretion in the stomach
C. Early gangrene and perforation B. Revival from initial shock
D. Respiratory embarrassment due to massive C. Dilution of acid in the peritoneal cavity
abdominal distension D. Fatigue of the reflex arc

232. The most common source of bacterial 239. The primary pathophysiologic problem in
infection of intravenous canulas is: idiopathic pulmonary fibrosis is believed to be
a. Contamination of fluids during the manufacturing a. Microorganism – mediated activation of pulmonary
process neutrophils
b. Contamination of fluids during insertion of the b. Immune complex-mediated activation of alveolar
cannula macrophages
c. Contamination at the site of entry through the skin c. Direct immune complex – mediated pulmonary
d. Contamination during the injection of medications interstitial damage
d. Primary fibroblast proliferation
233. Nodule in iris not found in:
a. Sarcoidosis b. Neurofibromatosis 240. A 19-yrs-old normal woman has a moderately
c. Tuberous sclerosis d. SLE severe pulmonary embolism while on oral
contraceptive pills. Which of the following is the
234. A 62-yrs-old gardener who has chronic most likely predisposing factor?
lymphocytic leukemia develops lymphangitis and a a. Abnormal factor V
painless, nodular lesion on his wrist. Subsequently, b. Abnormal protein C
he becomes severely ill with cavitary right-upper- c. Diminished protein C level
lobe pneumonia; Sporothrix schenckii is isolated. d. Diminished protein S level
He should be treated with:
a. Chloramphenicol b. Potassium iodide 241. CVP (central venous pressure) and
c. Penicillin d. Amphotericin B pulmonary wedge pressure give an accurate
assessment of all of the following except:
235. In an infant born with intestinal obstruction A. Tissue perfusion B. Volume depletion
due to meconium ileus, one should suspect: C. Volume overload D. Myocardial function
A. Hirschsprung’s disease B. Mucoviscidosis
C. Imperforate anus D. Bile duct atresia 242. A 50-yrs-old man is hospitalized for treatment
of enterococcal endocarditis. He has been receiving
236. Which of the following statements best ampicillin and gentamicin for the past 2 weeks but
describes long-acting nitrate preparations? is persistently febrile. Laboratory results are as
a. Tolerance often develops follows:

DBMCI, Ph: 9810150067, 9868387223, 27568832, E-mail: drbhatiainstitute@rediffmail.com, Visit us at: www.dbmci.com 19
Grand Test - 36

Na+ 145; K+ 5.0; Cl- 110; HCO3- 20 b. It exists in four subclasses, of which IgA 2 is
BUN: 40 mg/dL predominant
Serum creatinine: 3.5 mg/dL c. It can prevent attachment of microorganisms to
Urine sodium: 20 mmol/L epithelial cell membranes
Urine creatinine: 35 mg/dL d. It is prominent early in the immune response and is
Which of the following is the most likely cause of this the major class of antibody in cold agglutinins
patient’s acute renal failure?
a. Tubular necrosis 249. The most likely etiology for the eating
b. Insensible skin losses disorder anorexia nervosa is:
c. Renal artery embolism a. Decreased levels of luteinizing hormone – releasing
d. Cardiac failure hormone (LHRH)
b. Decreased level of growth hormone
243. As a consequence of severe liver damage, c. Decreased levels of insulin – like growth factor I
hepatic amino acid handling is deranged. In this (Somatomedin C)
situation, plasma levels of which of the following d. Psychiatric disorder
are likely to be lower than normal?
a. Ammonia (NH3) b. Ammonium (NH4+) 250. A young female patient has been admitted
c. Alanine d. Urea following a road accident with complaints of pain
in the right side of the chest. On examination she is
244. Which one of the following is diagnostic of found to have rapid and shallow breathing, a fast
complete transection of spinal cord after an Injury pulse rate and hyperresonant chest with absent
of three weeks’ duration? breath sounds on the right side. The most
A. The return of voluntary motor power below the appropriate immediate course of action would be
level of the lesion to:
B. The return of sensations below the level of the a. Perform a diagnostic chest aspiration on right side
lesion and put in a chest tube
C. The return of reflex activity without recovery of b. Order for urgent chest X-ray and start oxygen
sensations or voluntary motor power below the lesion c. Put the patient on positive pressure ventilation
D. The return of reflex activity with partial recovery d. Send for the cardiothoracic surgeon for exploration
of sensations below the lesion of chest

245. Which of the following statements concerning 251. A person with hypercalcemia caused by
gastrinoma is correct? sarcoidosis would have which of the following
a. Most are benign findings?
b. Most are unifocal a. A normal chest X-ray
c. They are associated with multiple endocrine b. Increased absorption of calcium from the
neoplasia (MEN) syndrome type II. gastrointestinal tract
d. They usually secrete multiple hormones. c. Normal urine calcium excretion
d. Increased serum parathyroid hormone level
246. A patient with diffuse cutaneous scleroderma
who had been stable for several years is recently 252. A comatose patient is being evaluated by
noted to have hypertension. This patient is at caloric stimulation of the vestibular apparatus.
significant risk of dying from: Cold- water irrigation of the right external
a. Thrombotic stroke auditory canal leads to deviation of both eyes to
b. Central nervous system hemorrhage the right for 2 min, followed by a slow drift back to
c. Renal failure the medline. This finding is most consistent with a
d. Pulmonary hypertension lesion in the.
a. Right labyrinth b. Midbrain
247. The commonest site of lodgement of a c. Medulla d. Cerebral hemispheres
pulmonary embolus is in the territory of:
a. Right lower lobe b. Right upper lobe 253. TUR (transurethral resection) syndrome is
c. Left lower lobe d. Left upper lobe due to:
a. Hyponatraemia b. Hyokalaemia
248. Human immunoglobulin A can be described c. Hypovolaemia d. Hypoxia
by which of the following statements?
a. It is the predominant immunoglobulin in plasma We have completely revised and updated our Notes &
Question Banks of all of 19 Subjects of undergraduate
syllabus. You need not read any other book!!

DBMCI, Ph: 9810150067, 9868387223, 27568832, E-mail: drbhatiainstitute@rediffmail.com, Visit us at: www.dbmci.com 20
Grand Test - 36

254. The most likely diagnosis for a patient with 261. In nine families surveyed, the numbers of
impotence and urinary incontinence who, over children per family were 4,6,2,2,4,3,2,1, and 7. The
years, sustains tremor at rest, bradykinesia, mean, median, and mode numbers of children per
rigidity, severe orthostatic hypotension, and family are, respectively.
anhidrosis is: a. 3.4, 2, 3 b. 3, 3.4, 2
a. The autonomic form of the Landry-Guillain-Barre c. 3, 3, 2 d. 3.4, 3, 2
syndrome
b. Shy-Drager syndrome 262. A study is undertaken to determine whether
c. Guanethidine intoxication drinking more than 8 cups of coffee a day is
d. Micturition syncope associated with hypertension. The blood pressure
readings were taken of persons who drink more
255. Duchenne’s muscular dystrophy is than 8 cups and persons who drink no coffee. The
characterized by: results are as follows:
a. Autosomal dominant inheritance
b. Onset in the second decade of life Hypertension Normal BP
c. Normal cardiac muscle
d. Universal elevation of serum creatine kinase > 8 cups 6 4
no coffee 2 7
256. Superficial extravasation of urine occurs in
the rupture of: Which of the following is the most appropriate test to
a. Bladder b. Prostatic urethra analyze the data?
c. Membranous urethra d. Bulbous urethra a. Chi – square test b. McNemar’s test
c. Fisher’s exact test d. Student t test
257. Patients with tuberous sclerosis are at an
increased risk of developing which of the following 263. Keloid is best treated by:
tumors? a. Intrakeloidal injection of triamcinolone
a. Meningioma b. Giant cell astrocytoma b. Wide excision and skin grafting
c. Pheochromocytoma d. Schwannoma c. Wide excision and suturing
d. Deep X-ray therapy
258. Which is not seen in keratoconus:
a. Munson sign b. Fleischer ring 264. All the following are steps necessary to plan
c. Scissor reflex d. Hypermetropia and conduct a case – control study EXCEPT:
a. Developing and testing research instruments
259. Which of the following statements concerning b. Defining the disease and exposure of interest
Friedreich’s ataxia is true? c. Selecting cases and defining a control group
a. Skin lesions represent a common manifestation of d. Determining the duration of the observational
this disorder (study) period.
b. The onset of symptoms occurs after age 25 years.
c. The molecular defect involves the expansion of a 265. As a new public health officer, you are tasked
trinucleotide repeat sequence. with developing criteria for a disease surveillance
d. Friedreich’s ataxia is associated with an increased system. All the following are important purposes
risk of cancer. for a public health surveillance system EXCEPT:
a. Detection of epidemics
260. Consider the following statements regarding b. Detection of rare but fatal conditions, such as
neuroblastoma: Alzheimer’s disease
1. 75% of the cases are encountered below the age of c. Description of trends and the natural history of a
4 years. health condition.
2. Spontaneous regression is known to occur d. Evaluation of hypotheses about the occurrence of a
3. Urinary level of VMA is elevated disease
4. Serum level of norepinephrine is raised
Of these statements: 266. Over 75% of the strength of the intact
a. 2, 3 and 4 are correct abdominal wall lies in the;
b. 1, 2 and 4 are correct a. Skin b. Subcutaneous tissue
c. 1, 2 , 3 and 4 are correct c. Aponeurosis d. Peritoneum
d. 1, 3 and 4 are correct
267. In country A there are 35 new cases of breast
cancer per 100,000 adult women per year; in

DBMCI, Ph: 9810150067, 9868387223, 27568832, E-mail: drbhatiainstitute@rediffmail.com, Visit us at: www.dbmci.com 21
Grand Test - 36

country B the number is 90 per 100,000. Which of 273. A 45-yrs-old male presents with vague
the following is the most likely explanation? discomfort in the right hypochondrium for the
a. Women in country A have a much higher rate of past three months. On examination, he is found to
nursing their infants. have 6 cm firm hepatic enlargement. X-ray of the
b. Women in country A are less likely to smoke chest reveals two sharply circumscribed round
cigaretters opacities of 5 cm each, in the left lower zone.
c. Women in country A receive more frequent Ultrasonography shows 3 to 4 anechoic round
preventive care, such as mammography lesions in the liver. The investigation of choice in
d. Women in country A are younger this case would be:
a. Serum alpha fetoprotein
268. All the following are clearly associated with b. Indirect hemagglutination test
acute episodes of asthma EXCEPT: c. CT of abdomen
a. Emotional stress d. Contrast examination of the gastrointestinal tract
b. Elevated ozone concentrations
c. Aspirin 274. A 21-years-old man present to the office for a
d. Caffeinated beverages sore throat .On examination, the patient is found
to be tall , with gynecomastia and testicular
269. A 30-yrs-old woman presents with a history of atrophy . Which is the most likely diagnosis ?
amenorrhoea and impaired vision of six months a. Testicular feminization syndrome
duration. Physical examination shows everything b.45, X (Turner’s syndrome)
normal except for pale optic discs and diminished c. Trisomy 21 (Down’s syndrome)
visual acuity. The most likely diagnosis is: d. 47, XXY (Klinefelter’s)
a. Pituitary adenoma
b. Craniopharyngioma 275. A child with Sprengel’s deformity would
c. Hypothalamic glioma most likely demonstrate all the following EXCEPT
d. Benign intracranial hypertension a. Limited abduction of the shoulder
b. Limited internal rotation of the shoulder
270. Exposure to 300 rem (3 Sv) of radiation will c. Elevation of one shoulder
most likely result in which of the following? d. Loss of normal contour of the neck
a. Bone marrow depression
b. Neurovascular syndrome 276. Treatment of acute myositis ossificans is:
c. Gastrointestinal syndrome a. Active mobilization b. Passive mobilization
d. Cardiovascular syndrome c. Immobilization d. Analgesics

271. Phacoemulsification is not done in: 277. Phalen’s test is positive in:
a. Presenile cataract a. Carpal tunnel syndrome
b. Immature cataract b. de Quervain’s tenosynovitis
c. Black cataract c. Fracture scaphoid
d. Posterior subcapsular cataract d. Anterior dislocation of hip

272. A 31/2 years-old girl is brought to your office 278. All the following are signs of complete airway
by her parents who are concerned about her obstruction EXCEPT
knock-knees (genu valgum). The parents state she a. Clutching the neck with thumb and fingers
was bowlegged as an infant and has becomes b. Inability to speak
progressively knock-kneed over the past 6 months. c. Inability to cough
The patient is otherwise healthy and has met all d. High –pitched noise with inhalation
developmental milestones. Height and weight are
within the 80th percentile .The femoral-tibial angle 279. A 58-year-old man presents your office with a
measure 14 degrees bilaterally. history of having an episode of sudden visual loss
The intramalleolar distance is 10cm within the knees of vision as similar to someone pulling a cover over
just touching. The most likely diagnosis is: his right eye. Vision returned to the right eye.
a. Blount’s disease Visions returned to the right eye after 10 min. this
b. Physiologic knock-knees visual field deffect is:
c. Malunion a. Scotoma b. TIA
d. Rickets c. Strabismus d. Esotropia

DBMCI, Ph: 9810150067, 9868387223, 27568832, E-mail: drbhatiainstitute@rediffmail.com, Visit us at: www.dbmci.com 22
Grand Test - 36

280. Most common type of dislocation of the hip is: b. Obesity reduces the ability to detect AAA on
a. Anterior physical examination
b. Posterior c. Most AAAs are detectable by physical examination
c. Central fracture dislocation d. Ninety-five percent of AAAs are palpable
d. Posterior dislocation with fracture
289. Auscultation of normal breath sounds of the
281. Catterall’s classification is useful for: chest demonstrates:
a. CDH b. CTEV a. Vesicular breath sounds that are high-pitched
c. Perthes disease d. Fracture neck femur b. Bronchial breath sounds heard over the trachea
c. Bronchovesicular breath sounds heard over the
282. On Physical examination ,Bell’s palsy is apices
characterized by all the following EXCEPT: d. Bronchial breath sounds that are the lowest in pitch
a. Drooping of the corner of the mouth and intensity.
b. Difficulty speaking
c. Inability to abduct the ipsilateral eye 290. Rout of administration for patient controlled
d. Drooling analgesia is:
a. Intravenous b. Intramuscular
283. Fibrous dysplasia with pigmentation and c. Oral d. Epidural
sexual precocity is seen in:
a. Neurofibromatosis 291. Post spinal hypotension in obstetrics patients
b. Albright’s syndrome is treated by:
c. Klinefelter’s syndrome a. Adrenaline subcutaneously
d. Turner syndrome b. Dobutamine infusion
c. Crystalloid infusion
284. A 51 year-old indigent man presents to the d. Dopamine infusion
emergency room with horizontal nystagmus, ataxic
gait, and confusion .Which of the following is the 292. All the following statements about physical
most likely diagnosis ? findings in a patient with peritonitis are true
a. Middle cerebral artery infarct EXCEPT:
b. Thiamine deficiency a. In extreme cases, abdominal rigidity (guarding)
c. Pellagra results in persistent abdominal wall stiffness, which
d. Viral encephalitis prevents movement of the abdomen during
inspiration.
285. All the following statements concerning b. The abdominal wall may be flaccid
tricuspid regurgitation are true EXCEPT : c. Having the patient lift the head so that the chin
a. Its is systolic murmur touches the chest will worsen the tenderness elicited
b. It does not radiate to the axilla on abdominal palpation
c. It causes prominent c waves of the venous pulse in d. Rebound tenderness results from the passive
the neck movement of the abdominal wall to its pretested
d. Inspiration decreases the murmur position

286. Not true about blood gas partition coefficient 293. True statements concerning colon cancer
is: include all the following EXCEPT:
a. Sevoflurane < isoflurane a. Lesions of the left colon commonly ulcerate and
b. Isoflurane < Enflurane lead to chronic, insidious blood loss
c. Desflurane < Nitrous oxide b. Lesions of the right colon usually do not change the
d. Sevoflurane < Desflurane appearance of the stool
c. Tumors arising in the descending colon present
287. Longest acting local anaesthetic is: with abdominal cramping, occasional obstruction, and
a. Lignocaine b. Chlorprocaine even perforation
c. Bupivacaine d. Tetracaine d. Neoplasms arising in the rectosigmoid often are
associated with hematochezia, tenesmus, and a
288. Which of the following statements is true of reduction in the caliber of the stool.
detection of abdominal aortic aneurysms (AAs) by
physical examination? 294. Most common cause of hypoxemia in
a. AAAs are not detectable by physical examination immediate post operative period is:
a. Low FIO2 b. Hyperventilation

DBMCI, Ph: 9810150067, 9868387223, 27568832, E-mail: drbhatiainstitute@rediffmail.com, Visit us at: www.dbmci.com 23
Grand Test - 36

c. Decreased metabolism d. Hypoventilation


 In AIPG 2006 Exam, 189 Questions came from our
295. Sudden decrease in tidal CO2 under general Notes & Question Bank!!!
anaesthesia suggests:  In AIIMS May 2006 Exam, 134 Questions came from our
a. Hypothermia b. Malignant hyperpyrexia Note & Question Bank!!!
c. Cardiac arrest d. Accidental extubation  Our Notes & Questions Bank are available as Postal
Coaching. For details contact:
296. All the following peripheral nerves and 09810150067, 09868387223, 011-27568832
function tests are correctly paired except:
a. Femoral nerve (L2-L4), knee extension
b. Axillary nerve (C5, C6) shoulder abduction
c. Radial nerve (C6 – C8), thumb extension
d. Musculocutaneous nerve (C5, C6), elbow extension “Dr. Bhatia Medical Institute”:

297. Pain reproduced by extending the wrist We have reasons: Why we are the best institute
against resistance with the elbow extended is most in entire India.
suggestive of : Ours is the only institute in the ENTIRE
a. Olecranon bursitis b. Tennis elbow WORLD, where we do classes for USMLE,
c. Golfer’s elbow d. Osteoarthritis PLAB, MD/MS & DM (Cardio) Entrance Exam.
 You can assess yourself our quality of
298. Involvement of pre-epiglottic space is seen in: teachers & teaching standard by:
a. Carcinoma larynx a) Very high standard of our Grand Test papers
b. Ludwig’s angina with their answers.
c. Submandibular gland infection b) Authenticity of answer key of AIIMS, AIPG
d. Vincent’s angina & UPSC exam.
c) High quality notes & question bank
299. False statement about tympanic membrane d) Our success rate
perforation is: : USMLE / PLAB = 100%
a. In central lesion there will be complete unilateral : MD/MS = 65 – 75%
deafness : DM (Cardio) = 80 – 85%
b. In central lesion there is painful deafness e) Largest network
c. Serous discharge is common : Doing classroom coaching in 19 cities all
d. Posterior perforations are usually safe over India.
: Doing Grand Test / Subject Test in 80 cities
300. Which of the following statements is true of all over India !!!
the crossed straight-leg causes ipsilateral pain. f. High class Faculty i.e. not only we have the
a. Straight –leg raising of the non-painful leg causes best faculty for PG entrance but also for
ipsilateral pain. superspecialty (DM Cardio) entrance exam
b. Straight-leg raising of the non-painful leg causes also!!!
worsened pain on the painful side g. Our test papers are set by those faculties who
c. Straight-leg causes pain on the non-painful side have high experience of setting actual papers
d. Straight-leg raising of the painful leg causes pain of AIIMS and AIPG Exams!!!
on both sides.
At Delhi Centre starting
1. Test & Discussion Batch on 10th of July.
2. Scholar Batch 20th of August.
3. Scholar Batch 24th Sept.

At Mumbai Centre
1. Regular Batch Starting on 29th of July.

DBMCI, Ph: 9810150067, 9868387223, 27568832, E-mail: drbhatiainstitute@rediffmail.com, Visit us at: www.dbmci.com 24
Answer Grand Test - 36
Grand Test - 36

1. B 58. A 115. C 172. B 229. B 286. D


2. B 59. C 116. A 173. A 230. A 287. C
3. A 60. C 117. D 174. D 231. C 288. B
4. D 61. C 118. D 175. B 232. C 289. B
5. D 62. C 119. C 176. B 233. D 290. A
6. C 63. A 120. D 177. D 234. D 291. C
7. D 64. A 121. D 178. A 235. B 292. C
8. A 65. C 122. B 179. B 236. A 293. A
9. B 66. B 123. D 180. B 237. A 294. D
10. D 67. C 124. B 181. B 238. C 295. D
11. A 68. A 125. D 182. C 239. B 296. D
12. B 69. D 126. A 183. D 240. A 297. B
13. A 70. D 127. B 184. C 241. A 298. A
14. D 71. B 128. A 185. C 242. A 299. A
15. A 72. D 129. C 186. D 243. D 300. B
16. B 73. A 130. D 187. D 244. C
17. A 74. B 131. C 188. C 245. D
See the result of today’s
18. C 75. D 132. A 189. C 246. C
exam on 8 July after 8
19. A 76. D 133. B 190. D 247. A
PM at our website:
20. B 77. B 134. A 191. B 248. C
www.dbmci.com
21. B 78. C 135. D 192. C 249. D
22. D 79. D 136. A 193. C 250. A Next Grand Test on
23. A 80. B 137. B 194. D 251. B Sunday, 6th August 2006.
24. C 81. C 138. C 195. B 252. D
Next Subject wise test is
25. B 82. A 139. B 196. B 253. A
Pathology On 9 July at 10
26. D 83. D 140. A 197. C 254. B
AM.
27. D 84. A 141. B 198. D 255. D
28. A 85. B 142. B 199. D 256. D In all centres of Kerala
29. D 86. A 143. D 200. B 257. B Tests are done on
30. B 87. C 144. D 201. D 258. D Monday.
31. B 88. D 145. B 202. D 259. C
32. A 89. B 146. A 203. A 260. C
33. D 90. C 147. D 204. C 261. D
34. B 91. B 148. D 205. C 262. C
35. B 92. A 149. A 206. D 263. A
36. A 93. D 150. A 207. D 264. ?
37. D 94. C 151. C 208. B 265. B
38. B 95. A 152. A 209. A 266. C
39. A 96. C 153. D 210. B 267. D
40. C 97. D 154. A 211. D 268. D
41. B 98. B 155. D 212. D 269. A
42. D 99. C 156. B 213. B 270. A
43. D 100. B 157. D 214. D 271. C
44. B 101. C 158. B 215. A 272. B
45. A 102. C 159. A 216. C 273. C
46. D 103. D 160. A 217. A 274. D
47. C 104. C 161. D 218. C 275. B
48. B 105. D 162. B 219. D 276. C
49. D 106. A 163. B 220. B 277. A
50. A 107. B 164. D 221. D 278. D
51. B 108. A 165. A 222. A 279. B
52. D 109. B 166. A 223. D 280. B
53. D 110. D 167. A 224. C 281. C
54. D 111. B 168. B 225. A 282. D
55. D 112. A 169. D 226. D 283. B
56. D 113. B 170. B 227. D 284. B
57. B 114. D 171. C 228. D 285. D

DBMCI, Ph: 9810150067, 9868387223, 27568832, E-mail: drbhatiainstitute@rediffmail.com, Visit us at: www.dbmci.com 25
Grand Test – 36 Explanatory Answer
Answers Grand Test – 36 (For Explanation of Answers P.T.O)
1. B 51. B 101. C 151. C 201. D 251. B
2. B 52. D 102. C 152. A 202. D 252. D
3. A 53. D 103. D 153. D 203. A 253. A
4. D 54. D 104. C 154. A 204. C 254. B
5. D 55. D 105. D 155. D 205. C 255. D
6. C 56. D 106. B 156. B 206. D 256. D
7. D 57. B 107. A 157. D 207. D 257. B
8. A 58. A 108. A 158. B 208. B 258. D
9. B 59. C 109. B 159. B 209. A 259. C
10. D 60. C 110. D 160. A 210. B 260. C
11. A 61. C 111. B 161. D 211. D 261. D
12. B 62. C 112. A 162. B 212. D 262. C
13. C 63. A 113. B 163. B 213. B 263. A
14. D 64. A 114. D 164. D 214. D 264. D
15. A 65. C 115. C 165. A 215. A 265. B
16. B 66. B 116. A 166. A 216. C 266. C
17. A 67. C 117. D 167. A 217. A 267. D
18. C 68. A 118. D 168. B 218. C 268. D
19. A 69. D 119. C 169. D 219. D 269. A
20. B 70. D 120. D 170. B 220. B 270. A
21. B 71. B 121. D 171. C 221. D 271. C
22. D 72. D 122. B 172. B 222. D 272. B
23. A 73. A 123. D 173. A 223. D 273. C
24. C 74. B 124. B 174. D 224. C 274. D
25. B 75. D 125. D 175. B 225. A 275. B
26. D 76. D 126. A 176. B 226. D 276. C
27. D 77. B 127. B 177. D 227. D 277. A
28. A 78. C 128. A 178. A 228. D 278. D
29. D 79. D 129. C 179. B 229. B 279. B
30. B 80. B 130. D 180. B 230. A 280. B
31. B 81. C 131. C 181. B 231. C 281. C
32. A 82. A 132. A 182. C 232. C 282. D
33. D 83. D 133. B 183. D 233. D 283. B
34. B 84. A 134. A 184. C 234. D 284. B
35. B 85. B 135. D 185. C 235. B 285. D
36. A 86. A 136. A 186. D 236. A 286. D
37. D 87. C 137. B 187. D 237. A 287. C
38. B 88. D 138. C 188. C 238. C 288. B
39. A 89. B 139. B 189. C 239. B 289. B
40. C 90. C 140. A 190. D 240. A 290. A
41. B 91. B 141. B 191. B 241. A 291. C
42. D 92. A 142. B 192. C 242. A 292. C
43. D 93. D 143. D 193. C 243. D 293. A
44. B 94. C 144. D 194. D 244. C 294. D
45. C 95. A 145. B 195. B 245. D 295. D
46. D 96. C 146. A 196. B 246. C 296. D
47. C 97. D 147. D 197. C 247. A 297. B
48. B 98. B 148. D 198. D 248. C 298. A
49. D 99. C 149. A 199. D 249. D 299. A
50. A 100. B 150. A 200. B 250. A 300. B

See the result of today’s exam on 10 July after 8 PM at our website: www.dbmci.com

DBMCI, Ph: 9810150067, 9868387223, E-mail: drbhatiainstitute@rediffmail.com, Visit us at: www.dbmci.com 1


Grand Test – 36 Explanatory Answer

1. The answer is b. Cause of death in hanging is compression of cervical


Primase synthesizes the short RNA molecules vasculature.
required for initiation of a new DNA strand. DNA
synthesis requires an RNA primer because 7. The answer is d.
polymerase, the major enzyme for DNA replication, The exposure of tissues to chronic hypoxia makes
can only add bases to an existing strand and cannot them rely more on anaerobic metabolism for the
begin a new strand de novo. DNA polymerase moves generation of energy as ATP and other high-energy
along the DNA in one direction and synthesizes both phosphates. Most tissues except for red blood cells
new DNA strands at once. Since DNA is always can metabolize glucose under anaerobic or aerobic
synthesized 5’ to 3’, one of the strands (the leading conditions (red blood cells do not have mitochondria
strand) is synthesized continuously, whereas the other for electron transport and must rely on other tissues to
(the lagging strand) is synthesized in short stretches generate glucose back from lactate). In most tissues, a
called Okazaki fragments. Helicase is the enzyme switch from aerobic to anaerobic metabolism greatly
responsible for unwinding increases glucose utilization and decreases energy
production. (A reduction of glucose utilization under
2. The answer is b. anaerobic conditions in bacteria is known as the
Although each type of mutation can cause protein Pasteur effect after its discoverer).
truncation, a frameshift mutation usually introduces a
stop codon very shortly after the frameshift. In coding 8. The answer is a.
sequences, there are three possible reading frames that Glycolysis results in a net production of 2 ATP per
can be translated, only one of which leads to synthesis glucose molecule. Glucokinase, which catalyzes the
of the correct protein. The other two reading frames first reaction of the pathway, requires hydrolysis of
generally have numerous stop codons throughout, one ATP to convert glucose to glucose-6-phosphate.
whereas the correct reading frame only has a stop Phosphofructokinase, phosphoglycerate kinase, and
codon at the very end. Small dele tions and insertions pyruvate kinase each generate one ATP The
will generally not affect protein size if the insertion or remaining enzymes in glycolysis do not generate or
deletion is a multiple of three bases (one codon). use ATP.
These types of insertions or deletions maintain the
same reading frame during translation. 9. The answer is B
Most specific and sensitive test for identification is
3. The answer is A DACTYLOGRAPHY.
First internal sign of putrefaction is found on under
surface of liver. 10. The answer is D.
Note: First external sign of putrefaction is seen on Oligomycin inhibits mitochondrial ATPase and thus
skin as greenish discoloration over caecum. prevents phosphorylation of ADP to ATP. It prevents
utilization of energy derived from electron transport
4. The answer is D for the synthesis of ATP Oligomycin has no effect on
• Ethmoidal polypi coupling but blocks mitochondrial phosphorylation so
- Mostly seen in cases of allergic and vasomotor that both oxidation and phosphorylation cease in its
rhinitis. presence.
- Usually multiple and bilateral
- Diagnosed on anterior rhinoscopy 11. The answer is A.
- Polypectomy is the treatment of choice. Lipogenesis is the main pathway for fatty acid
- Recurrence is common. synthesis. Lipogenesis occurs in the cytosol of cells in
many tissues such as liver, kidney, brain, lung,
mammary gland, and adipose tissue. Palmitate is the
end product of lipogenesis.
5. The answer is d.
A  turn consists of four residues in which the first 12. The answer is B
residue is hydrogen bonded to the fourth residue of  Rigor mortis first appears in involuntary muscles
the turn. Glycine residues are small and flexible, and and then in voluntary muscle.
proline residues generally assume a cis conformation.  In the involuntary muscle – rigor mortis first
It is these qualities that make these residues amenable appear in heart within an hour after death.
to tight turns.  In voluntary muscles, first appears in eyelids (3 to
4 hours).
6. The answer is C
Hanging causes large amount of injury to trachea.

DBMCI, Ph: 9810150067, 9868387223, E-mail: drbhatiainstitute@rediffmail.com, Visit us at: www.dbmci.com 2


Grand Test – 36 Explanatory Answer

13. The answer is c. - Aero-otitis Media


Purine salvage reactions convert purines, purine - Atelectatic ear
ribonucleosides, and purine deoxyribonucleoside to For cholesteatoma – Radical Mastoidectomy;
mononucleotides. Such salvage reactions require Coalescent Mastoiditis – simple mastoidectomy done.
much less energy than de novo synthesis. The liver is
the major site of purine nucleotide biosynthesis and 19. The answer is A
provides excess purines for other tissues that cannot This patient is suffering from an erotomanic delusion
synthesize purines. A defect in hypoxanthine-guanine – the delusion of having a special relationship with
phosphoribosyl transferase, one of the enzymes of another person, often someone famous.
purine salvage, is responsible for purine
overproduction and subsequent hyperuricemia 20. The answer is b.
observed in Lesch-Nyhan syn drome. According to Carl Jung, arche types are powerful
symbolic images that emerge from the collective
14. The answer is d. unconscious, that part of the psychic apparatus that
Under fed conditions, glucose is the preferred fuel for preserves the collective knowledge and experience of
most tissues. However, under starvation conditions, humankind. Archetypes emerge in dreams, fantasy,
glucose must be reserved for use by the central art, and free associations, and their interpretation
nervous system. There is a small initial decrease in provides useful insights into the patient’s intrapsychic
plasma glucose upon starvation but the concentration processes. In the scenario described in the question,
levels off after a time because of conversion of the snakes shedding their skins are usually interpreted
glycogen in the liver. Under starvation conditions, as a symbol for change and renewal. Primary
ketone bodies are made in the liver and provide a processes, manifest content of dreams, and phallic
major metabolic fuel source for skeletal and heart representations are concepts encountered in Freudian
muscle and can serve to meet some of the brain’s psychoanalytic theories.
energy needs.
21. The answer is B
15. The answer is A Trismus in parapharyngeal abscess is due to spasm of
Haemorrhagic otitis externa is caused by influenza medial pterygoid muscle.
virus. The haemorrhagic bullae on the tympanic
membrance and deep meatus rupture and cause blood 22. The answer is d.
stained discharge. Dysthymia is defined as a chronic depression that
lasts at least 2 years. Usually it begins in late
16. The answer is b. adolescence or early adulthood, and sometimes
A lesion to the left prefrontal area abolishes the patients describe being depressed for as long as they
normal mood-elevating influences of this area and pro can remember. Symptoms fluctuate but are usually
duces depression and uncontrollable crying. In not severe. Patients tend to have low self-esteem and
contrast, a comparable lesion to the right prefrontal perceive themselves as inadequate and inferior to
area may produce laughter, euphoria, and a tendency others. The somatic symptoms characteristic of major
to joke and make puns. depression or melancholia are less prominent in
dysthymia.
17. The answer is a.
Sleep apnea is the cessation of breathing during sleep 23. The answer is A
for 10 s or more. In obstructive sleep apnea, breath • Ebola virus does not cause haemorrhagic
ing stops due to airway blockage, while in central conjunctivitis.
sleep apnea the breathing stops due to an absence of • Enterovirus-70 and Cox sackie virus A-
respiratory efforts secondary to a neurological 24 do cause haemorrhagic conjunctivitis.
dysfunction. Features associated with obstructive • But regarding Echo virus, it is Echo virus
sleep apnea are excessive daytime somnolence, —34 that causes haemorrhagic conjunctivitis and not
snoring, restless sleep, and nocturnal awakening with Echo virus—18.
gasping for air. Patients often wake up in the morning
with dry mouth and headache. 24. The answer is c.
Hypochondriasis is defined by DSM-IV as a
18. The answer is C persistent fear, despite medical reassurance, that one
 Myringotomy is indicated in ASOM (acute has a serious physical illness. The patient’s beliefs are
suppurative otitis media) based on misinterpretation of benign bodily
 Other indications of myringotomy are: symptoms. The belief is not of delusional proportions,
- Serous otitis media and the condition must persist for 6 months for this

DBMCI, Ph: 9810150067, 9868387223, E-mail: drbhatiainstitute@rediffmail.com, Visit us at: www.dbmci.com 3


Grand Test – 36 Explanatory Answer

diagnosis to be made. It is estimated that from 3 to • In lateral view, the sphenoid sinuses gets
14% of patients seen in a general medical practice superimposed on each other.
may suffer from hypochondriasis.
29. The answer is d.
25. The answer is B The potency of an anti- psychotic describes its relative
• BERA (Brain stem evoked response audiometry) is ability to block postsynaptic dopamine receptors.
now widely used to estimate hearing thresholds in Haloperidol and fluphenazine are examples of high-
infants. potency neuroleptics. Chlorpromazine and
— It can he performed when an infant is sleeping or thioridazine are low-potency, while per phenazine and
after a feed or under sedation or general anaesthesia molindone are considered to have intermediate
(but in many cases it is avoided). potency. The potency of the neuroleptic will effect its
— It records electrical response in cochlear nuclei and therapeutic dosage (effective daily doses of
its central connection to brainstem to sound. haloperidol are usually between 5 and 20 mg, while
— It is popular in paediatric audiometry and for chlor promazine requires dosages of 200 to 600
detecting C.P. angle tumours. mg/d). Low- and high-potency neuroleptics also differ
• Cortical evoked response autiomentry (C.E.R.A.) in their side effect profiles. In general, the lower-
detects the responses at vertex. It is not reliable in potency medications are more anticholinergic and
young children because sedating and are more likely to cause hypotension.
— Of length of time it takes The higher-potency medications tend to cause more
— Affected by sleep, anaesthetics and sedatives extrapyramidal side effects.

26. The answer is d. 30. The answer is b.


The physical and emotional symptoms of Aplastic anemia is a rare, idiosyncratic, non-dose-
premenstrual dysphoric disorder (PMDD) are related side effect of carbamazepine. Stevens Johnson
restricted to the late luteal phase of the menstrual syndrome is a potentially life-threatening exfoliative
cycle and resolve 1 or 2 days after the onset of the dermatitis, also rarely associated with carbamazepine
menstruation. Although most women of childbearing treatment. Neuroleptic malignant syndrome, serotonin
age experience some symptoms of PMDD during syndrome, and malignant hyperthermia are not
some of their menstrual cycles, only 5 to 9% meet the associated with this medication.
criteria for the diagnosis.
31. The answer is B
27. The answer is d. • Pain along hip, back of thigh and loss of sensation
Clonidine, an 2 receptor agonist, is used to suppress along the lateral border of foot and decreased ankle
some of the symptoms of mild opioid withdrawal. jerk — suggests S1 nerve root compression.
Clonidine is given orally, starting with doses of 0.1 to • Neurologic deficit in disc prolapse.
0.3 mg three or four times a day. In outpatient
settings, a daily dosage above 1 mg is not Level Nerve Fot Motor Sensory Reflexes
recommended due to the risk of severe hypotension. Affected Weakness Loss
Clonidine is more effective on symptoms of L5-S1 S1 Weakness Over Ankle
autonomic instability, but is less effective than Of plantar lateral jerk
methadone in suppressing muscle aches, cravings, and Flexors of side of Sluggish
insomnia. Clonidine is particularly useful in the Foot foot. or absent
detoxification of patients maintained on methadone. L4–L5 L5 Weakness Over Ankle
Of EHL and dorsum Jerk
28. The answer is A Dorsiflexors of foot Normal
• The sphenoid sinuses are best shown on the full of foot and lateral
axial view but may be visible on the standard side of leg.
occipito-mental projection if mouth is opened during L3-L4 L4 Weakness Over great Knee
exposure. of extensors toe and jerk
• Postero-Anterior position (Schuller’s view) is good of knee medial Sluggish
for the nasal fossae, the anterior part of the frontal side of or
sinuses, the ethmoidal cells and also sphenoidal leg absent
sinuses. It is not good for Maxillary sinuses.
• Towne’s view is used to demonstrate interna 32. The answer is A.
auditory meatus, mastoid air cells, and antrum, These drugs all possess antimuscarinic (atropine-like)
superior and lateral semicircular canals. actions that often are sufficiently strong to cause all

DBMCI, Ph: 9810150067, 9868387223, E-mail: drbhatiainstitute@rediffmail.com, Visit us at: www.dbmci.com 4


Grand Test – 36 Explanatory Answer

the side effects and adverse responses associated with - Mallet finger: Avulsion of the extensor tendon from
atropine itself. Likewise, it is prudent to assume they base of distal phalanx.
share all the contraindications and precautions - Coxa vara: Intertrochanteric fracture
associated with atropine and that managing severe - Genu valgum: Condylar fracture of tibia (e.g.
overdoses of those drugs or drug groups will resemble bumper #)
(and need to be treated) in quite the same way as - Varus-valgus at ankle : Ankle injuries.
those of the prototype antimuscarinic. That would
include the potential need to administer 39. The answer is A
physostigmine, the acetyl cholinesterase inhibitor that Von — Rosen’s splint is used in CDH.
plays an important role in managing atropine In a children below the age of 1 yr. since the epiphysis
poisoning. of the femoral head is not ossified, it is difficult to
diagnose the dislocation of femoral head in plain X-
33. The answer is d. ray. Von-Rosen’s view is helpful for diagnosis.
The serotonin receptor agonist actions of sumatriptan, * Denis-Brown splint is used in CTEV.
can trigger intense vasoconstriction in various
vascular beds. The cerebral vasoconstrictor effects of 40. The answer is c.
these drugs contribute importantly to the relief they Chlorthalidone is a thiazide-like diuretic. If maximum
afford in migraine headaches. However, coronary dosages don’t yield the desired effects, there’s little
vasospasm can occur also, and these drugs are likelihood that switching to another thiazide or
therefore contraindicated for patients with coronary thiazide-like agent (e.g., hydrochlorothiazide,
artery disease. metolazone, many others) will do better. Likewise,
and given the relatively “flat” dose-response
34. The answer is B relationship for these drugs, nothing good is likely to
Trigger finger is seen in Rheumatoid Arthritis. It is be gained by adding “yet another” agent that works in
due to synovitis or a nodule in tendon. precisely the same way as the drug that has already
Mallet Finger : It is the avulsion of extensor tendon proven inadequate. If a maximum recommended dose
from base of distal phalanx. isn’t adequate, giving more of the same or a similar
drug won’t be better. So, in situations such as this, it’s
35. The answer is b. time to switch to a drug that is intrinsically more
Low K stores due to the effects of potassium- wasting efficacious and works via a different mechanism: a
diuretics such as hydrochlorothiazide increase loop diuretic.
susceptibility to cardiac glycoside toxicity.
41. The answer is b.
36. The answer is A Dextromethorphan is a centrally acting antitussive
Generally the frequency used in ophthalmics is 8-10 drug that is about as efficacious a cough-suppressant
MHZ. These high frequencies produce shorter as codeine. However, unlike codeine and
wavelengths which allow good resolution of minute hydrocodone (another useful antitussive in some
ocular and orbital structures. cases), dextromethorphan is not an opioid and so
lacks the potential for ventilatory suppression (and
37. The answer is d generalized CNS depression, analgesia, dependence
Heparin is a mixture of sulfated mucopolysaccharides risks, and other typical traits of the opioids).
and is highly acidic and highly charged. Protamine is Diphenhydramine and promethazine also have
a very basic polypeptide that combines with heparin. antitussive action. However, they too, can cause
The complex has no anticoagulant activity Excess generalized CNS (and ventilatory) depression.
protamine does have anticoagulant activity, so just Moreover, they have significant antimuscarinic
enough should be given to counteract the heparin effects. Although that may be good in terms of inhibit
effect. ing ACh-mediated bronchoconstriction, they may also
cause thickening of airway mucus, leading to some
38. The answer is B mechanical plugging of the airways with viscous
• Gun stock deformity is seen in supracondylar mucus deposits.
fracture of humerus due to malunion which is the
commonest complication and there is cubitus varus 42. The answer is D
deformity or Gun stock deformity. • Dislocation of the mandibular condyles is a
Deformities due to fracture : relatively uncommon condition occurs usually after
- Cubitus valgus : Fracture lateral condyle of humerus wide opening of mouth or — During convulsion and
- Dinner fork deformity: Colles’ fracture — Occasionally blow to jaw with mouth already open

DBMCI, Ph: 9810150067, 9868387223, E-mail: drbhatiainstitute@rediffmail.com, Visit us at: www.dbmci.com 5


Grand Test – 36 Explanatory Answer

• The head of condyle of one or both sides may slip positive. Depolarizing the membrane opens K+
anteriorly into infratemporal fossa, as a result of channels causing an increase in membrane
which inability to close mouth. It may be very painful conductance. Prolonged depolarization, whether
• Treatment of the dislocation is reduce the dislocation caused by an increase in extracellular K+ or by an
by manual manipulation. Sudden downward and action potential causes Na+ channels to inactivate.
backward movement is applied bilaterally with an Inactivation, which decreases the excitability of the
assistant supporting the head. nerve membrane, is observed in hyperkalemia and
during the absolute and relative refractory periods
43. The answer is d. following a normal action potential. The activity of
The clinical problems that arise with uric acid relate the Na-K pump can be reduced by a reduction, not an
to its poor solubility in body fluids— solubility that increase, in extracellular K+ concentration.
gets less as local pH falls. When a part of the body
(e.g., a joint in the great toe, which is a common site 48. The answer is B
of a gout attack) is damaged or otherwise insulted, Trident hand is seen in achondroplasia
uric acid crystals concentrate and precipitate in the Other anomally of achondroplasia
area. These crystals cause mechanical damage to the — Square iliac wings
joint surfaces and also evoke a typical inflammatory — ‘Champagne’ glass pelvic cavity
response. Leukocytes are attracted to the area, and in — Short, narrow sacrosciatic notch
an attempt to phagocytize the crystals release acidic — Posterior scalloping of vertebrae
metabolites that lower local pH, favoring precipitation — Small funnel shaped foramen magnum.
of even more uric acid and amplifying the entire
unwanted series of pathologic reactions. 49. The answer is d.
The rate of diffusion is described by Fick’s law,
44. The answer is b. which states that the flux of material across a
Lactulose is a synthetic, nonabsorbable disaccharide membrane is directly proportional to the area of a
(galactose-fructose). In moderate doses it acts as a membrane and the con centration difference of the
laxative. In higher doses it binds intestinal ammonia particles on either side of the membrane and is
and other toxins that accumulate in the intestine in inversely proportional to the thickness of the
severe liver dysfunction. These toxins, and perhaps membrane. In general, if all other properties of the
more so the ammonia, contribute to the signs and membrane are the same, the greater the lipid solubility
symptoms of encephalopathy. None of the other drugs of a particle, the greater its concentration in the
listed provide this benefit. membrane and, therefore, the greater its flux across
the membrane.
45. The answer is C
• Murrays test is for meniscus tear of both medial and 50. The answer is A
lateral meniscus. Presence of multiple perforations of tympanic
Other test for meniscus injury is Apley’s grinding test. membrane is a classical sign of tubercular otitis
media.
46. The answer is d.
These findings are characteristic of what one would 51. The answer is B
expect with long-term (and high dose) systemic • The Colour Doppler visually demonstrates patterns
glucocorticoid therapy (i e., prednisone and many and direction of blood flow. Various colours indicate
others, but not beclomethasone, which is given by oral change in direction and velocity of flow.
inhalation and is not absorbed appreciably). • In colour doppler system the pulses along each scan
Psychoses, peptic ulceration with hemorrhage (coffee- line are divided on return to transducer, and some are
grounds stool, indicative of gastric bleeding) or used to provide imaging information and the rest are
without (possibly causing guaiac positive stools) used to calculate the mean doppler shift within small
increased susceptibility to infection edema, pixels of image. This mean shift information is coded
osteoporosis myopathy, and hypokalemic alkalosis on a colour scale and displayed as a colour map over
can occur. the gray scale image. The Doppler shift frequency is
directly proportional to the blood velocity. Choice of
47. The answer is c. colour is an arbitrary; usually shades of blue and red
Orange juice contains a significant amount of K+ are used to represent flow towards and away from
Consuming a large amount of K+ when the transducer, with paler shades of colour representing
extracellular volume is low (dehydration) can cause a higher velocities.
significant rise in extracellular K+. An increase in
extracellular K+ makes the membrane potential more 52. The answer is d.

DBMCI, Ph: 9810150067, 9868387223, E-mail: drbhatiainstitute@rediffmail.com, Visit us at: www.dbmci.com 6


Grand Test – 36 Explanatory Answer

Abnormalities in coronary blood flow resulting in the duodenum. These muscular actions are under both
ischemia of the ventricular muscle will lead to a hormonal and neural control. Cholecystokinin
current of injury, which is reflected as an upward or contracts gallbladder smooth muscle by a direct action
downward shift in the ST segment of the ECG on the muscle and through activation of vagal afferent
recording. The electrical activity of the heart does not fibers leading to a vago-vagal reflex. Relaxation of
reflect changes in ventricular contractility, blood sphincter of Oddi smooth muscle occurs via activation
pressure, ejection fraction, or total peripheral of inhibitory enteric nerves. Vagal stimulation, which
resistance, although all of these can be altered by is cholinergically mediated and blocked by atropine,
changes in coronary blood flow. also promotes gallbladder contraction.

53. The answer is d. 57. The answer is B


Arterial vessels arranged in parallel supply the major • Hernia through the foramen of bochdalek is really
organs of the body. The relative resistance of each the persistence of pleuroperitoneal canal and the
parallel pathway determines the distribution of blood opening is in the dome of diaphragm posteriorly. It is
flow through a par allel network of vessels. The ratio the most common diaphragmatic hernia children.
of that resistance to the total resistance will determine There is classic triad of:
the percentage of flow going through each of the — Respiratory distress
parallel networks. The other factors mentioned would — Apparent dextrocardia
influence the magnitude of the cardiac output, which — Scaphoid abdomen
is the total amount of blood flowing through the entire • This type of hernia is apparent shortly after birth
system. However, none of these factors have any with over 80% presenting on Left side. There is
direct effect on the way blood flow is distributed to radiological appearances of bowel in hemithorax and
the organs of the body. mediastinal shifting.

54. The answer is D 58. The answer is a.


• Invertogram is done for detection of imperforate Expanding the lungs lowers the intrathoracic pressure,
anus. It is done after six hours of birth in infant u which causes the airways to expand. Expanding the
sufficient air may have collected in large intestine to airways increases their volume and, therefore, the
cast a X-ray shadow, volume of the anatomical dead space. Expanding the
airways also causes a decrease in airway resistance.
55. The answer is d. Maximum static inspiratory pressure is maximal at
Segmentation, the primary motility pattern of the function residual volume and decreases as lung
digestive period in humans, is defined as irregular and volume is increased. Lung compliance is decreased;
uncoordinated contraction of the circular muscle that is, the lungs are stiffer at high lung volumes.
layer. This pattern, which develops in response to Alveolar pressure is atmospheric at residual volume
intestinal wall distention, is determined by activation and at total lung capacity
of preprogrammed neural circuits within the
myenteric plexus. The contractile frequency of the 59. The answer is c.
small intestine decreases in an aborad direction. The V/Q mismatches will cause arterial oxygen levels
contractions, which are caused by underlying changes (Pao2 to decrease. Decreased Pao2 will stimulate the
in smooth-muscle electrical activity called electrical peripheral chemoreceptors, which, in turn, will
slow waves, decrease from approximately 11 per increase alveolar ventilation and decrease PaCo2. The
minute in the duodenum to approximately 6 to 7 per decreased PaCo2 will cause a respiratory alkalosis
minute in the distal ileum. Elimination of extrinsic (increasing pH). Hypoxemia will also cause lactate
input to the bowel wall has little functional effect on levels to rise, increasing the anion gap (and blunting
bowel motility during the digestive period. the rise in pH). The fall in Pao2 causes the A — a
Segmentation develops immediately upon the delivery gradient to rise.
of food into the small intestine and is accompanied by
abolishment of the migrating motor complexes 60. The answer is C
characteristic of the interdigestive period.  Phase – II blocker is scoline.
 Depolarising muscle relaxants have dual
56. The answer is d. mechanism of action.
Cholecystokinin is released from the upper small — Phase — I: Persistent depolarisation of muscle end
intestine in response to partially hydrolyzed dietary plate and is rapid onset.
lipids and proteins and promotes gallbladder — Phase —II: Slow onset, desensitization of the
emptying. Gallbladder contraction and sphincter of receptor to acetvlcholine
Oddi relaxation are necessary for delivery of bile into

DBMCI, Ph: 9810150067, 9868387223, E-mail: drbhatiainstitute@rediffmail.com, Visit us at: www.dbmci.com 7


Grand Test – 36 Explanatory Answer

61. The answer is c. gland. This zone undergoes rapid involution after
The intracellular Na concentration of renal epithelial birth.
cells is pumped out of renal epithelial cells by Na- K
pumps located on the basolateral surface of the
epithelial cells. The Na/H exchanger and the Na- 66. The answer is B
glucose transporter are located on the apical surface of • Development of true allergy or antibody formation
the epithelial cells. Na+ is transported from the may of course occur in days or weeks following
peritubular spaces to the capillaries by solvent drag. exposure to any of non depolarising muscle relaxants
(i.e. ALCURONIUM)
62. The answer is c. • They also cause TACHYPHYLAXIS.
GFR will decrease is there is a decrease in the net
glomerular capillary pressure or the flow of fluid 67. The answer is c.
through the glomerulus. The net glomerular capillary The Sertoli cells rest on a basal lamina and form a
pressure (for Starling forces) is equal to the layer around the periphery of the seminiferous
glomerular capillary pressure minus the sum of the tubules. They are attached to each other by
plasma oncotic pressure and intrarenal pressure. specialized junctional complexes that limit the
Compression of the renal capsule increases the movement of fluid and solute molecules from the
intrarenal pressure and therefore decreases the net interstitial space and blood to the tubular lumen, and
capillary filtration pressure. Constriction of the thus form a blood testis barrier that provides an
efferent arteriole increases glomerular capillary immunologically privileged environment for sperm
pressure. Decreasing the concentration of plasma maturation. Sertoli cells are intimately associated with
protein will decrease the plasma oncotic pressure and developing spermatozoa and play a major role in
lead to an increase in GFR. germ-cell maturation. They secrete a variety of serum
proteins and an androgen protein into the tubular fluid
63. The answer is A in response to FSH and testosterone stimulation.
• Mendelson’s syndrome is acid aspiration syndrome. Testosterone is synthesized and secreted by the
Aspiration of acid gastric contents cause a chemical interstitial Leydig cells. Estrogen is produced in small
trauma to bronchial and alveolar epithelia i.e. acute amounts by the Sertoli cells before puberty.
exudative pneumonitis
• Mendelson’s. syndrome usually occur with material 68. The Answer is A
(gastric acid) at a pH of 2. 5 or below, but known. to Astigmatism It is that condition of refraction wherein
occur with fluid of a neutral PH as well. a point focus of light cannot be formed upon the
retina. It includes those anomalies in the optical
64. The answer is a. system wherein an appreciable error is caused by the
Secretion of antidiuretic hormone (ADH) and unequal refraction of light in different meridians.
oxytocin by the neurohypophysis is regulated in the Hence it may be an error either of curvature, of
hypothalamic supraoptic and paraventricular nuclei, centering or of refractive index. There fore; this
respectively. This hypothalamic control of secretion question is controversial as both the options are given.
of pituitary hormone (inhibitory as well as releasing) But since the commonest form of astigmatism is due
in the case of the neurohypophysis is by direct neural to curvature defect, this will be the answer.
connection, and, in the case of the adenohypophysis,
by humoral factors conveyed by a microcirculation 69. The answer is d.
known as the hypothalamic-hypophyseal portal Free nerve endings contain receptors for temperature,
system. pain, and crude touch. However, fine touch, pressure,
and vibration are detected by nerve endings contained
65. The answer is c. within specialized capsules that transmit the stimulus
Because it lacks 3-dehydroxysteroid, the enzyme to the sensory receptors. Muscle length is encoded by
that converts pregnenolone to progesterone (the initial the primary nerve endings of Ia fibers, which are
step in both glucocorticoid and mineralocorticoid located on intrafusal fibers within the muscle spindle.
synthesis), the fetal cortex synthesizes primarily
dehydroepiandrosterone. This steroid is released as its 70. The answer is D
sulfate and is metabolized further to estrogen and • There is substantial protection against ischemia and
androgen by the placenta. During fetal life, the hypoxia is provided by. just 1—3°C hypothermia.
adrenal cortex consists of a thin subcapsular rim, Hypothermia reduces the tissue metabolic rate about
which eventually gives rise to the adult cortex, and a 8%/°C.
thick inner fetal cortex, which constitutes 80% of the • It decreases the cerebral metabolic rate and is
cerebroprotective during episode of cerebral ischemia

DBMCI, Ph: 9810150067, 9868387223, E-mail: drbhatiainstitute@rediffmail.com, Visit us at: www.dbmci.com 8


Grand Test – 36 Explanatory Answer

The protection afforded by mild hypothermia is so cm. Its measurement is inferred by subtracting 1.2 cm
great that reduced core temperature 34o probably from ft diagonal conjugate.
indicated in 3. Diagonal conjugate is the distance between the
— Carotid artery surgery lower border of symphysis pubis to the midpoint on
— Neurosurgery the s promontory. It is measures 12 cm. It is measured
— Procedures where tissue ischemia can be clinically during pelvic assessment.
anticipated 4. Obstetric conjugate can be calculated by
— Traumatic brain injury subtracting 1.5-2 cm from the diagonal conjugate.
—ARDS
- Malignant hyperthermia. 74. The answer is b
• Hypothermia is used in cardiac surgery as cooling B. burgdorferi, the causative agent of Lyme disease,
does slow heart and greatly depresses body elicits an acute antibody response. IgM appears within
metabolism. days to a few weeks following tick bite, and IgG
• Dysarhythmia occurs at temperature below 30oC, appears a few weeks later. IgG persists; IgM does not.
spontaneous ventricular fibrillation may be seen but is Cross-reactions occur with other treponemes.
not likely above 28oC. So hypothermia is Fifth disease is a viral exanthem commonly seen in
arrhythmogenic. children 8 to 12 years old. Children are ill for a few
days but recover without incident. Unfortunately, if a
71. The answer is b. pregnant female acquires the disease in the first
The Ruffini ending is a tonic receptor that produces a trimester of pregnancy, the fetus is at risk. The
train of action potentials proportional to the intensity causative agent is thought to be a Parvovirus
of pressure applied to the skin. The Pacinian (Parvovirus B19).
corpuscle is a very rapidly adapting receptor that fires
once or twice in response to skin deformation. It can 75. The answer is D
produce a continuous train of action potentials if the
stimulus is repetitively applied and withdrawn. 76. The answer is d.
Therefore, the Pacinian corpuscle is used to encode Of the intestinal amebas, Entamoeba hartmanni,
vibration. Entamoeba coil, Entamoeba polecki, and Entamoeba
nana are considered nonpathogenic. E. histolytica is
72. The answer is D distinctively characterized by its pathogenic potential
Of the many methods available for antigen and for humans, although infection with this protozoan is
antibody detection, LA, ELISA, RIA, CIE, and COA commonly asymptomatic (causing “healthy carriers”).
are the most widely used. Latex agglutination (LA) Symptomatic amebiasis and dysentery occur when the
employs latex polystyrene particles sensitized by trophozoites invade the intestinal wall and produce
either antibody or anti gen. LA is more sensitive than ulceration and diarrhea. Peritonitis can occur, with the
CIE and COA but slightly less sensitive than either liver the most common site of extraintestinal disease.
RIA or EIA. LA has been used to detect H. The life cycle of the ameba is simple by comparison.
influenzae, Neisseria meningitidis, and S. pneumoniae There is encystment of the troph, followed by
antigens in cerebrospinal fluid. LA has also been used excystation in the ileocecal region. The trophs
for detection of cryptococcal antigen. Most recently, multiply and become established in the cecum, where
LA has been widely used for rapid detection of group. encystation takes place and results in abundant
A streptococcal antigen directly from the pharynx. amebas, cysts, and trophozoites. Infection is spread by
The test is rapid (five minutes), sensitive the cysts, which can remain for weeks or months in
(approximately 90%), and specific (99%). appropriately moist surroundings.

73. The answer is A 77. The answer is B


Obstetric conjugate is the distance between the sacral The water in amniotic fluid is completely changed and
promontory and the .nearest point on the posterior replaced every 3 hours.
surface of the symphysis pubis. Important facts about amniotic fluid:
1. Amniotic fluid is of both maternal and foetal origin.
Important points: 2. It forms as a transudate from maternal serum.
1. Obstetric conjugate measures 10 cm. It cannot be 3. It forms as a transudate across the umbilical cord or
clinically measured but is to be inferred from diagonal from the foetal circulation in placenta or secretion
conjugate or by lateral radiopelvimetry. from the amniotic epithelium.
2. True/anatomical conjugate is the distance between 4. Contributions from foetal urine.
midpoint of sacral promontory to the inner margin of 5. Secretion from tracheobronchial tree and across the
CT. upper border of symphysis pubis. It measures 11 foetal skin.

DBMCI, Ph: 9810150067, 9868387223, E-mail: drbhatiainstitute@rediffmail.com, Visit us at: www.dbmci.com 9


Grand Test – 36 Explanatory Answer

6. The fluid is alkaline with a specific gravity of 1.010 5. Approximation of these vessels to syncytial
(osmolality 250 mOsm/l suggests foetal maturity). surface.
7. Meconium stained liquor Indicates foetal distress. At term the covering of the villi may be reduced to a
thin layer of syncytium with minimum connective
78. The answer is C tissue.
Dermatomycoses are cutaneous mycoses caused by The changes that suggest a decrease in efficiency for
three genera of fungi: Microsporum, Trichophyton, placental exchanges are:
and Epidermophyton. These infections are called tinea 1. Thickening of basement membranes of capillaries
or ringworm, a misnomer that has persisted from the and trophoblast.
days when they were thought to be caused by worms 2. Obliteration of certain foetal vessels
or lice. Tinea capitis (ringworm of the scalp) is due to 3. Deposition of fibrin on the surface of villi.
an infection with M. canis or T. tonsurans. It usually 4. Deposition of fibrin in the basal and chorionic
occurs during childhood and heals spontaneously at plates.
puberty. Circular areas on the scalp, with broken or no
hair, are characteristic of this disorder. Tinea corporis 81. The answer is c
(ringworm of the body) is caused by M. canis and Whooping cough is caused by Bordetella pertussis.
Trichophyton mentagrophytes. This disorder affects Bordetella infection is confirm by culture on specific
smooth skin and produces circular pruritic areas of media or direct fluorescent microscopy.
redness and scaling. Both tinea cruris (ringworm of
the groin, “jock itch”) and tinea pedis (ringworm of 82. The answer is A
the feet, athlete’s foot) are caused by T rubrum, T There are no known toxins, hemolysins, or cell-wall
mentagrophytes, or E. floccosum. These common constituents known to play a role in the pathogenesis
conditions are pruritic and can cause scaling. Tinea of disease by Brucella. Rather, the ability of the
versicolor (pityriasis versicolor) is not a organisms to survive within the host phagocyte and to
dermatomycotic condition but, rather, a superficial inhibit neutrophil degranulation is a major disease-
mycosis now thought to be caused by M. furfur. The causing factor. In infectious abortion of cattle caused
disorder is characterized by chronic but asymptomatic by Brucella, the tropism for placenta and the chorion
scaling on the trunk, arms, or other parts of the body. is a function of the presence of erythritol in allantoic
and amniotic fluid.
79. The answer is D
Rickettsiae are small bacteria that are obligate, intra 83. The answer is D
cellular parasites. Most but not all rickettsiae are The course of hepatitis B infection in mother is not
transmitted to humans by arthropods. Coxiella is affected by pregnancy. Fulminant hepatitis may be
transmitted through the respiratory tract rather than seen occasionally. Transplacental viral infection from
through the skin, and B. henselae from animal mother to the fetus is supposed to be rare but main
scratches. Coxiella may cause chronic endocarditis transmission of infection occurs by ingestion of
that is not very responsive to either antimicrobial infected material during delivery or exposure
therapy or valve replacement. B. henselae is a subsequent to birth. The infant may also acquire
fastidious, gram-negative rod that causes bacillary infection by breast-feeding.
angiomatosis, a disease that forms dermal or Infants at risk should be administered hepatitis B
subcutaneous nodules. The role of B. henselae in cat- immune globulin very soon after birth followed
scratch disease has recently been recognized. promptly by hepatitis B vaccine. A second and third
Molecular taxonomic studies have indicated that the dose of vaccine is given at 1 and 6 months of age.
causative organism is more closely related to
Bartonella than to Rochalimaea, hence the name 84. The answer is A
change. Enterococci cause a wide variety of infections ranging
from less serious—for example, urinary tract
80. The answer is B infections—to very serious, such as septicemia. A
The following histological changes that accompany gram-positive coccus resistant to penicillin must be
placental growth and ageing are suggestive of an assumed to be entero coccus until other, more
increase. In efficiency of transport and exchange to definitive biochemical testing places the isolate in one
meet the increasing metabolic requirements of the of the more esoteric groups of gram-positive cocci.
foetus: Once isolated, there are a variety of tests to speciate
1. Decrease in thickness of the syncytium. enterococci. However, penicillin-resistant, non_
2. Partial reduction of Langhans cells vancomycin-resistant, gram-positive cocci are most
3. Decrease in the stroma likely E. faecium. There are a variety of mechanisms
4. Increase in the number of capillaries for vancomycin resistance in E. faecium and they

DBMCI, Ph: 9810150067, 9868387223, E-mail: drbhatiainstitute@rediffmail.com, Visit us at: www.dbmci.com 10


Grand Test – 36 Explanatory Answer

have been termed Van A, B, or C. These isolates have degrades the original RNA, leaving a DNA-DNA
become one of the most feared nosocomial pathogens duplex; and, finally, it is responsible for integrating
in the hospital environment. Unfortunately, no the new viral DNA hybrid into the host genome.
approved antibiotics can success fully treat
vancomycin-resistant enterococci (VRE)—only some 88. The answers is D
experimental antibiotics such as Synercid. Capsules are found in both gram-positive and gram-
negative bacteria. With the exception of those found
85. The answer is b in Bacteroides fragilis, capsules are not in and of
Epstein Barr virus (EBV) is a herpesvirus that causes themselves toxic but rather are antiphagocytic and are
a number of syndrome most common is infectious immunologic (or serologic) determinants. Some
mononucleosis. It is a ubiquitous enve1op DNA virus. examples of capsular components are the following:
Complement fixation antibody measurements against 1. Sialic acid polymers are found in group B N.
viruses may be useful diagnostic tool if two serum meningttidis. This identical polymer is also found in
samples are collected from the patient. The acute E. coli K1.
serum is collected as early in the illness as Possible, 2. Group A streptococci in the early stages of growth
and the convalescent serum is collected 7 to 10 days have hyaluronic acid capsules. The capsule, however,
later. Both are included in IM same test run against is rapidly destroyed by the organism’s own
measles CF antigen. A Positive diagnostic test occurs hyaluronidase.
in the antibody titer when the convalescent serum is 3. B. anthracis, the causative agent of anthrax, is the
fourfold or greater than the acute sample. The sera are only bacterium to possess a polypeptide capsule that
diluted in a twofold manner (1:2, 1:4, 1:8, etc.) for is a polymer of glutamic acid.
quantitation. The measures given above (40 and 80) 4. S. pneumoniae type 3 has a repeating
are considered essentially the same level of antibodies polysaccharide capsule of glucose and glucuronic
in both samples. This patient must have had measles acid.
in the past and has the same amount of antibody in
both serum samples. 89. The answer is B
Infectious mononucleosis is an acute disease most This is a case of pre-eclampsia. Pre-eclampsia is a
commonly seen in younger people. It is characterized syndrome complex characterized by hypertension to
by a proliferation of lymphocytes lymph node the extent of 140/90 mmHg or more with oedema or
enlargement pharyngitis, fatigue and fever. Infection proteinuria or both induced by pregnancy after 20th
in young children is usually either asymptotic or week. The pregnancy is of 35 weeks duration and
characteristic of an acute upper respiratory infection hence foetal lung maturity is not complete and FHS is
Diagnosis is usually made by a Positive heterophil 140/mm, which indicates healthy foetus.
test. Heterophil antibodies are those that occur in one Hence considering these facts the mother should be
species (human) and react with antigens of a different hospitalized and BP should be controlled with
species. The heterophil test may be insensitive (30 to antihypertensive drugs. The aim should be to prolong
60%) in children. Definitive diagnosis is made by pregnancy till at least 37 weeks followed by induction
detection of antibodies to EBV components. of labour. During this period close monitoring of BP
and FHS should be done
86. The answer is A The main danger is development of eclampsia.
Treatment of abdominal pregnancy is immediate
laparotomy. There is a high risk of severe 90. The answer is c.
haemorrhage and hence 2 litres of blood should Many clinical and experimental studies have looked at
always be kept ready. the optimum bowel preparation and preoperative
The blood vessels supplying the placenta should be regimen for elective colonic surgery to reduce the
always ligated as removal of placenta always leads to postoperative infectious complications of wound
severe bleeding. - infection, intraabdominal abscess, and anastomotic
Almost all cases of abdominal pregnancy follow early leakage. Currently a postoperative rate of wound
rupture or abortion of a tubal pregnancy into the infection of only 5% can be attained by combining
pentoneal cavity. mechanical cleansing, oral antibiotics, and
perioperative parenteral antibiotics. The type of
87. The answer is c. mechanical cleansing does not matter as long as it is
The replication of a retroviral genome is dependent on effective. Preoperative oral antibiotics may be
the reverse transcriptase enzyme, which performs a administered one or more days prior to surgery and
variety of functions. It builds a complementary strand should cover aerobes and anaerobes (e.g., neomycin-
of DNA for the viral RNA template; it builds a second erythromycin). Parenteral antibiotics effective against
DNA strand complementary to the previous DNA; it aerobes and anaerobes (e.g., cefoxitin) should be

DBMCI, Ph: 9810150067, 9868387223, E-mail: drbhatiainstitute@rediffmail.com, Visit us at: www.dbmci.com 11


Grand Test – 36 Explanatory Answer

administered on call to the operating room as a single Early deceleration is not associated with foetal
dose and no more than 24 h postoperatively Both hypoxia, acidemia or low APGAR score. Head
antibiotic regimens yield maximum pro phylaxis compression leads to vagal stimulation that in turn
without fostering resistant transformation of microbes. probably leads to early deceleration.
Procedures that require operative time greater than 3 h Late deceleration can be an index of either uterine
or that involve the extraperitoneal rectum are perfusion or placental function. It is associated with
associated with an increased risk of infectious foetal hypoxia. Variable deceleration pattern is the
complications. commonest type of deceleration observed during
labour and it is mainly seen in cord compression.
91. The answer is b. Prolonged or persistent deceleration means isolated
Dopamine has a variety of pharmacologic deceleration lasting more than 60-90 s. Prolonged
characteristics that make it useful in critically ill deceleration is not an ominous FHR pattern.
patients. In low doses [1 to 5mg/(kg, min)], dopamine
affects primarily the dopaminergic receptors. 93. The answer is d
Activation of these receptors causes vasodilation of The significant observation in this question is the
the renal and mesenteric vasculature and mild description of lymphangitic inflammatory streaking
vasoconstriction of the peripheral bed, which thereby up the inner aspect of the patient’s leg. This is highly
redirects blood flow to kidneys and bowel. At these suggestive of a streptococcal infection, and the
low doses, the net effect on the overall vascular presumptive therapy should be high doses of a
resistance may be slight. As the dose rises [2 to 10 bactericidal antibiotic. Penicillin remains the mainstay
mg/(kg, min)], 1-receptor activity predominates and of therapy against presumed streptococcal infections.
the inotropic effect on the myocardium leads to Most streptococcal cellulitis is adequately treated by
increased cardiac output and blood pressure. Above penicillin, elevation of the infected extremity, and
10 mg/(kg. min), -receptor stimulation causes attention to the local wound to ascertain adequate
peripheral vasoconstriction, shifting of blood from local drainage and absence of any persisting foreign
extremities to organs, decreased kidney function, and body However, the clinician must be alert to the
hypertension. At all doses, the diastolic blood possibility of a more fulminant and life- or limb-
pressure can be expected to rise; since coronary threatening infection by clostridia, microaerophilic
perfusion is largely a result of the head of pressure at streptococci, or other potentially synergistic
the coronary ostia, coronary blood flow should be organisms that can produce rapidly progressive deep
increased. infections in fascia of muscle. Smears and cultures of
drainage fluid or aspirates should be taken. Close
92. The answer is A observation of the wound is essential, and aggressive
The ominous foetal heart rate patterns are: debridement in the operating room is mandatory at the
1. Loss of beat to beat rhythm. slightest suggestion that fasciitis or myonecrosis may
2. Late deceleration be ensuing.
3. Variable deceleration.
Beat to beat variability 94. The answer is c
Baseline variation of foetal heart is an index of CVS Tension pneumothorax is a life-threatening problem
function and is regulated by autonomic nervous requiring immediate treatment. A lung wound that
system. behaves as a ball or flap valve allows escaped air to
Decreased beat to beat variability is diagnosed when build up pressure in the intrapleural space. This
the foetal heart rate baseline is flat or nearly flat. causes collapse of the ipsilateral lung and shifting of
Diminished beat to beat variability may be an the mediastinum and trachea to the contralateral side,
ominous sign that indicates a seriously compromised in addition to compression of the vena cava and
foetus. Loss of variability with deceleration is contra lateral lung. Sudden death may ensue because
associated with foetal acidemia. Loss of variability is of a decrease in cardiac output; hypoxemia and
a result of metabolic acidemia that causes depression ventricular arrhythmia. To accomplish rapid
of fetal brainstem or the heart itself. decompression of the pleural space, a large-gauge
FHR acceleration refers to an increase in foetal heart needle should be passed into the intrapleural ca
rate above the baseline and deceleration to a decrease through the second intercostal space at the
below the baseline. midclavicular line. This may be attached temporarily
The timing of deceleration in reference to uterine to an under seal with sub sequent insertion of a chest
contraction is divided into early, late and variable. tube after the life-threatening urgency has been
1. Early deceleration: Head compression relieved.
2. Late deceleration: Uteroplacental insufficiency.
3. Variable deceleration: Cord compression.

DBMCI, Ph: 9810150067, 9868387223, E-mail: drbhatiainstitute@rediffmail.com, Visit us at: www.dbmci.com 12


Grand Test – 36 Explanatory Answer

95. The answer is A minimizing the quantity of dietary fluids and


The first two statements are correct. endogenous secretions in the gastrointestinal tract. A
During active labour uterus is transformed into two period of 4 to 6 weeks of TPN therapy is warranted to
distinct parts. The actively contracting upper segment allow for spontaneous closure of a low-output distal
that becomes thicker as labour advances and the lower fistula. Should conservative management fail, surgical
portion, comprising lower segment of uterus to cervix closure of the fistula is performed.
is relatively passive compared with upper segment
and develops into thin walled passage for the foetus. 97. The answer is d.
Upper segment contracts, retracts and expels the The management of malignant tumors may be guided
foetus and in response the ripened cervix and lower by knowledge obtained by grading and staging the
uterine segment dilates and expands. tumors. Histologic grading reflects the degree of
Uterine contractions of normal labour are anaplasia of tumor cells. Tumors in which histologic
characterized by gradient of myometrial activity being grading seems to have prognostic value include soft
greatest and lasting longest at fundus i.e., fundal tissue sarcoma, transitional cell cancers of the
dominance and diminishing towards the cervix. It is bladder, astrocytoma, and chondrosarcoma. Grading
reported that in addition to the gradient of activity, has been of little predictive value in melanoma,
there is a time differential in the onset of contractions hepatocellular carcinoma, or osteosarcoma. Staging is
in the fundus, midzone and lower uterine segments. based on the extent of spread rather than histologic
Exciting stimulus starts in one cornu and then several appearance and is more relevant in predicting the
milliseconds later in the other. The exciting waves course of lung and colorectal cancers.
then join and sweep over the fundus and down the
uterus. 98. The answer is B
The resting intra-amniotic pressure is about 20 According to Friedman except for cervical dilatation
mmHg. In 1St stage the pressure is 40-60 mmHg and and foetal descent none of the clinical features of the
in the 2nd stage it is 80-100 mmHg. parturient appears to be useful in assessing labour
progression. The pattern of cervical dilatation during
96. The answer is c. normal labour has a sigmoid shape.
Most enterocutaneous fistulas result from trauma There are two phases of cervical dilatation, the latent
sustained during surgical procedures. Irradiated, phase and the active phase. The active phase is
obstructed, and inflamed intestine is prone to subdivided into acceleration phase, phase of
fistulization. Complications of fistulas include fluid maximum slope and the deceleration phase.
and electrolyte depletion, skin necrosis, and According to Williams ‘in the descent pattern of
malnutrition. Proximal small-bowel fistulas tend to normal labor, a typical hyperbolic curve is formed
produce a high output of intestinal fluid and are less when the station of the foetal head is plotted as a
likely to close with conservative management than are function of the duration of labour.
distal, low-output fistulas. Small-bowel fistulas that In the nulliparous increased rate of descent of foetal
communicate with other organs, particularly the ureter head is observed during the phase of maximum slope
and bladder, may need aggressive surgical repair of cervical dilatation.
because of the risk of associated infections. The
presence of obstruction distal to the fistula (e.g., an 99. The answer is c.
anastomotic stricture) can be diag nosed by barium Risk factors for breast cancer include family history
contrast study and mandates correction of the nulliparity, previous breast cancer, early menarche,
obstruction. When these poor prognostic factors for and late menopause. A late age at first birth (after age
stabilization and spontaneous closure are observed, 30) doubles the risk of breast cancer compared with
early surgical intervention must be undertaken. The early parity (age 18 or earlier). Having one first-
patient in the question, however, appears to have a degree relative (mother, sister, or daughter) with
low-output, distal enterocutaneous fistula. Control of breast cancer also doubles the risk. Women of the
the fistulous drainage should be provided by upper social classes, as measured by either education
percutaneous intubation of the tract with a soft or income, have been found to have the highest
catheter. This is usually accomplished under incidence of breast cancer. Neither cigarette smoking
fluoroscopic guidance. Antispasmodic drugs have not nor the use of hair d has been correlated with breast
been proved effective; somatostatin has been used cancer.
with mixed success in the setting of high-output
(greater than 500 mL/day) fistulas. There is no 100. The answer is B
indication for antibiotics in the absence of sepsis. It is also known as “Schnables’ atrophy”.
Total parenteral nutrition (TPN) is given to maintain Features
or restore the patient’s nutational balance while 1. Atrophy of nerve fibres.

DBMCI, Ph: 9810150067, 9868387223, E-mail: drbhatiainstitute@rediffmail.com, Visit us at: www.dbmci.com 13


Grand Test – 36 Explanatory Answer

2. No glial cell proliferation. 104. The answer is c.


3. Mucoid degeneration of glial cells leading to Patients with achalasia typically present with distal
formation of lacunae or clear pools. esophageal obstruction, which leads to regurgitation
Occurs in of saliva and undigested food. The characteristic
1. Glaucoma. appearance of the esophagram is the tapered bird’s-
2. Methyl alcohol poisoning. beak deformity at the level of the esophagogastric
3. High myopia. junction. Chest pain may be seen in the early stages of
the disease. Manometry yields high resting pressures
101. The answer is c. of the lower esophageal sphincter, which fails to relax
Primary hyperparathyroidism is a common disease, or only partially relaxes. The absence of pen staltic
usually in women. Essential to the diagnosis of deglutitory contractions in the body of the esophagus
hyperparathyroidism is the finding of hypercalcemia. is also noted during manometry. Although both
Though there are many causes of hypercalcemia, surgical intervention and forceful dilatation have been
hyperparathyroidism is by far the most prevalent. used to treat this disease, surgery results in
With rare exceptions, operations for primary improvement in over 90% of patients, compared with
hyperparathyroidism should not be performed unless only 70% of patients treated by forceful dilatation.
the patient is hypercalcemic. Parathyroid hormone Surgical treatment is an esophagomyotomy.
(PTH) is not invariably elevated, but it should be
elevated relative to the serum calcium level. 105. The answer is D
Ordinarily, high serum calcium levels suppress This is a case of IUD. The cause of death is asphyxia
parathyroid secretion. Therefore, in the presence of because of prolonged labour.
hypercalcemia, normal levels of PTH are actually In this case the height of mother is only 142 cm,
abnormal. Patients with primary hyperparathyroidism which may lead to cephalopelvic disproportion.
have either normal or elevated urinary calcium. As the Besides this cervix is fully dilated but the head is still
name suggests, patients with familial hypocalciuric 2 cm above the ischial spine. This means the descent
hypercalcemia (FHH) have hypercalcemia. They also of foetal head is not adequate. 75 mg of pethidine
usually have elevated PTH, but surgery is not injection will not cause foetal death. There is no clue
indicated in this relatively rare setting of in the question to suggest placental separation or
hypercalcemia. rupture of membranes.

102. The answer is C 106. The answer is B


The second and the third statements are correct. In this case the best treatment is craniotomy.
Effacement of cervix occurs from above downwards. Craniotomy is a destructive operation in which a
The muscle fibres at the level of internal os are pulled perforation is made on foetal head to evacuate the
upwards or taken up into the lower uterine segment. contents followed by extraction of the foetus.
The condition of the external os remains unchanged Besides IUD with obstructed labour it can also be
temporarily. performed in severe hydrocephalus in a live foetus.
1. In multipara the descent of foetal head may be very Craniotomy is contraindicated when true conjugate is
rapid. less than 7.5 cm.
2. Full dilation of cervix means 10 cm dilation.
3. Often appreciable effacement of ripened cervix 107. The answer is a.
occurs before active labour begins. Omphalocele and gastroschisis result in evisceration
of bowel and require emergency surgical treatment to
103. The answer is d. effect immediate or staged reduction and abdominal
Radiation-induced thyroid cancer was first recognized wall clo sure. Patent urachal or omphalomesenteric
in 1950 by Duffy and Fitzgerald. It usually follows ducts result from incomplete clo sure of embryonic
low-dose external radiation. Most cancers occur after connections from the bladder and ileum, respectively,
exposure to 1500 rads or less to the neck, but an to the abdominal wall. They are appropriately treated
increase in thyroid cancer has been noted after as little by excision of the tracts and closure of the bladder or
as 6 rads. Salivary gland tumors and possibly ileum. In most children, umbilical hernias close
parathyroid adenomas are also associated with spontaneously by the age of 4 and need not be
radiation. The latent period for these tumors is 30 repaired unless incarceration or marbled enlargement
years or longer. Of all patients exposed to low-dose and distortion of the umbilicus occurs.
radiation, about 9% have been found to have thyroid
cancer, usually of the papillary type. 108. The answer is A
Campimetry is evaluation of central visual field
(Bjerrums screen, automated).

DBMCI, Ph: 9810150067, 9868387223, E-mail: drbhatiainstitute@rediffmail.com, Visit us at: www.dbmci.com 14


Grand Test – 36 Explanatory Answer

109. The answer is b. Patients who have a lower motor neuron lesion
The effects of radiation on the intestine depend on a (flaccid neurogenic bladder) can usually be managed
variety of factors, which include the age of the by conservative measures that prevent the
patient, temperature, degree of oxygenation, and development of a large residual urine volume in the
metabolic activity Acute intestinal radiation injury is bladder. These measures include intermittent self-
manifested in the bowel by the cessation of viable cell catheterization and scheduled voiding with increased
production and is seen clinically as diarrhea or abdominal pressure provided by the Valsalva
gastrointestinal bleeding. Progressive vasculitis and maneuver or manual pressure on the abdomen.
fibrosis are seen in the latter stages of radiation injury Detrusor contractions can sometimes be strengthened
and may result in malabsorption, ulceration, by parasympathomimetic agents such as bethanechol
fistulization, or perforation. Intussusception is chloride (Urecholine). Bladder augmentation to
generally not associated with radiation injury. increase capacitance, bladder neck resection to reduce
outlet obstruction, and supravesicale ureteral
110. The answer is d. diversion are indicated only in the presence of
The boundaries of the mediastinum are the thoracic deterioration of bladder compliance or gross
inlet, the diaphragm, the sternum, the vertebral ureterocalyxectasis that resists the foregoing measures
column, and the pleura bilaterally The mediastinum and threatens the loss of renal function or debilitating
itself is divided into three portions delineated by the urinary incontinence. Severely restrict ing fluid intake
pericardial sac: the anterosuperior and posterosuperior is impractical and may promote formation of calculi.
regions are in front of and behind the sac, respectively
while the middle region designates the contents of the 114. The answer is d.
pericardium. Mediastinal masses occur most The CT findings are consistent with any of the
frequently in the anterosuperior region (54%) and less suggested lesions. However, the most likely diagnosis
often in the posterosuperior (26%) and middle (20%) is metastatic disease. Almost 50% of intracranial
regions. Cysts (pericardial, bronchogenic, or enteric) neoplasms are metastatic lesions. Roughly 20 to 25%
are the most common tumors of the middle region; of cancer patients develop intracranial metastases
neurogenic tumors are the most common (40%) of the during the course of their disease. Cancers of the lung
primary tumors of the posterior mediastinum. The and breast and melanomas frequently metastasize to
primary neoplasms of the mediastinum in the the brain parenchyma. Leukemia shows a predilection
anteroposterior region are thymomas (3 1%), for the leptomeninges. A large majority of these
lymphomas (23%), and germ cell tumors (17%). More lesions become symptomatic owing to mass effect
commonly though, a mass in this area represents the from white matter edema.
substernal extension of a benign substernal goiter.
Diagnosis may be made by visualization of an 115. The answer is C
enhancing structure on CT; radioactive iodine Diarrhoea is defined as passage of liquid or watery
scanning is useful in management because it may stool more than 3 times per day. Recent change in
make the diagnosis if the mediastinal tissue is consistency of stool is more important.
functional and will also document the presence of Persistent diarrhoea is defined as diarrhoea of acute
functioning cervical thyroid tissue to prevent removal infectious aetiology that begins acutely and lasts for
of all functional thyroid tissue during mediastinal more than 14 days.
excision. Dysentery is characterized by presence of blood and
pus in stool.
111. The answer is B
Vertigo is not seen in Otosclerosis 116. The answer is A
The mucopolysaccharidoses (MPSs) result from
112. The answer is A deficiencies of specific enzymes involved in the
Pulmonary stenosis is seen in 7-10% cases of breakdown of glycosaminoglycans (GAGs), which are
congenital heart disease. It is the commonest cause of also called MPSs. The seven major types of GAGs are
dilation of pulmonary artery due to post stenotic hyaluronic acid, chondroitin sulfate, keratin sulfate,
dilation. dermatan sulfate, heparan sulfate, and heparin. The
Although VSD is one of the commonest congenital MPSs are charactenzed by accumulation of partially
heart disease, pulmonary artery dilatation is seen only degraded GAGs in multiple organs, including the
when it is associated with other congenital heart liver, spleen, heart, and blood vessels. Accumulations
lesions. of GAGs within leukocytes produce Alder-Reilly
bodies, while accumulations within neurons can
113. The answer is b. produce zebra bodies. The MPSs are also
characterized by the excretion of excess acid

DBMCI, Ph: 9810150067, 9868387223, E-mail: drbhatiainstitute@rediffmail.com, Visit us at: www.dbmci.com 15


Grand Test – 36 Explanatory Answer

mucopolysaccharides in the urine, a finding that helps 119. The answer is c.


to differentiate the MPSs from the mucolipidoses. The combination of trouble swallowing, hypertension,
Most of the MPSs are associated with coarse facial and sclerosis of the skin should raise the possibility of
features, clouding of corneas, joint stiffness, and progressive systemic sclerosis (scleroderma), a
mental retardation. The characteristic appearance of multisystem disease that involves the cardiovascular,
patients with type IH MPS (Hurler’s syndrome), gastrointestinal, cutaneous, musculoskeletal,
which results from a deficiency of alpha- has been pulmonary, and renal systems through progressive
described as “gargoylism.” These patients excrete interstitial fibrosis. Small arterioles in the
excess dermatan sulfate and heparan sulfate, both of aforementioned systems show obliteration caused by
which are mucopolysaccharides, in the urine. Type 11 intimal hyperplasia accompanied by progressive
MPS (Hunter’s syndrome) is the only MPS that has an interstitial fibrosis. In the skin, the changes begin in
X-linked recessive type of inheritance. These patients the fingers and hands and consist of sclerotic atrophy,
have a much milder disease than Hurler’s syn drome which is characterized by increased dermal collagen,
patients, but they also secrete dermatan sulfate and epidermal atrophy, and loss of skin adnexal structures.
heparan sulfate in the urine. Type IV MPS, known as Two antinuclear antibodies are unique to systemic
Morquio’s syndrome, is characterized by short stature, sclerosis. One is Scl-70, which is found in diffuse
aortic valvular disease, and normal intelligence. These progressive systemic sclerosis, and the other is an
patients are prone to development of subluxation of anticentromere antibody found in the CREST
the spine, which can produce quadriplegia. They syndrome, a variant of progressive systemic sclerosis.
secrete keratan sulfate in the urine.
120. The answer is D
117. The answer is d. Pathological myopia denotes myopia with fundus
The names given to tumors are based on the changes. Pathological myopia is progressive
parenchymal component of the tumor, which consists throughout in contrast to other forms of myopia which
of the proliferating neoplastic cells. In general, benign become stable at age of 20 to 22 years. And since it is
tumors are designated by using the suffix-oma progressive; it may finally amount to 20D to 25D.
attached to a name describing either the cell of origin
of the tumor or the gross or microscopic appearance 121. The answer is d.
of the tumor. Examples of benign tumors whose Nocardia (N. asteroides) and Actinomyces species are
names are based on their microscopic appearance classified as filamentous soil bacteria, although they
include adenomas, which have a uniform proliferation are often descanted among the fungi. A. israelii is a
of glandular epithelial cells; papillomas, which are normal inhabitant of the mouth; it can be seen in the
tumors that form finger-like projections; fibromas, crypts of tonsillectomy specimens. Actinomyces is a
which are composed of a uniform proliferation of branched, filamentous gram-positive bacteria. Two
fibrous tissue; leiomyomas, which originate from forms of disease produced by Actinomyces are
smooth muscle cells and have elongated, spindle- cervicofacial actinomyces and pelvic actinomyces.
shaped nuclei; hemangiomas, which are formed from The former consists of an indurated (lumpy) jaw with
a uniform proliferation of endothelial cells; and multiple draining fistulas or abscesses. Small yellow
lipomas, which originate from adipocytes. colonies called sulfur granules may be seen in the
draining material. Histologic section reveals tangled
118. The answer is D masses of gram-positive filamentous bacteria.
The commonest cause of severe obstructive uropathy Cultures of Actinomyces grow as white masses with a
in male infant is posterior urethral valves. This lesion domed surface, which is called a “molar tooth”
occurs in approximately I in 8,000 boys. Posterior appearance. Another filamentous gram-positive
uethral valve refers to tissue leaflets fanning distally bacteria is N. asteroides. A characteristic that helps to
from the prostatic urethra to the external urinary differentiate these two is the fact that Nocardia is
sphincter. partially acid-fast. Partial’ means using weak mineral
Approximately 30% of patients experience end-stage acids in the acid- last stain. Nocardia are aerobic and
renal disease or chronic renal insufficiency. The acid-fast, in contrast to Actinomyces species, which
prostatic urethra dilates and the bladder muscle are strict anaerobes and not acid-fast. Inhaled
undergoes hypertrophy. Vesicoureteral reflux occurs nocardial bacteria produce lung or skin infections.
in 50% of patients. Affected boys with posterior Progressive pneumonia with purulent and abscesses is
urethral valves are discovered prenatally when suggestive of nocardiosis, especially if dissemination
maternal ultrasonography reveals bilateral to the brain or subcutaneous tissue occurs. Nocardia is
hydronephrosis with a distended bladder. also one cause of mycetoma, a form of chronic
inflammation of the skin that causes indurated
abscesses with multiple draining sinuses. Patients who

DBMCI, Ph: 9810150067, 9868387223, E-mail: drbhatiainstitute@rediffmail.com, Visit us at: www.dbmci.com 16


Grand Test – 36 Explanatory Answer

develop nocardiosis are often immunosuppressed, and alveoli and also for the formation of hyaline
transplant rejection, steroid therapy, AIDS, or alveolar membranes. The most reliable test to determine
proteinosis are often antecedent. Organisms in pulmonary maturity is the ratio of lecithin to
sputum, pus, or bronchial lavage specimens are gram- sphingomyelin (U’S), both of which are
positive. A modified acid-fast stain should be used for phospholipids.
diagnosis.
124. The answer is b.
122. The answer is B Benign tumors of vessels may originate from either
UTI usually accompanies urinary incontinence and blood vessels or lymphatics. Lymphangiomas arc
incontinence persists even after treatment of UTI. tumors that are derived from lymphatic vessels. Histo
Cystitis is mentioned as a cause of incontinence of logically they reveal dilated vessels lined by
urine. Myelodyspiasia and meningomyelocle both endothelial cells, but they lack RBC in their lumen.
cause neurogenic bladder that lead to urinary The absence of red blood cells helps to distinguish
incontinence. Ectopic ureter is also a well defined these lesions from hemangiomas. Cystic hygromas are
cause of urinary incontinence. cystic lymphangiomas that typically occur in the neck
Chronic UTI is probably also a cause of urinary or axilla. They may grow to such a large size that the
continence but it is not clearly mentioned as a neck is deformed. These lesions may he found in
definitive cause. patients with Turner’s syndrome, an abnormality that
Daytime incontinence not secondary to neurologic results from complete or partial monosomy for the X
abnormalities is a common problem in children. At chromosome. Swelling of the neck in these
age 5 years, 95% have been dry during the day at individuals occurs because of dilated lymphatic
some time and 92% are dry. At 7 year, 96% are dry, vessels. With time the swelling decreases, hut patients
although 15% have significant urgency at times. At 12 may develop bilateral neck webbing and loose skin on
year, 99% are dry during the day. The most common the hack of the neck.
cause of daytime incontinence is a pediatric unstable
bladder (also termed uninhibited bladder, bladder 125. The answer is D
spasms). The child has developed acute leukemia.
Fever and anaemia are important features of leukemia.
123. The answer is d. Petechial hemorrhage occurs due to low platelet count
Hyaline membrane disease (HMD), which accounts that occurs in acute leukemia.
for 20% of all deaths in the first 28 days of life, is Lymphadenopathy is a common finding in acute
basically a disease of premature infants; most affected lymphoblastic leukemia.
infants weigh 1000 to 1500 g. Contributing factors in Bone tenderness is a very common finding in a case
the development of HMD include diabetes in the of leukemia. This feature is usually absent in rest of
mother (maternal diabetes with increased glucose the three conditions and this feature is the main clue
causes increased fetal secretion of insulin, which in this question.
inhibits the effects of steroids such as lung maturation Hepatosplenomegaly is also a very common finding
and production of surfactant) and cesarean sec tion. in acute leukemia. When all these facts are considered
Infants who develop HMD appear normal at birth, but together the diagnosis is nothing but acute leukemia.
within minutes to hours their respirations become
labored. Grossly the lungs are a mottled, red-purple 126. The answer is A.
color, while microscopically there are hyaline As the endocardial cushions join to divide the
membranes in air spaces, similar to those of ARDS. ventricles, the streams of blood from the right and left
Two defects have been identified in infants with ventricles are divided as they flow out of the truncus.
HMD. One is a deficiency of pulmonary surfactant. A spiral septum develops to physically separate the
Surfactant, a lipid consisting of dipalmitoyl two streams of blood. The fusion of the spiral ridges
phosphatidylcholine, reduces the surface tension in results iii division of the truncus into the pulmonary
air-fluid interfaces by getting between the molecules arid aortic arteries. Occasionally the spiral is reversed,
in the liquid and reducing their attraction to each resulting in the aorta arising from the right ventricle
other. This reduces the tendency for the alveoli to and the pulmonary artery from the left. This is a
collapse after birth on expiration. Synthesis of complete transposition of the great vessels and
surfactant increases throughout fetal development, but produces two completely separate blood systems. This
becomes maximal at 34 to 36 weeks. With a situation obviously is incompatible with life unless
deficiency of surfactant, the lungs tend to collapse on some type of mixing of blood can occur between
expiration (atelectasis) and become stiff. The other these separate systems. In utero, mixing of blood
defect is increased pulmonary epithelial permeability. occurs across the atrial septum and in connections
This accounts for the protein-rich edema fluid in the with the placental circulation. Cases that survive to

DBMCI, Ph: 9810150067, 9868387223, E-mail: drbhatiainstitute@rediffmail.com, Visit us at: www.dbmci.com 17


Grand Test – 36 Explanatory Answer

corrective surgery must have a persistent atrial septal contaminated ground meat, by verocytotoxin-
defect or patent ductus arteriosus to allow mixing of producing Escherichia coil. This toxin is similar to
blood. Therefore, clinical consideration should be to Shigella toxins, which, together with E. coli and
keep the ductus arteriosus open. Usually, at birth, viruses, are causes of the adult form of HUS. Multiple
breathing decreases pulmonary resistance and this microthrombi in glomerular capillaries result in renal
then reverses flow through the ductus arteriosus. This failure, while systemically the micro thrombi cause
oxygenated blood (flowing from the aorta into the microangiopathic hemolytic anemia. Subsequently
ductus) inhibits prostaglandin production, which in thrombocytopenia develops and leads to a bleeding
turn closes the ductus arteriosus. To keep the ductus diathesis, seen as vomiting of blood and hematuria.
arteriosus open, prostaglandin E should be given.
130. The answer is D
127. The answer is b. Polycythemia refers to an increased concentration of
Paroxysmal nocturnal hemoglobinuria (PNH) is an RBCs in the peripheral blood. This is manifested by
acquired clonal stem cell disorder that is characterized an increase in RBCs count, hemoglobin concentration,
by abnormal red cells, granulocytes, and platelets. The or hematocrit. An increase in red blood cell count,
red blood cells (RBCs) are abnormally sensitive to the reported clinically as number of cells per microliter, is
lytic activity of complement due to a deficiency of not the same thing as the RBC mass, which is a
glycosyl phosphatidyl inositol (GPO-linked proteins, radioactive test that is reported in ml/kg. The RBC
namely decay-accelerating factor (DAF, or CD 55), count and the RBC mass do not always parallel each
membrane inhibitor of reactive lysis (CD55), or CD59 other. For example, a decreased plasma volume
(a C8 binding protein). Complement is normally increases the RBC count but does not affect the RBC
activated by acidotic states, such as occur with mass. An increased red blood cell concentration may
exercise or sleep. In patients with PNH, the acidotic be a relative polycythemia or an absolute
condition that develops during sleep (which is usually polycythemia. A relative polycythemia is due to a
at night) causes hemolysis of red blood cells and decrease in the plasma volume (hemoconcentration),
results in red urine in the morning. The erythrocytes causes of which include prolonged vomiting, diarrhea,
of these patients lyse in vitro with acid (Ham’s test) or or the excessive use of diuretics. An absolute
sucrose (sucrose lysis test). Complications of PNH polycythemia is due to an increase in the total red cell
include the development of frequent thromboses, mass and may be primary, due to a defect in myeloid
particularly of the hepatic, portal, or cerebral veins. stem cells (polycythemia rubra vera), or secondary,
Since PNH is a clonal stem cell disorder, patients are due to an increase in the production of erythropoietin
at an increased risk of developing aplastic anemia or (EPO). In patients with primary polycythemia, a
acute leukemia. myeloproliferative disorder, the red cell mass is
increased but the levels of FF0 are normal or
128. The answer is A decreased in patients with secondary polycythemia,
Indian childhood cirrhosis occurs due to excessive the increased EPO may he appropriate or
copper deposition in liver. It is commonly seen in inappropriate. Appropriate causes of increased EPO,
children who drank water boiled in corroded in which the oxygen saturation of hemoglobin will be
copper/brass vessels for a long time. In these patients abnormal, include lung disease, cyanotic heart
serum copper is increased but not to a great extent like disease, living at high altitudes, or abnormal
that seen in Wilson’s disease. Besides this serum zinc hemoglobins with increased oxygen affinity
level is low and these patients suffer from features of Inappropriate causes of increased FF0 include EPO-
zinc deficiency. Zinc supplementation is strongly secreting tumors, such as renal cell carcinomas,
recommended in these patients. In ICC serum albumin hepatomas, or cerebellar hemangioblastomas.
is low; IgA, IgG and 1gM are raised; smooth muscle
antibody is present in up to 45% cases; C3 and C1q 131. The answer is C
are decreased. Cataract is seen in galactosemia.
Galactose-1 -phosphate uridyl transferase deficiency
129. The answer is c. causes galactosemia.
Hemolytic-uremic syndrome (HUS) is similar to In classic galactosemia, cataract, hepatomegaly,
thrombotic thrombocytopenic purpura (TIP) in that it malabsorption, neonatal sepsis, and mental deficiency
produces a microangiopathic hemolytic anemia, but it are all related to deficiency of the transferase enzyme,
is distinguished from TTP by the lack of neurologic which is the primary effect of the underlying
symptoms and by the severe acute renal failure, which autosomal recessive mutant gene.
is manifested clinically by a markedly increased
serum BUN. Classic HUS is seen in children and is 132. The answer is A.
related to infection, usually acquired from

DBMCI, Ph: 9810150067, 9868387223, E-mail: drbhatiainstitute@rediffmail.com, Visit us at: www.dbmci.com 18


Grand Test – 36 Explanatory Answer

Lymph nodes may be enlarged (lymphadenopathy) (‘potato nodes”) or lung involvement is present and
secondary to reactive processes, which can be either can be revealed by chest x-ray or transbronchial
acute or chronic. Acute reaction (acute non specific biopsy The eye and skin are the next most commonly
lymphadenitis) can result in focal or generalized affected organs, so that both conjunctival and skin
lymphadenopathy Focal lymph node enlargement is biopsies are clinical possibilities. Noncaseating
usually the result of bacterial infection (bacterial granulomas may be found in multiple infectious
lymphadenitis). Sections from involved lymph nodes diseases, such as fungal infections, but sarcoidosis is
reveal infiltration by neutrophils. In contrast, not caused by any known organism. Therefore, before
generalized acute lymphadenopathy is usually the the diagnosis of sarcoidosis can be made, cultures
result of viral infections and usually produces a must be taken from affected tissues, and there must be
proliferation of reactive T lymphocytes called I no growth of any organism that may produce
immunoblasts. These reactive T cells tend to have granulomas. In patients with sarcoidosis, blood levels
prominent nucleoli and can be easily mistaken for of angiotensin-converting enzyme are increased, and
malignant lymphocytes or malignant Hodgkin cells. this may also be used as a clinical test. In the past, the
Kveim skin test was used to assist in the diagnosis of
133. The answer is b. sarcoidosis.
Pulmonary edema refers to excess accumulation of
fluid in the extravascular spaces of the lung. 136. The answer is A.
Pulmonary edema can be classified based on the Two cysts that occur in the neck are the branchial
etiology into cardiogenic pulmonary edema and cleft cyst (usually located in the anterolateral part of
noncardiogenic pulmonary edema. Cardio genie the neck) and the thyroglossal duct cyst (usually
pulmonary edema results from abnormalities of located in the anterior part of the neck). Each of these
hemodynamic (Starling) forces, while noncardiogenic cysts may histologically reveal a lining composed of
pulmonary edema results from cellular injury. Causes squamous epithelium or pseudostratified columnar
of cardiogenic pulmonary edema include increased epithelium. Branchial cleft cysts, which arise from
hydrostatic forces, as seen with congestive heart remnants of the branchial (pharyngeal) apparatus,
failure (the most common cause of pulmonary may contain lymphoid tissue, while thyroglossal duct
edema); decreased oncotic pressure, such as resulting cysts may move up and down as the patient swallows.
from decreased albumin levels; and lymphatic There are several other types of cysts that occur in and
obstruction. Noncardiogenic edema may be the result around the oral cavity. The most common type is the
of either endothelial injury (infections, dis seminated radicular cyst. These cysts result from chronic
intravascular coagulopathy, or trauma) or alveolar inflammation of the tooth apex. Histologic sections
injury (from inhaled toxins, aspiration, drowning, or reveal chronic inflammation of the tooth apex with
near drowning). epithelialization of periapical granula tion tissue.

134. The answer is A 137. The answer is B


Toxic epidermal neorolysis (TEN) is often caused by Cephalohematoma is a subperiosteal hemorrhage,
drugs and ET producing strains of S. aureus. The most hence it is always limited to the surface of one cranial
severe firm of staphylococcal soalded skin syndrome bone. It does not cross suture line.
is termed TEN in older children. Person > 5 years old There is no discoloration of the overlying scalp, and
rarely develop staphylococcal TEN. swelling is usually not visible until several hours after
TE often begins with erythematous rash. Pastia lines birth, because subperiosteal bleeding is a slow
may be apparent Skin has sandpaper texture and often process. An underlying skull fracture, usually linear
tender. Within hours to days wrinkling nd sloughing and not depressed, is occasionally associated with
of epidermis begins and sloughing can be provoked cephalohematoma.
by gentle stroking of skin (Nikolsky’s sign) even in Cranial meningocele may be differentiated from
apparently uninvolved areas. Total detachment of> cephalohematoma by pulsation, increased pressure on
30% of body surface area occurs in TEN. Onset is crying, and the X-ray evidence of bony defect.
usually acute with fever of >39oF. About 30% of Most cephalohematomas are resorbed within 2 weeks-
affected persons die. 3 months, depending on their size. They may begin to
calcify by the end of the 2nd week.
135. The answer is d.
Sarcoidosis is a systemic disease characterized by 138. The answer is c.
noncaseating granulomas in multiple organs. The Most lesions of the esophagus present with similar
diagnosis of sarcoidosis depends on finding these symptoms, such as heartburn and dysphagia, but the
noncaseating granulomas in commonly affected sites. most serious disease, which carries the risk of
In 90% of cases, bilateral hilar lymphadenopathy exsanguination, is bleeding esophageal varices.

DBMCI, Ph: 9810150067, 9868387223, E-mail: drbhatiainstitute@rediffmail.com, Visit us at: www.dbmci.com 19


Grand Test – 36 Explanatory Answer

Varices occur in about two-thirds of all patients with capsule. It may be produced by hepatitis viruses
cirrhosis, and in the majority of patients the etiology (usually B or C), drugs, or chemicals.
is alcoholic cirrhosis. The cirrhosis causes portal
hypertension, which shunts blood into connecting 141. The answer is B
channels between the portal and caval systems, such Vitamin D resistant rickets is inherited as X-linked
as the subepithelial plexus of veins in the lower dominant condition. The main defect is in proximal
esophagus. Varices produce no symptoms until they tubular reabsorption of phosphate and in conversion
rupture and cause massive bleeding (hematemesis), of 25 (OH) D3 to 1,25 (OH)2 D3. The urinary
which may lead to death. Other diseases, such as phosphate level is increased. The main clinical
gastritis, esophageal laceration (Mallory- Weiss features are.
tears), or peptic ulcer disease, may cause 1. Short stature. 2. Waddling gait.
hematemesis. 3. Bow legs 4. Coxa vara.
In contrast, columnar epithelium in the distal 5. Genu valgum.
esophagus is seen with Barrett’s esophagus; decreased Serum calcium is normal or low, phosphorus is
ganglion cells in the myenteric plexus are seen with reduced and alkaline phosphatase is raised. Treatment
achalasia, a disorder that is characterized by is with oral phosphate (0.5-4 g/day) in divided doses
aperistalsis, incomplete relaxation of the lower along with vitamin D.
esophageal sphincter (LES) with swallowing, and
increased resting tone of the LES, all of which lead to 142. The answer is b.
esophageal dilation and symptoms of progressive Malignant hypertension is characterized clinically by
dysphagia. finding a diastolic blood pressure greater than 130
mmHg. Additional clinical signs and symptoms of
139. The answer is B malignant hypertension include severe headache, ear
Convergent squints lead to uncrossed diplopia and noises, flame-shaped retinal hemorrhages with AV-
divergent squints leads to crossed diplopia. nicking, and papilledema. The renal changes
associated with malignant hypertension are called
140. The answer is a. malignant nephrosclerosis. These characteristic
The type and distribution of necrotic hepatocytes is changes include fibrinoid necrosis of arterioles
often a clue as to the cause of the hepatic injury. Focal (necrotizing arteriolitis), hyperplastic
scattered necrosis is characteristic of viral hepatitis, arteriolosclerosis (onion skinning), necrotizing
but may also be seen with bacterial infections or other glomerulitis, and often a thrombotic microangiopathy.
toxic insults. In focal necrosis, there is necrosis of
single hepatocytes, or small clusters of hepatocytes, 143. The answer is d.
that is randomly located in some, but not all, of the Benign prostatic hyperplasia produces clinical
liver lobules. In contrast, zonal necrosis refers to the symptoms of urinary frequency, nocturia, difficulty in
finding of hepatocellular necrosis in identical areas in starting and stopping urination, dribbling, and dysuria.
all of the liver lobules. There are basically three types Histologically the hyperplastic nodules are composed
of zonal necro sis. Centrilobular (acinar zone 3) of a variable mixture of hyperplastic glands and
necrosis is characteristic of ischemic injury (heart hyperplastic stromal cells. Histologic signs of
failure or shock), toxic effects (acetaminophen malignancy are not present. The development of BPH
toxicity), carbon tetra chloride exposure, or is associated with increased age and higher
chloroform ingestion. Drugs such as acetaminophen testosterone levels. BPH results from androgen
may be metabolized in zone 1 to toxic compounds induced glandular proliferation, but estrogen also
that cause necrosis of zone 3 hepatocytes because they sensitizes the tissue to androgens. Urinary obstruction
receive the blood from zone 1. Midzonal (zone 2) results because the inner, periurethral por tions of the
necrosis is quite rare, but may be seen in yellow fever, prostate (the middle and lateral lobes) are affected
while pen- portal (zone 1) necrosis is seen in most commonly. BPH does not predispose the
phosphorus poisoning or eclampsia. Sub- massive individual to cancer. In contrast to the benign
necrosis refers to liver cell necrosis that crosses the histology of BPH, the histologic signs characteristic
normal lobular boundaries. Classically the necrosis of prostatic adenocarcinoma include small glands that
goes from portal areas to central veins (or vice versa) appear “back to back” without intervening stroma or
and is called bridging necrosis, If the hepatocellular that appear to be infiltrating beyond the normal
necrosis is severe, it is called massive necrosis. This prostate lobules.
type of extensive necrosis is described as acute yellow
atrophy, because grossly the liver appears soft, 144. The answer is D
yellow, flabby, and decreased in size with a wrinkled The child is suffering from retropharyngeal abscess.

DBMCI, Ph: 9810150067, 9868387223, E-mail: drbhatiainstitute@rediffmail.com, Visit us at: www.dbmci.com 20


Grand Test – 36 Explanatory Answer

Swelling in neck is seen only in retropharyngeal Ideally all samples taken from the distribution system
abscess. It is not seen in the other three conditions. should be free from coliform organisms. In practice
Retropharyngeal abscess is collection of pus in this standard is not always attainable and the
retropharyngeal space and is a potential complication following standard for water collected in the
of bacterial pharyngitis. distribution system is therefore recommended:
The child presents with fever stridor with features of 1. Throughout any year, 95% of samples should not
toxaemia. X-ray neck lateral view shows soft tissue contain any coliform organism in 100 ml.
bulge in posterior pharyngeal wall. Treatment is 2. No sample should contain E. coil in 100 ml.
surgical drainage. 3. No sample should contain more than 3 coliform
The main presentation of oesophagitis is retrosternal organisms per 100 ml.
burning with dysphagia. 4. Coliform organisms should not be detectable in 100
In achalasia the main clinical picture is dysphagia, ml of any two consecutive samples. Considering the
which is more for liquids than solid. There are several above facts the answer is choice D.
types of hiatus hernia and the main problem is reflux
of stomach contents into oesophagus that leads to 148. The answer is d.
reflux oesophagitis with heart burn, dysphagia etc. In the adrenal cortex, cholesterol is converted into
either mineralocorticoids (aldosterone in the zona
145. The answer is b. glomerulosa glucocorticoid (cortisol) in the zona
Gestational trophoblastic diseases include benign fasciculata, or sex steroid precursors in the zona
hydatidiform mole (partial and complete), invasive reticularis Congenital adrenal hyperplasia (CAH) is a
mole (chorioadenoma destruens), placental site syndrome that results from a defect in the synthesis of
trophoblastic tumor, and choriocarcinoma. cortisol This leads to excess ACTH secretion by the
Hydatidiform moles are composed of avascular, anterior Pituitary and resultant adrenal hyperplasia.
grapelike structures that do not invade the The defect in the synthesis of cortisol is the result of a
myometrium. In complete (classic) moles, all the deficiency in one of the enzymes in the normal
chorionic villi are abnormal and fetal parts are not pathway of cortisol synthesis, such as 2 or il-
found. They have a 46,XX diploid pattern and arise hydroxylase. Most cases of CAH result from a
from the paternal chromosomes of a single sperm by a deficiency of 2 Two forms 01 this deficiency include
process called androgenesis. In partial moles, only salt-wasting adrenogenitalism and simple virilizing
some of the villi are abnormal and fetal parts may be adrenogenitalism. The salt-wasting syndrome results
seen. These moles have a triploid or a tetraploid from a complete lack of the hydroxylase. There is no
karyotype and arise from the fertilization of a single synthesis of mineralocorticoids or glucocorticoids in
egg by two sperm. About 2% of complete moles may the adrenal cortex. Decreased mineralocorticoids
develop into choriocarcinoma, but partial moles are cause p sodium loss in the urine, hyponatremia,
rarely followed by malignancy Invasive moles hyperkalemia, acidosis, and hypotension.
penetrate the myometrium and may even embolize to Because of the enzyme block there is increased
distant sites. formation of 17-hydroxyprogesterone, which is then
shunted into the production of testosterone. This may
146. The answer is a. cause virilism (pseudohermaphroditism) in female
Fibrocystic change of the breast is one of the most infants. That is, XX females with CAH develop
common features seen in the female breast. It is most ovaries, female ductal structure, and external male
likely associated with an endocrine imbalance that genitalia. Much more often there is only a partial
causes an abnormality of the normal monthly cyclic deficiency of 21-hydroxylase, which leads to
events within the breast. These fibrocystic changes are decreased production of both aldosterone and cortisol.
subdivided into nonproliferative and proliferative The decreased cortisol levels cause increased pro
changes. Nonproliferative changes include fibrosis of duction of ACTH by the pituitary, which results in
the stroma and cystic dilation of the terminal ducts, adrenal hyperplasia, enough to maintain adequate
which when large may form blue-domed cysts. A serum levels of aldosterone and cortisol. In contrast to
common feature of the ducts in nonproliferative a complete deficiency of 21-hydroxylase, there is no
changes is apocrine metaplasia, which refers to sodium loss with a partial deficiency of 21-
epithelial cells with abundant eosinophilic cytoplasm hydroxylase. The excess stimulation by ACTH,
with apical snouts. Proliferative changes include however, leads to increased production of androgens,
epithelial hyperplasia of the ducts. which may cause virilism in female infants.

147. The answer is D 149. The answer is a.


Guidelines of WHO for water In distribution system: Actinic (solar) keratoses, found on sun-damaged skin,
microscopically show hyperkeratosis, parakeratosis,

DBMCI, Ph: 9810150067, 9868387223, E-mail: drbhatiainstitute@rediffmail.com, Visit us at: www.dbmci.com 21


Grand Test – 36 Explanatory Answer

atypia of the epidermal keratinocytes, and Transection of a peripheral nerve may result in the
degeneration of the elastic fibers in the dermis formation of a traumatic neuroma if the axonal
(referred to as solar elastosis). Clinically, actinic sprouts grow into scar tissue at the end of the
keratoses appear as irregular erythematous brown proximal stump. Sometimes peripheral nerves maybe
papules. When the atypia of the intraepidermal compressed (entrapment neuropathy) due to repeated
keratinocytes is extreme (full thickness), the lesion is trauma. Carpal tunnel syndrome is the most common
referred to as Bowen’s disease (carcinoma in situ). entrapment neuropathy and results from compression
of the medial nerve within the wrist by the transverse
150. The answer is A carpal ligament. Symptoms include numbness and
CO2, SO2 and chlorofluorocarbon (CFC) cause green paresthesias of the tips of the thumb and second and
house effect as well as depletion of ozone layer. third digits. Another type of compression neuropathy
In the atmosphere of earth there are several gases. is associated with a painful swelling of the plantar
During the day time the earth absorbs sunrays and digital nerve between the second and third or the third
radiates heat in the form of infrared rays. If the gases and fourth metatarsal bones. This lesion, called a
in the atmosphere increase they will trap the infrared Morton’s neuroma, is most often found in females.
rays and increase the temperature. This leads to global
warming and its harmful effects. 154. The answer is A
Best Indicators of air pollution are:
151. The answer is c. • Sulphur dioxide.
Pannus is the name given to describe the classic • Smoke,
destructive joint lesion found in individuals with • Suspended particles.
rheumatoid arthritis. This lesion is characterized by
proliferation of the s (hyperplasia) along with 155. The answer is d.
numerous chronic inflammatory cells. The thickened Progressive multifocal leukoencephalopathy (PML) is
synovial membrane may develop villous projections, a viral infection of oligodendrocytes that results in
which can destroy the joint cartilage. Nodular demyelination of the central nervous system. Signs
collections of lymphocytes resembling follicles are and symptoms of PML are varied but include
characteristically seen along with numerous plasma dementia and ataxia along with abnormal vision and
cells Palisades of proliferating cells may surround speech. The causative agents of PML are two closely
areas of necrosis. This latter histologic appearance can related polyomaviruses, JC virus and SV40. The
be seen in subcutaneous nodules (rheumatoid pathognomonic feature of PML is the
nodules), but rheumatoid arthritis most frequently oligodendrocytes in areas of demyelination, which
affects the small joints of the hands and feet. Larger have a “ground-glass” appearance of their nuclei due
joints are involved later. to infection with the viral particles. PML occurs as a
terminal complication in immuno suppressed
152. The answer is A individuals, especially individuals with AIDS. In
Rods and cones (photoreceptors) are the end organs of contrast to the viral inclusions of PML, beta-A4
vision which transform light energy into visual amyloid is found in the walls of medium to small
(nerve) impulse. arteries and arterioles in the brain and meninges of
RODS individuals with cerebral amyloid angiopathy (CAA),
• Contain photosensitive substance, visual purple a significant cause of intracerebral hemorrhage in
(rhodopsin) and subserve the peripheral vision and nonhypertensive elderly patients. Prion protein
vision of low illumination (scotopic vision). particles arc causes of the spongiform
• Rods are absent at the fovea in an area of 0.35 mm encephalopathies, while Rosenthal fibers within
(rod free zone) which corresponds to 1.25° of visual astrocytes are characteristic of juvenile pilocytic
field. But are present in a large number in a ring astrocytomas, and ubiquitin protein within neurons is
shaped zone 5-6 mm from the fovea. associated with Alzheimer’s disease.
CONES
• Cones also contain photosensitive substance and are 156. The answer is b
primarily responsible for highly discriminatory central An initial spontaneous abortion, irrespective of the
vision (photopic vision) and colour vision. karyotype of sex of the child, does not change the risk
• Highest density at fovea, and falls off rapidly outside of recurrence in a future pregnancy. The rate is
the fovea. commonly quoted as 15% of all known pregnancies.

153. The answer is d. 157. The answer is D


Peripheral nerve trauma may result in specific Cause of setback in National Malaria Eradication
symptoms and pathologic changes at specific sites. Programme after 1966:

DBMCI, Ph: 9810150067, 9868387223, E-mail: drbhatiainstitute@rediffmail.com, Visit us at: www.dbmci.com 22


Grand Test – 36 Explanatory Answer

Main causes were administrative and operational Free silica is still a major occupational hazard. Most
failure. often, progressive pulmonary fibrosis occurs in a
A. Technical failure: dose-response fashion after several years of exposure.
• Resistance of mosquito vector to insecticides. A radiographic picture of profuse miliary Infiltration
• Resistance of malaria parasites to drugs. Only partly or consolidation is characteristic of acute silicosis. In
responsible long-term, less intense exposure, small rounded
B. Administrative failure: opacities in the upper lobes, with retraction and hilar
• Shortage of insecticides and antimalarial drugs. lymphadenopathy, classically appear on the
• Rising cost of antimalarial operations. radiograph after 15 to 20 years.
• Inadequate budget provision by states. The nodular fibrosis may be progressive in the
• Difficulty in recruiting labour for spray work. absence of further exposure, with coalescence and
• Laxity in national commitment to root out malaria. formation of non-segmental conglomerates of
C. Operational failure: irregular masses in excess of 1 cm in diameter. These
• Inadequate surveillance. masses enlarge and are characteristic of progressive
• Inadequate coverage by residual insecticides during massive fibrosis. Significant functional impairment
acute phase leaving many houses unsprayed. with both restrictive and obstructive components may
• Premature take-off into consolidation and be associated with this form of silicosis. In the late
maintainence phases. stages of the disease, ventilatory failure may develop.

158. The answer is b. 161. The answer is d.


With velamentous insertion of the cord, the umbilical Syphilis is a chronic disease produced by the
vessels separate in the membranes at a distance from spirochete Treponema pallidum. Because of the
the placental margin, which they reach surrounded spirochete’s extreme thinness, it is difficult to detect
only by amnion. It occurs in about 1% of singleton by light microscopy; therefore, spirochetes are
gestations but is quite common in multiple diagnosed by use of a specially adapted technique
pregnancies. Feta malformations are more common known as dark-field microscopy Clinically, syphilis is
with velamentous insertion the umbilical cord. When divided into primary, secondary, and tertiary (or late)
fetal vessels cross the internal us (vasa previa), stages. In primary syphilis a hard chancre develops.
rupture of membranes may be accompanied by This is a painless ulcer with an indurated base that is
rupture of a fetal vessel, leading to fetal usually found on the vulva, vagina, or cervix.
exsanguination. An increased risk of premature Secondary syphilis is the result of hematogenous
rupture of membranes and of torsion of the umbilical dissemination of the spirochetes and thus is a systemic
cord has not been described in association with dis ease. There are a number of systemic symptoms
velamentous insertion of the cord. depending on the major organs involved. The classic
rash of secondary syphilis is red macules and papules
159. The answer is b. over the palms of the hands and the soles of the feet.
An important feature of the lymphatic drainage of the
vulva is the existence of drainage across the mid line. 162. The answer is b.
The vulva drains first into the superficial inguinal Psychological symptoms during the climacteric occur
lymph nodes, then into the deep femoral nodes, and at a time when much is changing in a woman’s life.
finally into the external iliac lymph nodes. The Steroid hormone levels are dropping, and the menses
clinical significance of this sequence for patients with is stop ping. However, studies show these two factors
carcinoma of the vulva is that the iliac nodes are to be unrelated to emotional symptoms in most
probably free of the disease if the deep femoral nodes women. Many factors, such as hormonal,
are not involved. Unlike the lymphatic drainage from environmental, and intrapsychic elements, combine to
the rest of the vulva, the drainage from the clitoral cause the symptoms of the climacteric such as
region bypasses the superficial inguinal nodes and insomnia; vasomotor instability (hot flushes, hot
passes directly to the deep femoral nodes. Thus, while flashes); emotional lability; and genital tract atrophy
the superficial nodes usually also have metastases with vulvar, vaginal, and urinary symptoms.
when the deep femoral nodes are implicated, it is
possible for only the deep nodes to be involved if the 163. The answer is B
carcinoma is in the midline near the clitoris. Maternity benefit is given to employees under ESI
act.
160. The answer is A This benefit is payable in cash to an insured women
Silicosis for confinement/miscarriage or sickness arising out of
pregnancy/confinement or premature birth of child or
miscarriage.

DBMCI, Ph: 9810150067, 9868387223, E-mail: drbhatiainstitute@rediffmail.com, Visit us at: www.dbmci.com 23


Grand Test – 36 Explanatory Answer

• For confinement, duration of benefit is 12 weeks. eyes, and nervous system. Most patients with the
• For miscarriage, 6 weeks and for sickness arising out disease develop antibodies to certain proteins in the
of confinement etc. 30 days benefit is allowed at full cytoplasm of neutrophils called antineutrophil
wages. cytoplasmic antibodies (ANCA).
Benefits to employees under ESI act (1948)
168. The answer is b.
The complaints described are characteristic of Sjogren
syndrome, an autoimmune disease with presenting
Medical Sickness Maternity Disability Dependents symptoms of dry eyes and dry mouth. The disease is
Benefit Benefit Benefit Benefit Benefit caused by lymphocytic infiltration and destruction of
lacrimal and salivary glands. Dry eyes can be
164. The answer is d. measured objectively by the Schirmer test, which
As patients age, the incidence of vesicle instability or measures the amount of wetness of a piece of filter
unstable bladder increases dramatically. Although paper when exposed to the lower eyelid for 5 minutes.
estrogen has been reported to decrease urgency, Most patients with Sjogren syndrome produce
frequency, and nocturia in menopausal women, its autoantibodies particularly anti-Ro (SSA). Lip biopsy
effect on correction of stress urinary incontinence or is needed only to evaluate uncertain cases, such as
vesicle instability is unclear. In the elderly population when dry mouth occurs without dry eye symptoms.
there are also many transient causes of incontinence
that the physician should consider. These include 169. The answer is D
dementia, medications (especially a-adrenergic Socio-economic status has been defined as the
blockers), decreased patient mobility, endocrine position that an individual or family occupies with
abnormalities (hypercalcemia, hypothyroidism), stool reference to the prevailing average standards of
impaction, and urinary tract infections. cultural and material possessions, income and
participation in group activity of the community.
165. The answer is a There have been many attempts at developing scales
Herpes simplex encephalitis can occur in patients of for measuring socio-economic status. Kuppuswamy in
any age usually in immunocompetent patients. The India prepared a scale based on education, income and
bizarre behavior includes personality aberrations, occupation which are three major variables
aberration, hypersexuality or sensory hallucinations contributing to socio-economic status in urban areas.
CSF shows lymphocytes with a close to normal sugar
and protein. Focal abnormalities are seen in the 170. The answer is b.
temporal lobe by CT scan, MRI, or EEG. The diagnosis in this patient is suggested by the
physical exam findings. The findings of poor
166. The answer is A excursion, flatness of percussion, and decreased
Severe diarrhoea is not a clinical feature of chronic fremitus on the right side are all consistent with a
lead poisoning. Constipation is seen in it. right-sided pleural effusion. A large right- sided
Stippling of RBC and blue line in gums are very effusion may shift the trachea to the left.
important features. Histoplasmosis would be one possible cause of such
The various features of lead poisoning are: an effusion. A pneumothorax should result in
1. The most serious manifestation of lead poisoning is hyperresonance of the affected side. Atelectasis on the
acute encephalopathy. Encephalopathy includes right side would shift the trachea to the right. A
persistent vomiting, ataxia, seizures, papilloedema, consolidated pneumonia would characteristically
impaired consciousness, and coma. result in increased fremitus, flatness to percussion,
2. Lead colic, characterized by occasional vomiting, and bronchial breath sounds, and would not cause
intermittent abdominal pain, and constipation is an tracheal deviation.
important feature.
3. Peripheral neuropathy, which is common in adults, 171. The answer is c.
is rare in children, except for those with sickle cell The patient’s chest pain that is relieved by sitting up is
disease. Foot and wrist drop are characteristic. most likely due to pericarditis. A pericardial friction
rub may initially be present, then disappear, with the
167. The answer is a heart sounds becoming fainter as an effusion
Wegener’s granulomatosis is a granulomatous develops. Lung sounds are typically clear. An
vasculitis of small arteries and veins that affects the enlarged cardiac silhouette without other chest x-ray
lungs, sinuses, nasopharynx, and kidneys, where it findings of heart failure suggests pericardial effusion.
causes a focal and segmental glomerulonephritis.
Other organs can also be damaged, including the skin,

DBMCI, Ph: 9810150067, 9868387223, E-mail: drbhatiainstitute@rediffmail.com, Visit us at: www.dbmci.com 24


Grand Test – 36 Explanatory Answer

172. The answer is B An active lifestyle and good exercise program can
Health education: if people were adequately informed prevent the complications of diabetes. Benefits
about diseases and were encouraged to take necessary include blood pressure control, reduction in body fat,
precautions in time, a large number of diseases could weight loss, and increased insulin sensitivity.
be prevented with little or no medical intervention. it However, there are pitfalls to such a program. Some
can help to increase knowledge and to reinforce forms of exercise might jeopardize adequate foot care,
desired behaviour patterns. and foot exam becomes particularly important in the
patient who is doing weight-bearing exercise.
173. The answer is a Exercise can induce hypoglycemia by potentiating
Hypokalemia typically increases automaticity of insulin action; this is particularly true in the type 1
myocardial fibers, which results in ectopic beats or diabetic. Diabetics who have risk factors for
arrhythmias. Electrocardiography in hypokalemia cardiovascular disease, such as hypertension and
reveals flattening of the T wave and prominent U hyperlipidemia, should undergo an exercise stress test
waves. Hyper kalemia decreases the rate of prior to engaging in a rigorous exercise program. This
spontaneous diastolic depolarization in all pace maker is important because asymptomatic cardiovascular dis
cells. It also results in slowing of conduction. One of ease is more common in diabetics.
the earliest electrocardiographic signs of
hyperkalemia is the appearance of tall, peaked T 178. The answer is a.
waves. More severe elevations of the serum potassium Paracentesis is required to evaluate new-onset ascites.
result in widening of the QRS complex. At serum While cirrhosis and portal hyper tension are most
sodium levels compatible with life, neither likely in this patient, complicating diseases such as
hyponatremia nor hypernatremia results in any tuberculous peritonitis and hepatoma are ruled out by
characteristic electrocardiographic abnormalities. analysis of ascitic fluid. An ultrasound or CT scan can
be used to demonstrate ascitic fluid in equivocal
174. The answer is D cases.
The other three conditions mentioned will cause
decrease in lactic acid level in aqueous humour. Lens 179. The answer is B
metabolises its 80% glucose by anaerobic metabolism • Addiction to barbiturates leads to craving or both
which leads to formation of lactic acid in the lens physical and psychic dependence.
which diffuses in the aqueous humour. Hence absence • Amphetamines also called ‘superman drugs’ are
of lens will lead to decreased lactic acid levels in the basically stimulants. Use of these drugs result in
aqueous humour. Hence the answer is senile cataract. psychic dependence. With large doses this Is often
rapid and strong.
175. The answer is b. • Cocaine is a central nervous system stimulant and
This patient has widespread Paget’s disease of bone. causes no physical dependence, no withdrawal
Excessive resorption of bone is followed by symptoms and no tolerance.
replacement of normal marrow with dense, trabecular, Cannabis use is associated with psychic dependence.
disorganized bone. Hearing loss and tinnitus are due
to direct involvement of the ossicles of the inner ear. 180. The answer is b.
Plasma alkaline phosphatase levels represent The excess ability to displace a flexed leg anteriorly is
increased bone turnover. Neither myeloma or the anterior drawer sign, used to test for disruption of
metastatic bone disease would result in bony the anterior cruciate ligament (ACL) tear. If the ACL
deformity such as skull enlargement. Alkaline is torn, when the knee is flexed, the cranial part of the
phosphatase is a marker of bone formation and does leg may be pulled forward excessively. Answer a
not rise in pure lyric lesions such as multiple would be true if the posterior cruciate ligament were
myeloma. torn.

176. The answer is B 181. The answer is b.


The social and environmental factors associated with The deep incisure in the inferior border of the pedicle
mental ill health comprise: worries, anxieties, ensures that the spinal nerve associated with that
emotional stress, tension, frustration, unhappy vertebra will exit through the intervertebral fora men
marriages, broken homes, poverty, industrialisation, well above the intervertebral disk so that it will not be
urbanisation, changing family structure, population affected by a herniation at that level. However, a
mobility, economic insecurity, cruelty, rejection, posterolateral herniation (the usual direction) will
neglect and the like. impinge on the next lower nerve as it courses toward
its associated intervertebral foramen. In this case, pain
177. The answer is d. was distributed along the medial side of the leg and

DBMCI, Ph: 9810150067, 9868387223, E-mail: drbhatiainstitute@rediffmail.com, Visit us at: www.dbmci.com 25


Grand Test – 36 Explanatory Answer

foot as far as the great toe—the distribution of the The left adrenal gland usually anastomoses with the
saphenous branch of the femoral nerve (L5). hemiazygos vein and may provide an important route
Herniation of the fourth lumbar intervertebral disk of collateral venous return. The right adrenal gland
between vertebral bodies L4-L5 would affect nerve usually drains directly into the inferior vena cava.
L5.
187. The answer is d.
182. The answer is C The ilioinguinal nerve exits the abdominal wall
Some of the factors identified for the suboptimal through the superficial inguinal ring, where it is
functioning of primary health care institutions are: applied to the anterior surface of the spermatic cord.
a. Inappropriate locations, poor access and poor Section of this nerve will result in paresthesia over the
maintenance. base of the penis and scrotum. The femoral branch of
b. Gaps in critical manpower. the genitofemoral nerve innervates the upper medial
c. Mismatch between personnel and equipment. surface of the thigh, where it mediates the afferent
d. Entrenchment of a curative culture within the limb of the cremasteric reflex. The efferent limb of
existing health system. this reflex is carried by the genital branch of the
e. High concentration of health services and health genitofemoral nerve, which lies within the are master
personnel in urban areas. layer. The dartos response, which is sympathetic,
arises from the sacral sympathetic chain and reaches
183. The answer is d. the pudendal nerve via gray rami communicantes.
The pedicle attaches to the vertebral body and extends
posteriorly and from there both the lamina and the 188. The answer is C
transverse process are given off. The syphilitic chancre, like the condylomata, is
generally a painless lesion. Gonorrhea in the male is
184. The answer is c characterized by purulent urethral discharge rather
The body and tail of the pancreas receive most of their than lesions on the external genitalia. Epispadias is a
blood supply from the splenic artery via the great congenital defect in which the urethral meatus appears
pancreatic, dorsal pancreatic, and caudal pancreatic on the dorsum of the penis.
arteries. The head of the pancreas is supplied by the
superior pancreaticoduodenal artery that arises from 189. The answer is c.
the gastroduodenal branch of the common hepatic In a coronary bypass procedure a blood vessel is
artery. In addition, the pancreatic head is supplied by added to allow blood to flow distal to the occluded
the inferior pancreaticoduodenal arteries that arise coronary artertery This is generally done by removing
from the superior mesenteric artery. the internal thoracic artery which runs along the inner
The chief supply to the left side of the gastric fundus surface of the sternum, and then its proximal end is
is from the splenic artery via the short gastric attached to the ascending aorta and the distal end is
branches. The splenic artery also gives rise to the left connected to the occluded coronary artery, just distal
gastro-omental artery that runs along the greater to the blockage, thus bypassing the problem.
curvature to anastomose with the right gastro-omental Remember that the internal thoracic artery supplies
branch that arises indirectly from the common hepatic blood to each subcostal artery, which are branches of
artery. the thoracic aorta. All of the other answers involve
attaching the blood vessel to a venous structure or in
185. The answer is C the case of the pulmonary trunk attaching to relatively
Network analysis is a graphic plan of all events and unoxygenated blood, which is not done.
activities to be completed in order to reach an end
objectives. It brings greater discipline in planning. 190. The answer is d.
2 common types of network analysis are: The afferent limb of the cardiac reflex is mediated by
a. PERT. the carotid branch of the glossopharyngeal nerve (CN
b. CPM (critical path method). IX) from the aortic body and sinus as well as by the
CPM is longest path of network analysis. If any va nerve (CN X) from the aortic body The efferent
activity along the critical path is delayed, the entire limb, which is ca by the sym pathetic division of the
project will be delayed. autonomic nervous system, mediates increases in
heart rate and strength of heart beat through release of
186. The answer is d. norepinephrine at the postganglionic effector site. The
The venous drainage from each adrenal gland tends to sympathetic cardiac accelerator fibers, affecting
be through a single vein. The left adrenal gland primarily the ventricles, are derived from the superior,
usually drains into the left renal vein superior to the middle, and inferior cervical ganglia (cervical cardiac
point where the gonadal vein enters the left renal vein. nerves) as well as from the upper four thoracic

DBMCI, Ph: 9810150067, 9868387223, E-mail: drbhatiainstitute@rediffmail.com, Visit us at: www.dbmci.com 26


Grand Test – 36 Explanatory Answer

ganglia (thoracic cardiac nerves), whence they 194. The answer is D


converge on the cardiac plexus before reaching the Post-neonatal mortality rate is defined as the ratio of
heart. Parasympathetic fibers derived from CN X and post-neonatal deaths of children (between 28 days and
its recurrent laryngeal branch decrease heart rate and one year of age) in a given year to the total number of
stroke volume through release of acetylcholine, live births in the same year. It is expressed as rate per
principally in the vicinity of the sinoatrial node. 1,000.
No. of deaths of children
191. The answer is B between 28 days and one
All travellers including infants exposed to risk of year of age in a given year
yellow fever or passing through endemic zones of Post neonatal mortality rate = x1000
yellow fever must possess a valid international Total live births in the same year
certificate of vaccination against yellow fever before Applying the given data
they are allowed to enter yellow fever ‘receptive
areas’. 8
If no such certificate is available, the traveller is Post neonatal mortality rate = x 1000 = 80
placed on quarantine, in a mosquito-proof ward, for 6 100
days from date of leaving an infected area.
If the traveller arrives before the certificate becomes 195. The answer is b.
valid, he is isolated till the certificate becomes valid. The vestibular hair cells are the sensory transduction
system of the inner ear and are responsible for the
192. The answer is c. conversion of mechanical energy into an electrical
The posterior cricoarytenoid muscles rotate the signal for cranial nerve VIII. These cells are called
arytenoid laterally, which swings the vocal process of hair cells because their surface contains stereocilia.
that cartilage outward to abduct the vocal cords and These are modified microvilli that contain a large
open the glottis. These are the sole abductors of the number of actin filaments and extend from the surface
vocal folds. The lateral cricoarytenoid muscles and of the cell. The stereocilia are different lengths and
the unpaired transverse arytenoid muscle adduct the are arranged in order by size with a large kinocilium
vocal folds. The thyroarytenoid muscle and its at one end. The arrangement of the stereocilia is very
innermost portion, the vocalis muscle, act to tense the important because bending in one direction (i.e.,
cords. The cricothyroid muscle lengthens the vocal toward the kinocilium) depolarizes the cell and
cords. increases the rate of nervous dis charge, whereas
bending them in the other direction (i.e., away from
the kinocilium) results in hyperpolarization and
193. The answer is c. decreased neural discharge. The classification of type
The symptoms indicate that the lesion is at the level of of hair cell (I or II) is based on the pattern of efferent
the midbrain. The spastic paralysis, hyperreflexia, and and afferent innervation.
positive Babinski reflect an upper motor neuron
lesion. The corticobulbar and corticospinal tracts pass 196. The answer is b.
through the cerebral peduncles (basis pedunculi). The retinal pigment epithelium (RPE) is a single layer
Those originating in the right cortex will pass through of cells that phagocytose old components of
the right peduncle and then cross to the contralateral photoreceptor cells. It is derived from the outer layer
side in the pyramidal decussation, resulting in left- of the optic cup and is continuous from the ora serrata
side hemiplegia. It is of interest that the lower motor retinae to the optic nerve. Microvilli are prominent on
neurons innervating muscles of facial expression the apical surfaces of the RPE and play an important
located below the eye receive upper motor neurons role in the maintenance of the blood-retinal barrier. In
(corticobulbar tract) only from the contralateral addition, the RPE synthesizes melanin and stores
cortex, whereas lower motor neurons innervating vitamin A for the photoreceptor cells. Rod and cone
facial muscles above the eye (e.g., frontalis) receive perikarya and amacrine cells are found in the
input from both sides of the cortex. This explains why photosensitive retina derived from the inner layer of
only the lower portion of the left face was paralyzed. the optic cup.
The deficit in movement of the right eye indicates
damage to the ipsilateral oculomotor nerve (CN III), 197. The answer is C
which passes through the cerebral peduncle en route Types of biological transmission
to the interpeduncular fossa. The “down and out” • Cyclodevelopmental: The disease agent undergoes
direction of the right eye is explained by unopposed only development but no multiplication e.g.,
contraction of the lateral rectus (CN VI) and superior microfilaria in mosquito.
oblique (CN IV) muscles.

DBMCI, Ph: 9810150067, 9868387223, E-mail: drbhatiainstitute@rediffmail.com, Visit us at: www.dbmci.com 27


Grand Test – 36 Explanatory Answer

• Cyclopropagative: Agent changes in form and • Hill-Sach’s lesion is depression on the humeral head
number e.g., malaria parasites in mosquito. in its postero-lateral quadrant caused by impingement
• Propagative: Agent merely multiplies in vector, but by the anterior edge of the glenoid on the head as it
no change in form e.g. plague bacilli in rat fleas. dislocates.
• Rounding off of the anterior glenoid rim and
198. The answer is D associated fractures of greater tuberosity of the
To find out any association between two variables we humerus or rim of glenoid may be associated with
often calculate what is known as co-efficient of anterior dislocation of shoulder.
correlation, which is represented by the symbol ‘r’. • Typical history of fall on outstretched hand,
The correlation coefficient lies between —1.0 and abducted shoulder and elbow supported with opposite
+1.0. hand. Lost normal contour of shoulder with flattening
If correlation coefficient is +1, if indicates a strong and fullness below clavicles is present on
positive association. examination.
Value near—1 indicates strong negative association
between two variables. If r = o it indicates there is no 201. The answer is d.
association between two variables. Patients with a pheochromocytoma often have
Correlation does not necessarily prove causation. paroxysms that are the hall mark of this tumor. These
are seizure-like catecholamine-induced attacks that
199. The answer is D include headache, profuse sweating, palpitations, and
The mammary gland enlarges during pregnancy in overall anxiety Pheochromocytoma is a common
response to several hormones, including prolactin tumor of the adrenal medulla that leads to an excess of
synthesized by the anterior pituitary estrogen and norepinephrine, which causes hypertension and hyper
progesterone synthesized by the corpus luteum, and glycemia. Vasoconstriction of arterioles occurs in
placental lactogen. The alveoli at the end of the duct conjunction with the increased blood pressure.
system respond to those hormones by cell Epinephrine (e.g., cortisol and growth hormone) have
proliferation, which increases the size of the anti-insulin effects, thus causing hyperglycemia.
mammary glands. Growth continues throughout
pregnancy; however, secretion is most notable late in 202. The answer is d.
pregnancy Milk is synthesized in the alveoli and is Hirschsprung’s disease (congenital megacolon) and
stored in their lumina before passage through the Chagas’ disease have different etiologies, but both
lactiferous ducts to the nipples. Secretion of milk inhibit intestinal motility by affecting the myenteric
lipids occurs by an apocrine mechanism whereby (Auerbach’s) plexus located between the layers of the
some apical cytoplasm is included with the secretory muscularis externa in the figure. The submucosal
product. In comparison, milk proteins, such as the (Meissner’s) plexus is more involved in regulation of
caseins, are secreted by exocytosis. Oxytocin is lumenal size and, therefore, will affect defecation, but
required for the release of milk from the mammary will be less involved in peristalsis. Vascular smooth
gland through the action of the myoepithelial cells muscle, the muscularis mucosa, and enteroendocrine
that surround the alveoli and proximal (closer to the cells do not play a major role in the regulation of
alveolus) portions of the duct system. Oxytocin is not peristalsis, which is observed even after removal of
required for milk synthesis. Neurohumoral reflexes the gut and placement in a nutrient solution.
are involved in the suckling-milk ejection response. Hirschsprung’s disease, also known as aganglionic
megacolon, results from failure of normal migration
200. The answer is B of neural crest cells to the colon, resulting in an
After a history of fall inability to lift upper limb with aganglionic segment. Although both the myenteric
loss of normal rounded contour of shoulder, and submucosal plexuses are affected, the primary
prominent acromion is seen characteristically in regulator of intrinsic gut rhythmicity is the myenteric
anterior dislocation of shoulder. plexus. Chagas’ disease is caused by the protozoan
Important points about dislocation of shoulder: Trypanosoma cruzi. Severe infection results in
• Commonest joint in human body to dislocate. extensive damage to the myenteric neurons.
• Anterior dislocation is much more common than
posterior. 203. The answer is A
• In anterior dislocation subcoracoid type is most • CT is more sensitive than MRI for visualizing fine
common. osseous details.
• Bankart’s lesion is stripping of glenoid labrum along • CT is also more sensitive and specific than MRI for
with the periosteum from the anterior surface of the acute subarachnoid hemorrhage.
glenoid and scapular neck. • Ultrasound is used only when fontanel are open in
child (infant).

DBMCI, Ph: 9810150067, 9868387223, E-mail: drbhatiainstitute@rediffmail.com, Visit us at: www.dbmci.com 28


Grand Test – 36 Explanatory Answer

204. The answer is c.


Psoriasis is a chronic disease that affects both the 207. The answer is d
epidermis and dermis of the skin. There is hyperplasia During prenatal development the first site of blood
of the epidermis and abnormal microcirculation in the cell development (hematopoiesis) is extraembryonic,
dermis as venules predominate in the capillaries in the yolk sac. The yolk sac produces
resulting in increased extravasation of inflammatory hematocytoblasts and primitive erythroblasts from the
cells. Thus, the underlying cause is the infiltration of third week through the second month of gestation.
inflammatory cells into the dermis with further Hepatic erythropoiesis begins during the sixth week,
migration of neutrophils into the epidermis. Those reaches its maximum in the third month, and then
inflammatory cells release cytokines that induce an ceases about the seventh month. The bone marrow
inflammatory response. There is hyperplasia of the begins to function in the second month and becomes
epidermis as keratinocytes traverse the cell cycle in a the predominant hematopoietic site during months 5
shorter period of time. Microabscesses form in the to 9 of gestation, whereas the spleen is involved
epidermis and epithelia are avascular. specifically in the production of red blood cells
(erythropoiesis) from months 2 to 5 of gestation with
205. The answer is c. some activity continuing until shortly after birth.
Immunoglobulin switching normally occurs in the Although erythropoiesis ceases in the spleen, this
germinal centers during the maturation of B cells. organ continues to produce monocytes and
Synthesis of B cell antibody begins as IgM inserted lymphocytes throughout life. In addition, from the
into the cell membrane and then switches to second month the lymph nodes produce lymphocytes,
membrane-bound IgM and IgD. After antigen and the thymus is responsible for the education of T
stimulation, a switch to surface IgM, IgA, IgG, or IgE cells after the second month of gestation. Those I
occurs, and these antibodies are secreted. Most lymphocytes seed to I-dependent areas, such as the
antibody production occurs in the germinal centers of deep cortex of the lymph node and periarteriolar
the lymph nodes, tonsils, and spleen. It occurs to a lymphoid sheath (PALS) of the spleen.
lesser extent in the bone marrow, but the bone marrow
functions in the education of B cells as well as 208. The answer is B
representing the major site of hematopoiesis in the Toxoplasmosis --- causes choroiditis
adult. The thymus is responsible for the education of
T cells. The splenic red pulp is the site of red blood 209. The answer is a.
cell breakdown. The sarcoplasmic reticulum provides a mechanism for
the muscle cell to regulate the concentration of
206. The answer is D cytosolic calcium. It is a modified smooth
There is suppression of cell mediated of immunity. If endoplasmic reticulum that serves alternatively as a
immunity improves there is increase in production of storage site and a source of cellular calcium. Calcium
oxygen metabolites in macrophages stimulated by is released from the sarcoplasmic reticulum during
lymphokines from the activated lymphocytes muscle contraction; it is stored during relaxation.
generated during the Immune response and During repolarization, calcium is actively transported
Leishmania are killed. Although having a doubtful from the cytosol to the sarcoplasmic reticulum
role in immunity the production of immunoglobulin through the activity of a Ca2+- ATPase. Calsequestrin
especially IgG is increased. is a calcium-binding protein found in the sarcoplasmic
Important points about leishmaniasis (Kala-azar): reticulum that sequesters calcium within the
• Man is only reservoir of infection in Indian kala- sarcoplasmic reticulum. The sarcoplasmic reticulum
azar. contains many calcium channels that open in response
• Leishmania multiply within monocytes. to depolarization and result in a massive increase in
• Clinical features of loss of weight and appetite, cytosolic calcium. During repolarization of the I
fever, splenomegaly, hepatomegaly, leucopenia, system, Ca2+ is transported from the cytosol and
pigmentation of skin and peripheral lymphadenopathy sequestered in the sarcoplasmic reticulum through the
in some setting (Sudan) are seen. activity of Ca2+-ATPase and calsequestrin,
• Diagnosis is by splenic aspiration, 90% cases give respectively Glycogen is stored as particles or
positive result and by bone marrow aspirate and droplets in the cytoplasm, which contains the enzymes
biopsy, positive in over 70% cases. required for the synthesis and breakdown of glycogen.
• Post Kala-azar dermal leishmaniasis, occurs one to
several years after apparent cure. 210. The answer is B
• Pentavalent antimonial compounds are used as first As malignant change rarely occurs in Peutz-Jeghers
level of management. syndrome, resection is necessary only for
• In resistant cases amphotencin B is used. intussusception or intractable bleeding.

DBMCI, Ph: 9810150067, 9868387223, E-mail: drbhatiainstitute@rediffmail.com, Visit us at: www.dbmci.com 29


Grand Test – 36 Explanatory Answer

• Hamartomatous polyp throughout intestine 214. The answer is d.


(maximum in jejunum). Fibronectin is an adhesive glycoprotein that is
• Autosomal dominant inheritance. important for cell attachment. It is important for
• Associated with theca or granulosa cell tumours of modulation of eel migration in the adult and during
ovary. development. Neural crest and other Cells appear to
• Characteristically pigmentation (melanosis) of the be guided along fibronectin-coated pathways in the
oral mucosa and lips is seen. embryo. Fibronectin is found in three forms: a plasma
• Sometimes also present on the digits and the form that is involved in blood clotting; a cell-surface
perianal skin. form, which binds to the cell surface transiently; and a
• It is a type of familial polyposis syndrome. matrix form, which is fibrillar in arrangement.
Fibronectin contains a cell-binding domain (RGD
211. The answer is d. sequence), a collagen-binding domain, and a heparin-
The osteosarcoma cells synthesize bone and function binding domain. Type IV collagen is responsible for
similarly to osteoblasts, from which they are derived. providing support.
Osteoblasts and osteosarcoma cells synthesize
alkaline phosphatase which is critical in increasing the 215. The answer is a.
local calcium phosphate ratio, thereby inducing the In the formation of tubular structures from flat sheets,
calcification of osteoid (prebone) to form bone. there is contraction of the microfilament bundles
Osteoblasts synthesize type I collagen. Fibroblasts in associated with the adhesion belt junctions (zonula
highly cellular organs such as the spleen synthesize adherens). This occurs in neural tube formation,
type III collagen, the primary component of reticular which involves the conversion of the neural plate into
fibers. Chondrocytes in hyaline and elastic cartilage the neural tube. In the apical part of the cells, the actin
synthesize type II collagen. Acid phosphatase is filament bundles con tract, narrowing the cells at their
synthesized by osteoclasts and is essential for the apical ends. The position of the zonula adherens,
dissolution of bone and its subsequent resorption. forming a contractile ring around the circumference of
Elastin is synthesized by smooth-muscle cells, the cell, coupled with the contractile nature of the
endothelial and microvascular cells, chondrocytes, actin microfilament bundles is ideal for regulating
and fibroblasts. morphogenetic changes. Desmosomes are involved in
resisting shear forces and are not directly involved in
212. The answer is d. this process. The zonula occludens prevents leakage
Growth in the length of long bones after birth between cells. Gap junctions facilitate communication
(postmeatal) occurs through cell proliferation of between cells.
immature chondrocytes (chondroblasts) in the
secondary ossification centers of the epiphyses. The 216. The answer is C
primary ossification centers “close” soon after birth. The patient is suffering from post Kala azar dermal
Fetal development of long bones occurs by the leishmaniasis. It is not drug eruption because there is
process of endochondral ossification in which a no history of drug intake and drug eruptions are
cartilage model is replaced by bone. Before birth, usually itchy.
growth in length of the long bone occurs primarily In lepromatous leprosy AFB will be present in slit
through the proliferation of chondrocytes within the smear from nodules.
diaphysis of the cartilage model (primary ossification VDRL reactive in 1:2 dilution is false positive and
center). Growth in the width of the long bone occurs hence it indicates PKDL and rules out secondary
by the addition of osteoblasts from the periosteum and syphilis.
deposition of a periosteal collar
217. The answer is a.
213. The answer is B The chaperonin are proteins that regulate the
Bullous lesions are seen in porphyria cutanea tarda. unfolding of cytosolic proteins. They are members of
The lesions mainly appear on exposure to sunlight. the heat shock protein family (e.g., hsp 70). The
The lesions heal with scarring. Alcohol is the chaperonin assist with the translocation of proteins
commonest aetiological agent. across internal membranes of the cell (e.g.,
The main abnormality is in hepatic uroporphyrinogen mitochondria) by maintaining precursor proteins in
decarboxylase. Diagnosis is by increased level of their unfolded state during movement across the
urinary uroporphyrin. membrane. They do not function in the docking of the
Skin biopsy reveals subepidermal blisters with signal peptide or as the start-transfer signal in
perivascular deposition of PAS +ve material. translocation of the internal membrane of the
Venesection and chloroquine are used in treatment. endoplasmic reticulum. Microtubules are stabilized
by GTP capping, covalent modifications of tubulins,

DBMCI, Ph: 9810150067, 9868387223, E-mail: drbhatiainstitute@rediffmail.com, Visit us at: www.dbmci.com 30


Grand Test – 36 Explanatory Answer

and microtubule-associated proteins (MAPs). The


MAPs regulate disassembly of MTs and provide the 222. The answer is D
linkage between organelles and MTs. Clathrin-coated The farmer is suffering from actinic reticuloid.
vesicles are responsible for the selective transport of Actinic reticuloid is a severe and incapacitating
membrane receptors. photodermatitis usually seen in persons doing outdoor
work with exposure to sun rays (UV-B). Clinically, it
218. The answer is c. is characterised by extreme photosensitivity to a broad
The formation of the acrosome, a specialized spectrum of radiation. Scaly erythema develops
secretory granule, is one of many maturation events gradually on the head, neck and dorsa of the hands in
occurring in spermio genesis (the process by which light exposed areas. The eruption progresses and
mature sperm are formed from the spermatids). becomes lichenified. Thick pruritic plaques develop
Acrosome formation involves lytic enzyme which may become confluent and form furrows. In
maturation and occurs after division of secondary severe cases, the eruption may involve the hairy scalp,
spermatocytes. It involves no mitotic or meiotic shielded areas of the face and most of the body
activity. The acrosome develops from Golgi vesicles surface. Pruritus may be severe and intractable, and
just like any other secretory granules. It contains can cause depression. Histologically there is
acrosin, a serine protease, hyaluronidase, and lymphomatoid infiltrate. The cause is unknown but
neuraminidase, responsible for the penetration ability sometimes it is a progression from photocontact
of the sperm. The developing cells are in contact with dermatitis.
Sertoli cells for all of the stages of spermiogenesis. At The condition is far more common in men than
the end of spermiogenesis, spermatids are released by women, and in the elderly.
Sertoli cells in a process called spermiation.
Decapacitation factors are not involved in acrosomal 223. The answer is d.
maturation. The pituitary gland (hypophysis cerebri) is formed
from ectoderm. An outgrowth of the oral ectoderm,
219. The answer is D Rathke’s pouch, forms the structures that compose the
In diabetes mellitus polyneuropathy is one of the adenohypophysis: pars distalis, pars intermedia and
commonest complication. It may be sensory, motor, pars tuberalis The pars distalis include the classic
autonomic or mixed neuropathy but nerve thickening histologic cell types: chromophils (acidophils and
is absent. basophils) and chromophobes (acidophils and
In rest of the three conditions nerve is thickened and basophils that are depleted of secretory product) Since
tuberculoid leprosy is the commonest cause of lateral the development of immunocytochemistry the
popliteal nerve thickening. Usually there is fusiform classification scheme for pars distalis cell types has
dilatation of nerve. been changed to include lactotrophs (prolactin)
somatotrophs (growth hormone) corticotrophs (ACTH
220. The answer is b. -l1potropin -MSH and -endorphin) thyrotrophs (TSH),
Lipid digestion occurs in the small intestinal lumen by and gonadotrophs (FSH and LH). The pars intermedia
the action of bile (from the liver and bile duct) and is also formed from the oral ectoderm is rudimentary
lipase (from the pancreas). Bile serves to emulsify the in humans, and may produce a prepro-
lipid to form micelles, while lipase breaks down the opiomelanocortic peptide, The pars tuberalis forms a
lipid from triglycerides to fatty acids, glycerol, and collar around the pituitary stalk and is also derived
monoglycerides. These three breakdown products from the oral ectoderm The pars ne (including Herring
diffuse freely across the micro border to enter the bodies) and the remainder of the Pituitary stalk
apical portion of the enterocyte. Triglycerides are (infundibular stem and median eminence) are formed
resynthesized in the smooth endoplasmic reticulum. from a down growth of the diencephalon. The
Proteins are synthesized in the RER and are combined posterior pituitary (pars nervosa and stalk) retains this
with sugar and lipid portions in the Golgi to form close relationship with the brain (i.e. hypothalamus)
glycoproteins and lipoproteins. throughout life.

221. The answer is d. 224. The answer is c.


The saliva contains water, enzymes (amylase and To a minor extent, the uterine cervical stroma changes
lysozyme), mucins, lactoferrin, and secretory IgA. during each reproductive cycle; however, during
Amylase digests starches; mucins primarily lubricate; pregnancy (especially parturition) there is a thinning
lactoferrin is an iron-binding protein; and IgA is of the uterine stroma. This results in eversions
secretory immunoglobulin involved in the host (mistakenly called “erosions”), which are sites of
defenses of the oral cavity Fc receptors on the salivary exposed uterine columnar epithelium in the acidic,
epithelia help transport IgA into, and across, the cell. vaginal milieu. These sites often become re-

DBMCI, Ph: 9810150067, 9868387223, E-mail: drbhatiainstitute@rediffmail.com, Visit us at: www.dbmci.com 31


Grand Test – 36 Explanatory Answer

epithelialized as stratified epithelium (squamous C2: Nodes accompanying the supplying blood vessels
metaplasia) and are believed to be the location of are involved up to the point of division.
cancerous transformation in the cervix. As part of the Superior rectal artery, a direct continuation of inferior
process of re-epithelialization, the openings of mesenteric artery is the main blood supply of rectum
cervical mucous glands are obliterated, which results and hence the answer is choice D.
in the formation of cysts called nabothian follicles.
229. The answer is B.
225. The answer is A Most clinical problems arising in patients with sickle
A serosanguinous (pink) discharge from the wound is cell anemia are due to vaso-occlusive phenomena
a forerunner of disruption in 50% of cases. It is most caused by sickling of deoxygenated red blood cells in
pathognomonic sign of impending wound disruption capillaries. Microinfarcts can occur suddenly and
and it signifies intraperitoneal contents are lying cause severe pain in almost any part of the body,
extraperitoneally. although the abdomen, chest, back, and joints are
An emergency operation is required to replace the most commonly affected. These crises may be
bowel, relieve any obstruction and to resuture the precipitated by upper respiratory infection, cold
wound. While awaiting operation, reassure the patient weather, or dehydration.
and cover the intestine with a sterile towel.
230. The answer is A.
226. The answer is D. Although patients with myelodysplasia, especially
Microdeletions involving chromosome 15 can lead to those with normal cytogenetics and no excess
the Prader-Willi syndrome or to Angelman’s myeloblasts in the marrow or blood, can be man aged
syndrome. Therefore, this pattern of inheritance is for long periods of time with only supportive care,
consistent with imprinting. In PWS, for example, including transfusion, limitations exist. Fortunately,
nondeletion patients invariably have two maternal given careful screening for bloodborne viral
copies of chromosome 15 but no paternal copy. This infections, the incidence of transfusion-associated
is referred to as maternal uniparental disomy. For infection is now rare. However, the need for chronic
some nondeletion Angelman syndrome patients, transfusional therapy may be associated with iron
however, the reverse is true. Patients have two overload and the requirement to use an iron chelating
paternal copies but no maternal copy of chromosome agent such as desferrioxamine. Furthermore, either
15, referred to as paternal uniparental disomy. This due to their disease and associated immune
indicates that at least some genes on chromosome 15 dysregulation or due to exposure to many different
are differentially expressed, depending on which blood group antigens, chronically transfused patients,
parent contributed the chromosome. such as the one in question, frequently develop a large
panel of serum allo-antibodies. The presence of these
227. The answer is D. antibodies may make it very difficult to find blood
Patients with erythema multiforme major have typical that will be negative in a cross-match with the
target lesions, usually on the extremities, which are patient’s serum. While the presence of these
associated with mucous membrane lesions. This antibodies may or may not produce hemolysis if blood
syndrome usually follows mycoplasma or herpes containing a potential target antigen is transfused,
simplex infections, has a benign course, and is not blood bank practice requires a negative cross-match
thought to be associated with drug reaction. On the except in emergency situations.
other hand, the Stevens-Johnson syndrome is due to
reaction to a systemically administered drug such as a 231. The answer is C
sulfa, phenytoin, allopurinol, penicillin, and Acute mechanical large bowel obstruction should be
nonsteroidal anti-inflammatory drugs. Typical lesions urgently operated otherwise there is a chance of
of Stevens-Johnson syn drome are small blisters on perforation and peritonitis. This will occur when
dusky purpuric macules. 10 to 30% of cases include ileocaecal valve is competent. This will lead to
fever and lesions of the respiratory and/or building up of pressure within the large bowel and
gastrointestinal tracts. caecum perforation is very common in this situation.
Sometimes emergency caecostomy has to be done
228. The answer is D where cecal perforation is imminent.
Duke classified CA rectum into 3 stages:
A: Growth limited to rectal wall. 232. The answer is C.
B: Extends beyond extra rectal tissues but no Infection of cannulas occurs most commonly by
metastasis to regional lymph nodes. contamination during insertion or manipulation.
C: Metastasis in regional lymph nodes. Although the daily application of an antibacterial
Cl: Local pararectal lymph nodes are involved. ointment is recommended by some authorities, the

DBMCI, Ph: 9810150067, 9868387223, E-mail: drbhatiainstitute@rediffmail.com, Visit us at: www.dbmci.com 32


Grand Test – 36 Explanatory Answer

best way to prevent these infections is to change the effects and should not conform to a rigidly
cannula periodically, no less frequently than every 2 standardized regimen. Tolerance is common and must
or 3 days. An exception is the use of cuffed catheters, be considered if a patient fails to respond to a
which are inserted surgically into the subclavian vein previously efficacious dose. Long-acting preparations
and can be used for many weeks. Infections of such such as transdermal patches are less likely to produce
devices with relatively nonpathogenic organisms, the nitrate-associated side effects of headaches and
such as coagulase-negative staphylococci, may be dizziness than are the more rapidly acting sublingual
treated with intra venous antibiotics; however, gram- forms.
negative rod and candidal infections usually mandate
removal of the indwelling catheter. Infections of 237. The answer is A.
cannulas occur much less frequently as a result of the While prompt initiation of thrombolytic therapy
other factors listed in the question. during an acute myocardial is infarction associated
with improvement in mortality and limitation of the
233. The answer is D size of the infarct, all thrombolytic agents, including
Sarcoidosis tissue plasminogen activator, are associated with an
Sarcoid nodules may be present in eyelid, increased risk of major bleeding. These agents should
conjunctiva, sclera, iris. not be given if there is a history of a cerebrovascular
Neurofibromatosis accident, a surgical procedure within the past 2 weeks,
Lisch nodules are melanocytic iris hamartomas. active peptic ulcer disease, or marked hypertension
SLE during acute presentation (systolic pressure >180 or
Ocular features diastolic pressure >100 mmHg). Other situations in
1. Punctate epithelial keratopathy—most common which the risk of bleeding may be higher, such as
2. Keratoconjunctivitis sicca advanced age, diabetic retinopathy, CPR for <10 mm,
3. Scleritis are not absolute contraindications, and the potential
4. Retinal vasculitis benefit from the administration of thrombolytic the
5. Optic neuropathy. should be considered carefully in each case.

234. The answer is D. 238. The answer is C


Patients who have localized sporotrichosis can be Whenever there is perforation there are three clinical
treated successfully with potassium iodide. However, stages.
systemic infections, particularly pneumonia in In the first stage there is severe pain abdomen due to
immunocompromised persons, should be treated with irritant effect of gastric contents on the peritoneum.
amphotericin B. Untreated persons can develop In the second stage there is dilution of acid in the
chronic sporotrichosis. Itraconazole may also be peritoneal cavity that results in decrease in the
effective in treating this condition. intensity of pain.
In the third stage gradually the pain returns and there
235. The answer is B is board like rigidity of abdomen and the abdomen
The baby is suffering from cystic fibrosis does not move with respiration.
(mucoviscidosis). At birth the meconium may set into However these three stages may not be clear-cut in
a very sticky mass and cause meconium ileus i.e., most of the patients. Important points:
intestinal obstruction due to sticky meconium. 1. Commonest site of perforation is anterior aspect of
In this disease there is generalized dysfunction of duodenum.
exocrine glands. 2. Gastric ulcers may perforate into lesser sac.
Lungs and panaceas are involved and steatorrhoea is a
common finding. 239. The answer is B.
The child is salty when kissed because of high level of Bronchoalveolar lavage in patients with idiopathic
Na and Cl in sweat. pulmonary fibrosis, a chronic inflammatory disorder
of the lower respiratory tract characterized by dyspnea
236. The answer is A. and reticulonodular infiltrates on chest radiography,
Nitrates are generalized smooth-muscle dilators discloses an abundance of alveolar macrophages.
whose direct effect on the vasculature cannot be Probably related to locally generated immune
blocked by any currently available agents. Long- complexes, alveolar macrophages become activated
acting preparations of nitroglycerin may be and then produce several mediators that recruit and
completely degraded by the liver in some patients and induce fibroblast proliferation, which causes
thus are generally less effective than sublingual forms. secondary damage. Macrophage-derived mediators
Because individual variability in metabolism is believed to be important in this process include
considerable, doses should be titrated against side fibronectin, a 200-kDa dimeric glycoprotein that

DBMCI, Ph: 9810150067, 9868387223, E-mail: drbhatiainstitute@rediffmail.com, Visit us at: www.dbmci.com 33


Grand Test – 36 Explanatory Answer

interacts with connective tissue matrix as well as by the plasma sodium times the urine creatinine, and
specific receptors on fibroblasts, and platelet-derived multiplying by 100. In this case the result is
growth factor, whose  chain is encoded by the c-sis approximately 1.4, which suggests that impaired
proto oncogene. Platelet-derived growth factor is reabsorption of sodium is ongoing and that intrinsic
believed to play an important role in recruiting renal failure is occurring.
fibroblasts to the site of inflammation. Macrophages
also produce chemotaxin such as leukotriene B4 and 243. The answer is D.
interleukin 8, which attract neutrophils and Amino acids (except for the branched-chain amino
eosinophils into the region. acids leucine, isoleucine, and valine) are taken up by
the liver via the portal circulation and are metabolized
240. The answer is A. to urea. Severe liver damage disrupts normal amino
Many patients who develop pulmonary acid metabolism and is reflected in elevated serum
thromboembolism have an underlying inherited levels of non-branched-chain amino acids. Since urea
predisposition that remains clinically silent until they cannot be produced, ammonia cannot be handled.
are subjected to an additional stress, such as the use of Elevated levels of serum ammonia certainly play a
oral contraceptive pills, surgery, or pregnancy. The large role in the development of hepatic
most frequently inherited pre disposition to encephalopathy in patients with liver failure and
thrombosis is so-called activated protein C resistance. portal hypertension. Therefore, levels of ammonia
The inability of a normal protein C to carry out its and, in the case of alkylosis, ammonium ion rise at the
anticoagulant function is due to a missense mutation expense of urea. Other mechanisms leading to
in the gene coding for factor V in the coagulation increased blood ammonia levels include excessive
cascade. This mutation, which results in the amounts of intestinal nitrogen (e.g., resulting from
substitution of a glutamine for an arginine residue in bleeding); decreased intestinal motility allowing
position 506 of the factor V molecule, is designated greater bacterial deamination of amino acids;
the factor V Leiden gene. Based on the Physicians depressed renal function leading to an increase in
Health Study, about 3% of healthy male physicians blood urea nitrogen and a greater opportunity for
carry this particular missense mutation. Carriers are bacterial urease to convert this to ammonia; alkalosis,
clearly at an increased risk for deep venous which will preferentially lead the NH equilibrium in
thrombosis and also for recurrence after the favor of ammonia; and portal hypertension, which
discontinuation of warfarin. The allelic frequency of will allow ammonia from the gut to bypass hepatic
factor V Leiden is more common than are all other detoxification.
identified inherited hypercoagulable states combined,
including deficiencies of protein C, protein S, and 244. The answer is C
antithrombin III and disorders of plasminogen. After complete transection of spinal cord there is a
stage of spinal shock. During this period there is loss
241. The answer is A of power, sensation and reflex activity below the
CVP and pulmonary wedge pressure does not give lesion. After this period there is gradual return of
accurate estimation of tissue perfusion. Hourly urine reflex cord activity i.e., bulbo-cavemosus reflex, anal
output measurement gives an idea of tissue perfusion. reflex and plantar reflex without improvement in
CVP and pulmonary wedge pressure helps in motor power or sensory deficit Gradually the lower
assessment of volume depletion or overload and limbs become spastic.
myocardial function.
245. The answer is D.
242. The answer is A. Zollinger-Ellison syndrome consists of ulcerative
To offer optimal management to patients with acute disease of the upper GI tract, marked increases in
renal failure, it is helpful to distinguish prerenal gastric acid secretion, and non-- islet cell tumors of
azotemia (generally managed with volume the pancreas (gastrinomas). Gastrinomas generally
replacement or amelioration of cardiac dysfunction) occur as multiple tumors in the pancreatic head and
from intrinsic renal dysfunction. Sodium reabsorption, are usually malignant, with one-third of these patients
which is quite avid in prerenal azotemia, is impaired presenting with metastatic disease. Metastasis is most
in intrinsic renal disease. However, creatinine is commonly found in the regional lymph nodes and
reabsorbed less efficiently than sodium in both liver. In 20 to 60% of those with Zollinger-Ellison syn
conditions. Therefore, the fractional excretion of drome the gastrinoma is a component of the multiple
sodium is very helpful in distinguishing between these endocrine neoplasia syndrome type I. This is an
two etiologies of renal failure. The fractional autosomal dominant disorder that is linked to
excretion of sodium is calculated by multiplying the chromosomes 11 (q ii through q13). Patients with
urine sodium by the plasma creatinine, dividing this MEN type I have neoplasms of the parathyroid

DBMCI, Ph: 9810150067, 9868387223, E-mail: drbhatiainstitute@rediffmail.com, Visit us at: www.dbmci.com 34


Grand Test – 36 Explanatory Answer

glands, pancreatic islets, and pituitary. In addition to is the usual antibody in cold agglutinins. The half-life
gastrin, most gastrinomas secrete other hormones, of IgA is about 6 days; IgE has the shortest half-life,
including ACTH, glucagon, and vasoactive intestinal approximately 2 to 2.5 days.
peptide.
249. The answer D.
246. The answer is C. A host of endocrinologic abnormalities may occur as
Patients with the more malignant variant of systemic a consequence of the loss of muscle mass and fat in
sclerosis (scleroderma) have diffuse cutaneous disease patients with severe anorexia nervosa. The disease
characterized by skin thickening in the extremities, usually begins shortly after puberty and is
face, and trunk. It is this subset of patients, in contrast characterized by profound weight loss caused by a
to those with limited cutaneous disease who often lack of caloric intake and a high level of physical
have the CREST syndrome, who are at risk for activity. Other features include cold intolerance
developing kidney and other visceral disease. caused by a secondary defect in regulatory
Hypertension heralds the onset of a renal crisis thermogenesis, hypothalamic amenorrhea,
manifested by malignant hypertension, hypokalemia, low serum immunoglobulins, normal or
encephalopathy, retinopathy, seizures, and left elevated growth hormone levels, decreased levels of
ventricular failure. The renin-angiotensin system is somatomedin C, and low serum triiodothyronine
markedly activated; therefore, angiotensin converting concentrations. Again, all these abnormalities seem to
enzyme inhibitors are particularly effective. Even result from, rather than cause, the eating disorder.
patients who require dialysis may reverse course and Most authorities favor a psychiatric etiology.
have a slow return of renal function after the passage Unfortunately, the benefits of psychiatric intervention
of several months. Patients with systemic sclerosis and behavior modification have been somewhat
may also develop esophageal dysfunction, marginal. Hospitalization may be required to save the
hypomotility of the small intestine (which can patient’s life if the anorexia nervosa is quite severe.
produce pain and malabsorption), pulmonary fibrosis
sometimes progressing to pulmonary hypertension, 250. The answer is A
and heart failure due to myo cardial fibrosis. The patient has developed pneumothorax. The best
treatment is to perform a diagnostic chest aspiration
247. The answer is A and insert a chest tube.
Once released into venous circulation,
thromboembolism is distributed to both the lungs in 251. The answer is B.
65% of cases, to the right lung in 25% and to the left The hypercalcemia of sarcoidosis is usually associated
lung in 10%. Lower lobes are involved in 4 times with disseminated disease. Therefore, almost all
more often than upper lobes. Most thromboembolism persons with sarcoidosis who have hypercalcemia also
lodge in large or intermediate (elastic or muscular) have an abnormal chest x-ray (diffuse fibronodular
pulmonary arteries; 35% or fewer reach the smaller infiltration, marked enlargement of hilar nodes, or
arteries so right lower lobe is most frequently both). This is an important point in the differential
involved. diagnosis of hypercalcemia— sarcoidosis is unlikely
as a cause of hypercalcemia if the chest x-ray is
248. The answer is C. normal. Hypergammaglobulinemia is another helpful
Immunoglobulin A is the predominant clue to the presence of sarcoidosis. The hypercalcemia
immunoglobulin in body secretions (IgG is of sarcoidosis is thought to be the consequence of
predominant in serum). Each secretory IgA molecule increased synthesis of 1, 25 (OH) vitamin D and the
is a dimer consisting of a secretory component and a J subsequent increased intestinal ab sorption of
chain. The secretory component, a protein of calcium. The elevated serum calcium concentration in
molecular weight 70,000, is synthesized by epithelial sarcoidosis causes a decreased level of serum
cells and facilitates IgA transport across mucosal parathyroid hormone, resulting in marked
tissues. The J chain is a small glycopeptide that aids hypercalciuria.
the polymerization of immunoglobulins. IgA exists as
two subclasses: IgAl (75% of the total) and IgA2 252. The answer is D.
(25% but more prevalent in secretions). IgA provides Bilateral conjugate eye movement to the side of the
defense against local infections in the respiratory, caloric stimulation indicates integrity of the brainstem
gastrointestinal, and genitourinary tracts, and prevents pathways from the medulla to the midbrain (where the
access of foreign sub stances to the general systemic third nerve originates), as do full conjugate
immune system. It also can prevent virus binding to oculocephalic motions (doll’s-eye maneuvers). The
epithelial cells. 1gM, not IgA, is the principal absence of the rapid corrective phase manifested by
immunoglobulin in the primary immune response and nystagmus-like leftward gazing indicates a bilateral

DBMCI, Ph: 9810150067, 9868387223, E-mail: drbhatiainstitute@rediffmail.com, Visit us at: www.dbmci.com 35


Grand Test – 36 Explanatory Answer

hemispheric lesion. Failure of an eye to adduct the midperineal point by attachment of Colle’s fascia
properly in the initial phase of the caloric response to the triangular ligament and by the attachment of
indicates a lesion in the ipsi lateral third nerve Scarpa’s fascia below the inguinal ligament. The urine
(midbrain) or in the medial longitudinal fasciculus collects in scrotum and penis and below the deep
producing internuclear ophthalmoplegia. In the former layer of superficial fascia in the abdominal wall.
case, the pupil would be dilated and the eye would be Treatment is urgent suprapubic cystostomy.
abducted at rest.
257. The answer is B.
253. The answer is A Tuberous sclerosis is characterized by cutaneous
TUR syndrome is mainly due to water intoxication: lesions, seizures, and mental retardation. Gene
During TUR there is absorption of water into the carriers are at an increased risk of developing
circulation resulting in CHF, dilutional hyponatraemia ependymomas as well as childhood astrocytomas,
and hemolysis. Often there is confusion and CVA like most of which are subependymal giant cell
features. astrocytomas. Patients with Von Hippel-Lindau
The incidence of TUR syndrome is reduced due to use syndrome are at an increased risk for the development
of isotonic glycine while performing resection and the of renal cell carcinoma and pheochromocytomas.
use of isotonic saline for post-operative irrigation. Patients with neurofibromatosis are at an increased
Treatment is restriction of fluid, furosemide and risk of meningiomas as well as schwannomas and
hypertonic saline. astrocytomas.

254. The answer is B. 258. The answer is D


The combination of autonomic insufficiency and In keratoconus there is myopia (progressive) and
parkinsonian symptoms is known as the Shy-Drager irregular astigmatism.
syndrome. The autonomic form of the Landry- • Munson sign V-shaped deformity of lower lid in
Guillain-Barré syndrome causes acute autonomic down gaze (due to conical cornea)
paralysis but does not cause the parkinsonian • Fleischers ring: Brown ring encircling base of cone
symptoms of tremor at rest, bradykinesia, and rigidity. due to deposition of iron.
A number of anti- hypertensive agents cause • Scissors reflex Seen in retinoscopy due to irregular
orthostatic hypotension, but none cause parkinsonism. astigmatism.
Micturition syncope is a condition in which syncope
occurs because of vagal surge at the time of release of 259. The answer is C.
intravesicular pressure. Friedreich’s ataxia (FA) is the most common of the
inherited spinocerebellar ataxias, displaying
255. The answer is D. autosomal recessive inheritance. The molecular defect
Duchenne’s muscular dystrophy is an X-linked was recently shown to involve a GAA trinucleotide
recessive disorder in which affected boys develop repeat expansion on chromosome 9. Affected persons
progressive weakness of limb girdle muscles usually present with progressive ataxia before age 25.
beginning at age 5 or earlier. By age 12 walking is Other symptoms include progressive dysarthria,
impossible, and these patients usually succumb to pyramidal-type weakness with bilateral extensor
respiratory failure by age 25. Most muscular tissues, plantar responses, posterior column sensory loss, and
including cardiac tissues, are involved. An abnormally an axonal sensory j with absent deep tendon reflexes
high creatine kinase level is found in all these patients in the lower extremities. Scoliosis and pes cavus
before disease onset and in many female carriers. The (skeletal deformities) may also be seen in these
responsible gene has been identified. This 2000-kb patients.
gene codes for a product termed dystrophin, a 400-
kDa protein localized to the muscle plasma 260. The answer is C
membrane. Since about 60% of these patients have an All the given choices are correct. Although the
exon deletion or duplication in the dystrophin gene, it incidence of neuroblastoma varies in different
is possible to test directly for these genetic standard textbooks the first statement seems to be
abnormalities in utero, obviating the need for more correct. According to Bailey and Love about 90%
cumbersome family studies to determine RFLPs for cases occur before 8 years of age.
linkage. The serum level of catecholamines and their
byproducts (homovanillic acid, vanillylmandelic acid,
256. The answer is D 3-methyltyrosine, metanephrine, dopamine), ferritin
Superficial extravasation of urine occurs in complete and rarely acetylcholine are raised. 24 hour urinary
rupture of bulbar urethra and in ruptured urethral VMA is elevated in more than 85% cases. Nelson
abscess. The extravasated urine is confined in front of mentions that norepinephrine is the predominant

DBMCI, Ph: 9810150067, 9868387223, E-mail: drbhatiainstitute@rediffmail.com, Visit us at: www.dbmci.com 36


Grand Test – 36 Explanatory Answer

catecholamine in children. Hence it is clear that However according to Bailey and Love the best
norephinephrine level will also be raised. results are obtained by combination of surgery and
Spontaneous regression of tumour seen in many postoperative interstitial radiotherapy.
patients has led to the investigation of immunotherapy Treatment of keloid:
as a possible therapeutic tool. • Many keloids are untreatable and surgical treatment,
According to Bailey and Love it is the most common as single modality will be met with recurrence.
solid tumour of infancy and childhood and by far the • Some keloid will also improve with the application
most common in the newborn. Surgery and of pressure.
chemotherapy are the main treatment. • Intralesional steroids such as triamcinolone can be
Neuroblastoma, a common tumour of neural crest useful.
origin, may undergo spontaneous regression or
differentiation into benign ganglioneuroma. 264. The answer is D.
• About 90% of cases are diagnosed before the age of Case-control studies do not involve an observational
5 years. period, as do cohort studies. The disease in question
• Neuroblastoma may resemble small round cell has already occurred and the study is interested in
tumours such as rhabdomyosarcoma, Ewing’s examining antecedent exposures. Conducting field
sarcoma and non-Hodgkin’s lymphoma. operations or pilot studies is also useful in planning
• Tumour markers are elevated in 95% cases and help and refining study protocols.
in diagnosis [homovanillic acid (HVA) and vanillyl
mandelic acid (VMA)]. 265. The answer is B.
Detecting fatal conditions that have no treatment or
261. The answer is D. improved prognosis from early detection and
The correct values for mean, median, and mode are intervention is not an effective use of time or money.
3.4, 3, and 2. The mean is the average: the sum of the Limited public resources are best spent on health
observations divided by the number of observations. conditions that have the most impact on public health.
In this case, the mean is 31/9 = 3.4. The median is the Studies that track and describe Alzheimer’s disease,
middle observation in a series of ordered however, are valid epidemiologic research.
observations, i.e., the 50th percentile (when the
number of observations is even, it is midway between 266. The answer is C
the two middle observations). In this case, when the Aponeurosis of rectus sheath is very strong and
observations are ordered—I, 2, 2, 2, 3, 4, 4, 6, 7—the provides maximum strength of intact abdominal wall.
median is 3. The mode is the observation that occurs
with greatest frequency; in this case it is 2, which 267. The answer is D.
occurs three times. The most important risk factor for breast cancer (like
most cancers) is age the rate in women 75 to 84 years
262. The answer is C. old is about 50 times that of women 35 to 44 years
The chi-square is used if no cell has an expected count old. If crude incidence rates are compared (new cases
less than 1, and no more than 20 percent of the cells per 100,000 adult women), one country may have
have an expected count less than 5. In this case the much larger numbers of women in the peak risk
expected counts are 4.2 (10 X 8 / 19) for cell a, 5.7 groups and have a much higher incidence for that
(10 X 11 / 19) for cell b, 3.7(9 X 8 / 19) for cell c, and reason. Therefore, either comparison of the age-
5.2 (9 X 11 I 19) for cell d. Because 50 percent of the specific rates for each age group or else some type of
cells have an expected count of less than 5, the age adjustment is essential. Although nursing may
Fisher’s exact test is appropriate. In general, it is used have a protective effect on breast cancer, it is of
when the sample size is small. The McNemar’s test is nowhere near the magnitude of the effect of age.
used for paired dichotomous data, the student t for Cigarette smoke is not a major risk factor for breast
independent continuous data, and the analysis of cancer. Early diagnosis, if it had any effect, would be
variance is for analysis of several independent means. expected to increase the incidence rate (since some
cases might be discovered that otherwise might
263. The answer is A spontaneously resolve or not be noticed before the
According to Schwartz perhaps the most effective woman died of another cause). Finally, efficacy of
treatment of keloid is intralesional injection of treatment might affect the death rate, but would not
trimamciolone which is a long lasting 9-- affect the incidence of the disease.
fluorocortiocosteroid. For small lesions it is often
sufficient. 268. The answer is D.
Acute episodes of asthma have been associated with a
wide variety of triggers, including dust, animal

DBMCI, Ph: 9810150067, 9868387223, E-mail: drbhatiainstitute@rediffmail.com, Visit us at: www.dbmci.com 37


Grand Test – 36 Explanatory Answer

dander, respiratory infections, ozone pollution, 273. The answer is C


aspirin, and emotional factors. Asthma is an episodic Sharply circumscribed opacity rules out malignancy.
disease of the airways characterized by increased Anechoic lesion in ultrasound means fluid filled cyst/
responsiveness of the tracheobronchial tree to these cavity. Hence the most probable diagnosis is either
and other stimuli. There are allergic and idiosyncratic polycystic disease or parasitic cystic disease
forms of asthma. Caffeine is a methylxanthine related (hydatid). Hence the investigation of choice is CT
to theophylline and is a low-potency bronchodilator. scan of abdomen. It will reveal the presence of cysts
in other organs like kidney.
269. The answer is A
The patient is suffering from pituitary adenoma, most 274. The answer is D.
likely prolactinoma. It secretes prolactin, which Persons with Klinefelter’s syndrome have a 47,XXy
causes amenorrhoea. karyotype and are typically tall and eunuchoid in
Pituitary adenoma may press over optic chiasma appearance with gynecomastia, testicular atrophy, and
leading to visual disturbance (bitemporal hemianopia) a female pattern of hair distribution Patients with
Turner’s syndrome are 45,X or are mosaic and have a
270. The answer is A. female phenotype. Trisomy 21 in either sex presents
Following an acute exposure to 100 to 800 rem (1 to 8 with characteristic facial features and invariable
Sv) of ionizing radiation, hematopoietic disturbances mental retardation. Testicular feminization syndrome
occur. Between 800 and 3000 rem (8 and 30 Sv), results from defective or absent androgen receptors
gastrointestinal effects pre dominate, and fatal central throughout the body. These patients are 46,XY, but
nervous and cardiovascular effects occur at an have a female phenotype.
exposure of over 3000 rem (30 Sv). Few effects are
seen with an exposure of less than 100 rem (1 Sv). 275. The answer is B.
Sprengel’s deformity is congenital elevation of the
271. The answer is C scapula due to failure of the normal caudal migration
• Presenile cataract: It refers to cataract formation of the scapula. In this deformity the scapula is usually
before senility. Occurs in Myotonic dystrophy, elevated 2 to 10 cm and adducted with its inferior pole
Diabetes mellitus and Atopic eczema. medially rotated. The medial rotation results in an
Immature cataract: Whether posterior subcapsular abnormal position of the superomedial angle of the
or not can be extracted by phacoemulsification. scapula, which causes the ipsilateral side of the neck
• Black cataract Phacoemulsification not preferred as to appear fuller with loss of the normal contour. The
background glow is not visible due to cataract and rotation also causes the glenoid cavity to face
hence capsulorrhexis not possible. downward. This rotation combined with a relatively
fixed scapula limits abduction to less than 1000 in 40
272. The answer is B. percent of the patients with this condition. External
Physiologic bowleg occurs in the first 2 years of life. and internal rotation are normal.
Lateral bowing of the tibia is seen in the first year and
bowleg involving both the tibia and distal femur 276. The answer is C
occurs during the second year. Physiologic knock- Treatment of acute myositis ossificans is rest and
knee is most pronounced between 3 and 4 years of analgesics.
age. Physiologic variations are common and most
resolve with time. Pathologic forms are uncommon 277. The answer is A
and generally do not resolve. The physician should In carpal tunnel syndrome, mere occurs tingling in
assess growth, nutrition, symmetry, degree of hand when the wrist is fully flexed. This is positive
deformity, an other abnormalities Blount’s disease is Phalen 's test.
tibia vara, which results from abnormal growth of the
proximal medial physis and metaphysis. It is more 278. The answer is D.
common in blacks and obese children. Medial tibial Signs of complete airway obstruction include
torsion often accompanies the deformity. An clutching the neck with thumb and fingers (universal
asymmetric deformity with a history of trauma may distress signal) and inability to speak, breathe, or
suggest malunion or partial physeal arrest. Rickets cough. With a complete obstruction, there is no air
should be suspected in a patient with progressive exchange, and therefore patients generally will make
varus deformity whose stature is below the 5th no sounds with attempted breathing. A partial
percentile. The diagnosis is confirmed by a low obstruction may cause a weak, ineffective cough;
calcium and phosphorus and high alkaline high-pitched noise with inhalation; and respiratory
phosphatase activity. difficulty.

DBMCI, Ph: 9810150067, 9868387223, E-mail: drbhatiainstitute@rediffmail.com, Visit us at: www.dbmci.com 38


Grand Test – 36 Explanatory Answer

279. The answer is B. Tricuspid regurgitation causes a systolic murmur best


The patient is describing a transient ischemic attack heard at the left sternal border in the fourth and fifth
(TIA). These attacks occur suddenly and produce interspaces. It does not radiate to the axilla. The
reversible, unilateral visual loss or neurologic deficits. regurgitated blood causes prominent systolic waves in
The attacks last from a few minutes to a few hours. the jugular veins. Inspiration increases the murmur of
These attacks are produced by emboli that occlude the tricuspid regurgitation.
retinal artery and may be associated with carotid
athero sclerosis, which may be the source of emboli. 286. The answer is D
Auscultation of the carotids may reveal a bruit. Blood gas partition coefficients
Seroflurane-0.69
280. The answer is B Desiflurane-0.42
• Most common type of dislocation of hip is posterior
dislocation. 287. The answer is C
• Most common type of dislocation of shoulder is • Longest acting local anaesthetic is bupivacaine. Its
anterior dislocation. duration of action is 5-16 hours.

281. The answer is C 288. The answer is B.


• Catterall’s classification of Perthes' disease Early detection of abdominal aortic aneurysms
Group 1 - Epiphysis has retained its height and less (AAAs) has been advocated to decrease the high
than half the head is sclerotic. mortality of ruptured AAAs. In a study of 243 patients
Group 2 - Upto half the head is sclerotic and there with AAAs, only 38 percent were detected by
may be some collapse of the central portion physical examination (PE). Forty-three percent of
of head. patients with AAAs detected on radiologic
Group 3 - Most of head is involved and there is examination had palpable AAAs and should have
sclerosis, fragmentation and collapse of the been detected on PE. Twenty-three percent of AAAs
head. were not palpable on PE, even when the diagnosis
Group 4 - Whole head is involved, with marked was known. Obese patients had only 15 percent of
metaphyseal resorption. AAAs detected by PE, and only 33 percent were
palpable.
282. The answer is D.
Bell’s palsy is the loss of motor function of cranial 289. The answer is B.
nerve VII to the ipsilateral side of the face. This Vesicular breath sounds are low-pitched and of low
causes drooping of the mouth and facial muscles, intensity and are sometimes described as “breezy.”
inability to close the ipsilateral eye, and difficulty They are heard over the lesser bronchi, bronchioles,
speaking and eating. With the drooping of the mouth, and lobes. Bronchovesicular breath sounds are
the patient may have trouble with drooling from the normally heard over the main bronchi and are a
corner of the mouth on the affected side. Abduction of combination of sounds made from normal vesicular
the eye is controlled by cranial nerve VI. breath sounds and bronchial breath sounds. They are
likened to “air passing through a tube” and are highest
283. The answer is B in pitch and intensity. Vesicular breath sounds are
Polyostotic fibrous dysplasia with precocious puberty normally heard over the apices of the lungs.
and cutaneous pigmentation is seen in McCune
Albright Syndrome. 290. The answer is A
• Route of administration for patient controlled
284. The answer is B. analgesia (usually opioid analgesia) is intravenous.
The triad of nystagmus, ataxia, and confusion is
associated with the Wernicke-Korsakoff syn drome, 291. The answer is C
often seen in poorly nourished alcoholics. While • Post spinal hypotension in obstetrics patients is
strokes and encephalitides may present with a treated by:
confusional state and motor dysfunction, they would 1. Crystalloid infusion
be less likely than thiamine deficiency in an indigent 2. Elevation of legs
patient. Niacin deficiency (pellagra) classically 3. Lateral tilt
presents with the triad of dermatitis, diarrhea, and 4. Ephedrine and oxygen administration
dementia.
292. The answer is C.
285. The answer is D. Peritonitis often results in abdominal rigidity
(guarding). Findings may range, however, from a

DBMCI, Ph: 9810150067, 9868387223, E-mail: drbhatiainstitute@rediffmail.com, Visit us at: www.dbmci.com 39


Grand Test – 36 Explanatory Answer

persistently stiff, motionless abdomen to stiffness innervates the muscles that extend the elbow, wrist,
only with palpation to a completely flaccid abdomen. and fingers.
When a patient raises the head from the bed to touch
the chin to the chest, the abdominal muscles contract 297. The answer is B.
and guarding is induced. This protects the abdominal Tennis elbow is most commonly characterized by
contents from the palpating hand and may reduce the tenderness of the common extensor muscles at their
amount of tenderness elicited by palpation. Rebound origin, the lateral epicondyle of the humerus. Having
tenderness is tested by depressing the abdominal wall the patient extend the wrist against resistance causes
and then abruptly withdrawing the examining hand. pain. Passive flexion of the fingers and wrist causes
The abdominal wall passively springs back into place, tension of the extensor muscles and pain.
carrying with it the inflamed peritoneum. The test is
positive if the rebound portion of this test causes 298. The answer is A
severe pain. The referred rebound test is conducted in Involvement of pre-epiglottic space is seen in stage T3
the same fashion but in a location away from the area and T4 of supraglottic carcinoma larynx.
of tenderness. The patient will experience pain in the
area of stated tenderness, rather than the site where 299. The answer is A
the test is performed. Perforation of tympanic membrane produces only
mild hearing loss (10-40 dB) of conductive type.
293. The answer is A.
Lesions of the right colon commonly ulcerate, which 300. The answer is B.
leads to chronic, insidious blood loss without a The crossed straight-leg raising test is performed with
change in the appearance of the stools. Consequently, the patient in the supine position. The test is
patients with tumors of the ascending colon often considered positive when straight-leg raising of the
present with symptoms such as fatigue and nonpainful leg worsens the pain on the other side. A
palpitations and are found to have a hypochromic, positive test is indicative of disk herniation.
microcytic anemia indicative of iron deficiency. Stool
is relatively liquid as it passes through the right colon;
however, it becomes more concentrated as it
progresses into the transverse and descending colon,
Thus, tumors arising in the left colon tend to impede
the passage of stool and may present with a picture of
obstruction and even perforation. Radiographs of the
abdomen often reveal the characteristic annular
constricting lesions (“apple-core,” or “napkin ring”).
Neoplasms of the rectosigmoid often are associated
with hematochezia, tenesmus, and narrowing in the
caliber of the stool; nonetheless, anemia is an
infrequent finding.

294. The answer is D


Most common cause of hypoxaemia in immediate
post operative period is hypoventilation.
• Causes of hypoventilation in post operative period:
1. Pain
2. Obstructed airway
3. Residual effect of anaesthetic
4. CO2 narcosis
5. Shock

295 The answer is D


Sudden decrease in end-tidal CO2 as shown by
capnography suggests accidental extubation.

296. The answer is D.


The musculocutaneous nerve (C5-C7) supplies the
anterior compartment of the arm and is responsible for
flexion of the elbow. The radial nerve (C5-C8)

DBMCI, Ph: 9810150067, 9868387223, E-mail: drbhatiainstitute@rediffmail.com, Visit us at: www.dbmci.com 40

Вам также может понравиться